Está en la página 1de 263

TEORÍA DE NÚMEROS

Problem-solving

Gerard Romo Garrido


Toomates Coolección
Los documentos de Toomates son materiales digitales y gratuitos. Son digitales porque están pensados para ser consultados mediante un
ordenador, tablet o móvil. Son gratuitos porque se ofrecen a la comunidad educativa sin coste alguno. Los libros de texto pueden ser digitales o
en papel, gratuitos o en venta, y ninguna de estas opciones es necesariamente mejor o peor que las otras. Es más: Suele suceder que los mejores
docentes son los que piden a sus alumnos la compra de un libro de texto en papel, esto es un hecho. Lo que no es aceptable, por inmoral y
mezquino, es el modelo de las llamadas "licencias digitales" con las que las editoriales pretenden cobrar a los estudiantes, una y otra vez, por
acceder a los mismos contenidos (unos contenidos que, además, son de una bajísima calidad). Este modelo de negocio es miserable, pues
impide el compartir un mismo libro, incluso entre dos hermanos, pretende convertir a los estudiantes en un mercado cautivo, exige a los
estudiantes y a las escuelas costosísimas líneas de Internet, pretende pervertir el conocimiento, que es algo social, público, convirtiéndolo en un
producto de propiedad privada, accesible solo a aquellos que se lo puedan permitir, y solo de una manera encapsulada, fragmentada,
impidiendo el derecho del alumno de poseer todo el libro, de acceder a todo el libro, de moverse libremente por todo el libro.
Nadie puede pretender ser neutral ante esto: Mirar para otro lado y aceptar el modelo de licencias digitales es admitir un mundo más injusto, es
participar en la denegación del acceso al conocimiento a aquellos que no disponen de medios económicos, en un mundo en el que las modernas
tecnologías actuales permiten, por primera vez en la historia de la Humanidad, poder compartir el conocimiento sin coste alguno, con algo tan
simple como es un archivo "pdf". El conocimiento no es una mercancía.
El proyecto Toomates tiene como objetivo la promoción y difusión entre el profesorado y el colectivo de estudiantes de unos materiales
didácticos libres, gratuitos y de calidad, que fuerce a las editoriales a competir ofreciendo alternativas de pago atractivas aumentando la calidad
de unos libros de texto que actualmente son muy mediocres, y no mediante retorcidas técnicas comerciales.
Este documento se comparte bajo una licencia “Creative Commons”: Se permite, se promueve y se fomenta cualquier uso, reproducción y
edición de todos estos materiales siempre que sea sin ánimo de lucro y se cite su procedencia. Todos los documentos se ofrecen en dos
versiones: En formato “pdf” para una cómoda lectura y en el formato “doc” de MSWord para permitir y facilitar su edición y generar versiones
parcial o totalmente modificadas. Se agradecerá cualquier observación, comentario o colaboración a toomates@gmail.com

La biblioteca Toomates Coolección consta de los siguientes libros:


Bloques temáticos: Problem-solving Libros de texto (en catalán)
Geometría Axiomática pdf 1 2 ... 23
Problemas de Geometría pdf 123456789
Introducción a la Geometría pdf doc
Teoría de números pdf 1234
Trigonometría pdf doc pdf doc
Desigualdades pdf doc
Números complejos pdf doc pdf doc
Álgebra pdf doc pdf 1234
Combinatoria pdf doc
Probabilidad pdf doc
Guía del estudiante de Olimpiadas Matemáticas pdf
Combinatòria i Probabilitat pdf doc
Estadística pdf doc
Funcions pdf doc
Geometria analítica pdf 12
Àlgebra Lineal 2n batxillerat pdf doc
Geometria Lineal 2n batxillerat pdf doc
Càlcul Infinitesimal 2n batxillerat pdf 12
Programació Lineal 2n batxillerat pdf doc
Recopilaciones de pruebas PAU:
Catalunya TEC , Catalunya CCSS , Galicia , Portugal A , Portugal B
Recopilaciones de problemas olímpicos y preolímpicos (España):
OME , OMEFL , OMEC , OMEM , Canguro , Cangur
Recopilaciones de problemas olímpicos y preolímpicos (Internacional):
IMO , OMI , USAMO , AIME , AMC 8 , AMC 10, AMC 12 , SMT
Kangourou, Kangaroo , Mathcounts

Versión de este documento: 20/06/2021

Todos estos documentos se actualizan constantemente. ¡No utilices una versión anticuada! Descarga totalmente gratis la última versión de los
documentos en los enlaces superiores.

www.toomates.net
Índice.

Primera parte. Primeros pasos con números enteros.

1 Principios. →
Bases de numeración. El principio de buena ordenación. El principio de inducción.
2 Problemas de la primera parte. →

Segunda parte (Divisibilidad)

3 Divisibilidad. →
El algoritmo de la división.
4 Máximo común divisor y mínimo común múltiplo. →
Números coprimos. El Teorema de Bezout. El algoritmo de Euclides.
5 Números primos. →
5.1 Concepto de número primo.
5.2 El Teorema fundamental de la aritmética (TFA)
5.3 Resolución de problemas mediante identidades algebraicas.
5.4 Resolución de problemas mediante factorización de polinomios.
6 Problemas de la segunda parte. →

Tercera parte (Congruencias lineales y polinómicas)

7 Aritmética modular. →
8 Congruencias y sistemas de congruencias lineales. →
Congruencias lineales. Sistemas de congruencias lineales. El Teorema chino del residuo.
9 Congruencias cuadráticas. →
10 Ecuaciones diofánticas. →
10.1 Ecuaciones diofánticas lineales.
10.2 Ternas pitagóricas.
10.3 La ecuación diofántica x  y  k .
2 2

10.4 La técnica del descenso infinito de Fermat.

11 Problemas de la tercera parte. →


Cuarta parte (Congruencias exponenciales)

12 El pequeño teorema de Fermat. →


13 La función Phi de Euler. →
Teorema de Euler.
14 Orden de un entero. →
15 Congruencias exponenciales. →
16 Problemas de la cuarta parte. →

Quinta parte.

17 Números factoriales. →
La función suelo. Números factoriales. Orden p-ádico. La fórmula de Polignac.
18 Números combinatorios. →
19 Números y primos de Fermat y de Mersenne. →
20 Número de divisores de un entero. →
El producto de los divisores de un entero.
21 Suma de los divisores de un entero. →
Números perfectos.

Soluciones. →

Fuentes.

El capítulo 14 del dossier de Desigualdades està dedicado a la aplicación de las desigualdades en la resolución de
ecuaciones.
Descarga de www.toomates.net/biblioteca/Olimpiadas.pdf una guía general de las Olimpiadas Matemáticas y sus
contenidos curriculares.
1 Principios.
La "Teoría de números" o "aritmética" estudia las propiedades de los números enteros.
Los conceptos teóricos de esta rama de las matemáticas pueden ser complicados, muy
complicados y terriblemente complicados. Sin embargo, muchos problemas se
resuelven con solo utilizar el sentido común, toda una serie de estrategias y conceptos
que, de tan obvios que son, los libros de teoría no dedican tiempo a explicarlos.

En este primer apartado se incluyen problemas cuya resolución no necesita ningún


concepto teórico previo, sólo el sentido común, la pura lógica, y algunas formulitas de
la matemática elemental. Sin embargo, no hay que despreciarlos. Es fundamental que el
estudiante dedique a cada problema tanto tiempo como sea necesario, y si no llega a
resolverlo, estudie detenidamente la solución que se presenta al final del libro.

Bases de numeración.
1.1 Definición. Base de numeración.
Diremos que n se escribe como an an 1 an  2 ... a1 a0 en base b  2 si
n  anbn  an 1bn 1  an  2bn  2  ...  a1b  a0

con 0  ai  b , ai  IN , an  0 .

Por ejemplo, 3562 en base 7 es el número 3  73  5  72  6  7  2  1318 , y se escribe


35627 .

1.2 MF
¿Cuál de los siguientes enteros se puede expresar como la suma de 100 enteros positivos
consecutivos?

(A) 1,627,384,950 (B) 2,345,678,910 (C) 3,579,111,300 (D) 4,692,581,470 (E) 5,815,937,260

ASHME 1997 #20

1.3 M
Consideremos el entero N  9  99  999  9999  ..  99
...
99
321 cifras

Calcula la suma de todas las cifras de N.

AIME I 2019 #1

1.4 M
Para cada entero positivo n , sea d n la cifra de las unidades de 1  2  3  ..  n .
2017
Determina el residuo cuando d
n 1
n se divide entre 1000.

AIME I 2017 #3
1.5 MF
Sea a0  2 , a1  5 , a2  8 , y para cada n  2 , define an recursivamente como el
residuo cuando 4an 1  an  2  an 3  se divide entre 11. Determina a2018  a2020  a2022 .

AIME II 2018 #2

1.6 F
Multiplicamos todos los números pares del 2 al 98 inclusive, excepto aquellos acabados
en 0. ¿Cuál será la cifra de las unidades del resultado?

(A) 0 (B) 2 (C) 4 (D) 6 (E) 8

AMC10 1999 Sample #14

1.7 M
Demostrar que si entre los infinitos términos de una progresión aritmética de números
enteros hay un cuadrado perfecto, entonces infinitos términos de la progresión son
cuadrados perfectos.

OME 1993-94 (Primera sesión) #1

1.8 MF
Un entero positivo N se representa como a b c en base 11, y se representa como 1b c a ,
donde a, b, c representan dígitos, no necesariamente distintos. Determina el valor
mínimo de N expresado en base 10.

AIME I 2020 #3

Orden en los números enteros.


1.9 Principio de la buena ordenación.
Todo conjunto S de números enteros no negativos contiene un elemento mínimo, es
decir, existe un elemento a  S tal que a  b para todo b  S .

1.10 D
a 2  b2
Demostrar que, si a, b son enteros positivos tales que es un entero, entonces
1  ab
a 2  b2
es un cuadrado perfecto.
1  ab
IMO 1988
1.11 Teorema. Propiedad arquimediana de los números naturales.
Si a y b son números enteros positivos, entonces existe un entero positivo n tal que
na  b.

Demostración. Supongamos que no es cierto, es decir, que existen dos números a, b  0


tales que n a  b para todo n  0 . Consideremos el conjunto S  b  na , n  0 .
Está claro que es un subconjunto de números enteros positivos, pues
n a  b  0  b  na , y por tanto le podemos aplicar el Principio de la buena
ordenación, es decir, contendrá un elemento mínimo b  m a , para cierto m  0 .
Pero, por hipótesis, b  (m  1) a también pertenecerá a S, luego:
b  (m  1) a  b  ma  a  b  ma , pues a  1 , luego b  m a no puede ser el mínimo,
llegando a contradicción.
Así pues, la propiedad arquimediana de los números naturales debe ser cierta, pues su
negación nos lleva a contradicción.

1.12 Principio de Inducción.


Sea S un conjunto de números enteros positivos cumpliendo las dos condiciones
siguientes:
a) 1 pertenece a S.
b) Si n  S , entonces n 1 S
Entonces S es el conjunto de todos los enteros positivos: S  1, 2 , 3, ... 

Demostración. Sea T  1, 2 , 3, ...  S , y supongamos que T   , es decir, que no está


vacío, o lo que es lo mismo, que no se cumple el Principio de Inducción.
Aplicando el Principio de la buena ordenación, T contendrá elemento mínimo,
llamémosle a .
Puesto que 1 T , pues por hipótesis, 1 S , está claro que a  1 , luego 0  a  1  a .
El número a  1 tampoco pertenecerá a S, pues si a  1 S  a  1  1  a  S ,
contradiciendo la hipótesis. Pero a 1  a , llegando a contradicción, pues habíamos
supuesto que a era mínimo.

El principio de inducción es una herramienta muy poderosa para demostrar fórmulas


que nos serán muy útiles para solucionar una enorme variedad de problemas.

1.13 Ejemplo.
n(2n  1)(n  1)
12  22  32  ...  n 2  para todo n  1, 2 , 3, ...
6

Demostración. Sea S el conjunto de números enteros positivos para los que la fórmula
anterior es cierta.
Está claro que 1 pertenece a S, pues
1(2  1  1)(1  1)
12  1 
6
Supongamos que la fórmula anterior se cumple para un cierto valor n , y veamos que,
entonces, se cumplirá también para n  1 :
n(2n  1)(n  1)  n(2n  1) 
12  22  32  ...  n 2  n  1   (n  1) 2  (n  1)  (n  1)  
2

6  6 
 n(2n  1)  6(n  1) 
 (n  1)   (*)
 6 
n(2n  1)  6(n  1)  2n  n  6n  6  2n2  7n  6  (n  2)(2n  3)
2

 (n  2)(2n  3)  (n  1)(n  2)(2n  3) (n  1)(2(n  1)  1)(n  1  1)


(*)  (n  1)   
 6  6 6
k (2k  1)(k  1)
 , tomando k  n  1 , luego la fórmula también es válida para n  1 .
6
Así pues, S  1, 2 , 3, ...  , es decir, la fórmula es válida para todos los enteros positivos.
2 Problemas de la primera parte.
2.1 F
Tenemos tres cartulinas y en cada una se ha escrito un número de cinco cifras. Como se
ve en la figura tres de las cifras están tapadas. La suma de los tres números es 57263.
¿Cuáles son las cifras ocultas?

A) 0, 2 y 2 B) 1, 2 y 9 C) 2, 4 y 9 D) 2, 7 y 8 E) 5, 7 y 8

CANGURO 2019 N5 #10

2.2 F
Los enteros positivos a, b y c tienen cada uno tres cifras, y en cada entero la primera
cifra es la misma que la última. También cumplen que b  2a  1 y c  2b  1 . ¿Cuántos
valores distintos hay para el entero a?

A) 0 B) 1 C) 2 D) 3 E) más de 3

CANGURO 2019 N5 #26

2.3 F
Tenemos tres cartulinas y en cada una se ha escrito un número de cuatro cifras. Como se
ve en la figura tres de las cifras están tapadas.

La suma de los tres enteros de cuatro cifras es 11126. ¿Cuáles son las cifras ocultas?

A) 1, 4 y 7 B) 1, 5 y 7 C) 3, 3 y 3 D) 4, 5 y 6 E) 4, 5 y 7

CANGURO 2019 N6 #6

2.4 F
¿Cuál es la primera cifra (la situada más a la izquierda) del número entero positivo más
pequeño cuyas cifras suman 2019?

A) 2 B) 3 C) 4 D) 5 E) 6

CANGURO 2019 N6 #7
2.5 F
Un gráfico consta de 16 vértices y algunos segmentos que los conectan, como en la
imagen.

Ahora hay una hormiga en el vértice A. En cada movimiento, puede caminar desde un
vértice a cualquier vértice vecino a lo largo de un segmento de conexión.
¿En cuál de los vértices P, Q, R, S, T puede estar la hormiga después de 2019
movimientos?

A) sólo P, R o S, no Q y T B) sólo P, R, S o T, no Q C) sólo Q D) sólo T E) en


cualquiera

CANGURO 2019 N5 #25

2.6 M
Dado un entero positivo n , sea f (n) la suma de los dígitos de la representación de n
en base cuatro, y sea g (n) la suma de los dígitos de la representación de f (n) en base
ocho.
Por ejemplo: f (2020)  f 1332104   10  128 , y g (2020)  1  2  3 .
Determina el valor mínimo de n de forma que la representación en base 16 de g (n) no
pueda ser representada usando solo los dígitos 0 a 9.

AIME II 2020 #5

2.7 MF
Los 25 enteros entre -10 y 14, inclusive, se pueden organizar para formar un cuadrado
de 5 por 5 en el que la suma de los números de cada fila, de cada columna y de las dos
diagonales sumen lo mismo. ¿Cuál es el valor de esta suma común?

(A) 2 (B) 5 (C) 10 (D) 25 (E) 50

AMC 12A 2020 #5

2.8 M
Determina el número de pares ordenados ( x, y) de enteros que satisfacen la ecuación
x 2020  y 2  2 y

(A) 1 (B) 2 (C) 3 (D) 4 (E) hay infinitos pares


AMC 12B 2020 #8
2.9 M
Sean a y b números reales positivos cumpliendo la condición

log a  log b  log a  log b  100

y en donde los cuatro términos de la izquierda son enteros positivos, denotando por log
el logaritmo en base 10. Determina ab .

(A) 10 52 (B) 10100 (C) 10144 (D) 10164 (E) 10 200

AMC 12A 2019 #15

2.10 F
Determina la cantidad de números enteros no negativos que se pueden escribir de la
forma
a7  37  a6  36  a5  35  a4  34  a3  33  a2  32  a1  31  a0  30

donde ai    1 , 0 , 1  para todo 0  i  7 .

(A) 512 (B) 729 (C) 1094 (D) 3281 (E) 59048

AMC 12A 2018 #13

2.11 M
Sea el triángulo ABC de lados AB  9 , BC  5 3 y AC  12 . Marcamos los puntos
A  P0 , P1 , P2 ,..., P2450  B en el segmento AB de forma que Pk se encuentra entre Pk 1
y Pk 1 para todo k  1,2,...,2449 , y marcamos los puntos A  Q0 , Q1 , Q2 ,..., Q2450  C en
el segmento AC de forma que Qk se encuentra entre Qk 1 y Qk 1 para todo
k  1,2,...,2449 . Además, todo segmento Pk Qk , con k  1,2,...,2449 , es paralelo a BC .
Estos segmentos cortan el triángulo en 2450 regiones, consistiendo en 2449 trapecios y
un triángulo. Todas estas 2450 regiones tienen el mismo área. Determina el número de
segmentos Pk Qk , con k  1,2,...,2450 cuya longitud es racional.

AIME II 2018 #7

2.12 F
Se toma aleatoriamente un número m del conjunto 11,13 ,15 ,17 ,19 , y se toma
aleatoriamente un número n del conjunto 1999 , 2000 , 2001, ..., 2018 . Determina la
probabilidad de la cifra de las unidades de m n sea 1.

1 1 3 7 2
(A) (B) (C) (D) (E)
5 4 10 20 5
AMC 10A 2018 #19

2.13 D
Utilizando la igualdad 732  342  5  1297 , escribe 1297 como suma de dos cuadrados.
2.14 M
Determina la expresión equivalente a

2  322  32 24  34 28  38 216  316 232  332 264  364 


(A) 3127  2127 (B) 3127  2127  2  363  3  263 (C) 3128  2128 (D) 3128  3128 (E) 5127

AMC 10 2021 #10

2.15 MF
La suma de los cinco números de tres cifras ABC, BCD, CDE, DEA y EAB es 2664.
¿Cuál es el valor de la suma de las cifras A, B, C, D y E?

A) 4 B) 14 C) 24 D) 34 E) 44

CANGURO 2020 N6 #5

2.16 MF
Sean a, b y c enteros que satisfacen 1  a  b  c y a  b  c  1000000 . ¿Cuál es el mayor
valor posible de b?

A) 100 B) 250 C) 500 D) 1000 E) 2000

CANGURO 2020 N6 #7

2.17 F
Sean a, b y c tres números enteros. ¿Cuál de los siguientes valores nunca puede ser igual
a a  b  (b  c)2  (c  a)2 ?
2

A) 0 B) 1 C) 2 D) 6 E) 8
CANGURO 2020 N6 #14

2.18 F
El número entero 29...... tiene 100 cifras. ¿Cuántas cifras tiene su cuadrado?

A) 101 B) 199 C) 200 D) 201 E) no se puede saber

CANGURO 2020 N6 #15

2.19 F
La sucesión f n viene dada por f1  1 , f 2  3 y f n  2  f n  f n 1 para n  1 . ¿Cuántos de
los primeros 2020 términos de la sucesión son pares?

A) 673 B) 674 C) 1010 D) 1011 E) 1347

CANGURO 2020 N6 #18


2.20 MF
En los cálculos mostrados, cada letra representa una cifra y se usan para formar algunos
números de dos cifras.

Los dos números de la izquierda suman 79. ¿Cuál es la suma de los cuatro números de
la derecha?

A) 79 B) 158 C) 869 D) 1418 E) 7979

CANGURO 2020 N5 #5

2.21 MF
La suma de cuatro enteros consecutivos es 2. ¿Cuál es el menor de estos enteros?

A) -3 B) -2 C) -1 D) 0 E) 1

CANGURO 2020 N5 #6

2.22 MF
Los años 2020 y 1717 se forman con un número de dos cifras repetido dos veces.
¿Cuántos años transcurrirán a partir de 2020 para encontrarnos en el siguiente año que
tenga esta misma propiedad?

A) 20 B) 101 C) 120 D) 121 E) 202

CANGURO 2020 N5 #7

2.23 MF
El camino más corto para ir de A a C pasa por B. Paseando por este camino de A a C,
primero encontramos en el lado izquierdo del camino el poste de señales que se muestra
a la izquierda de la figura, y después, en el lado derecho del camino el otro poste.

¿Qué distancia estaba escrita en el cartel roto?

A) 1 km B) 2 km C) 3 km D) 4 km E) 5 km

CANGURO 2020 N5 #13


2.24 F
En cada una de las nueve celdas de la figura se escribe un número de modo que la suma
de los tres números en cada diámetro es 13 y la suma de los ocho números en la
circunferencia es 40.

¿Qué número debe escribirse en la celda central?

A) 3 B) 5 C) 8 D) 10 E) 12

CANGURO 2020 N5 #15

2.25 MF
Lucas comienza un viaje de 520 km en coche con 14 litros de combustible en el
depósito. Su automóvil consume 1 litro de combustible por cada 10 km. Después de
conducir 55 km, lee una señal de tráfico que muestra las distancias desde ese punto
hasta cinco estaciones de servicio en la carretera. Estas distancias son 35 km, 45 km, 55
km, 75 km y 95 km. La capacidad del depósito de combustible del coche es de 40 litros
y Lucas quiere detenerse solo una vez para llenarlo. ¿A qué distancia está la estación de
servicio donde debe repostar?

A) 35 km B) 45 km C) 55 km D) 75 km E) 95 km

CANGURO 2020 N5 #18

2.26 F
Carmen etiquetó los vértices de una pirámide de base cuadrada con los números 1, 2, 3,
4 y 5, uno para cada vértice. Para cada cara calculó la suma de los números en sus
vértices. Cuatro de estas sumas son 7, 8, 9 y 10. ¿Cuál es la quinta suma?

A) 11 B) 12 C) 13 D) 14 E) 15
CANGURO 2020 N5 #25
2.27 F
En cada una de las celdas de una tabla se escribe un número, de manera que las sumas
de los 4 números en cada fila y en cada columna sean iguales. ¿Qué número tiene la
celda sombreada?

A) 5 B) 6 C) 7 D) 8 E) 9

CANGURO 2020 N5 #27

2.28 F
1 1 1 3
Determina las soluciones enteras de la ecuación   
a b c 4
con 1  a  b  c

Olimpiada Matemática de Chile 2011


3 Divisibilidad.
Concepto de divisibilidad.
3.1 Definición. Divisor de un número.
Dados dos números enteros a, b , diremos que a divide a b, o que b es divisible entre a,
y escribiremos a | b , cuando exista un tercer número entero c tal que a c  b .
Puesto que a  0  0 , todo número entero es divisor de cero. Incluso 0 | 0 .

3.2 Proposición. Propiedades básicas de la divisibilidad.


a) a | a para todo entero a (propiedad reflexiva)
b) a | b y b | c  a | c (propiedad transitiva)
c) a | b y a | c  a | x b  y c para cualquier par de enteros x, y
d) a | b y c | d  ac | bd . En particular, a | b  a | b c .
e) a | b y a | b  c  a | c

3.3 Observación.
a | a n , y por tanto a n | c  a | c , por la propiedad transitiva: a | a n | c  a | c

3.4 Criterios básicos de divisibilidad.


Entre 2: Cuando acaba en cero o cifra par.
Entre 3: Cuando la suma de sus cifras es múltiplo de 3.
Entre 4: Cuando el número formado con sus dos últimas cifras es un múltiplo de 4.
Entre 5: Cuando acaba en 0 o en 5.
Entre 9: Cuando la suma de sus cifras es múltiplo de 9.
Entre 11: Cuando la diferencia entre la suma de sus cifras pares y la suma de sus cifras
impares sea 0 o múltiplo de 11.

3.5 Teorema. Algoritmo de la división.


Para todo a  Z y b  N , existen q  Z y 0  r  b únicos, llamados respectivamente
cociente y residuo de la división, tales que
a  q b  r

3.6 Problema resuelto.


Aplicando el Algoritmo de la división, demuestra que todo cuadrado perfecto es
siempre de la forma 4k o 4k  1.

Solución. Aplicando el Algoritmo de la división, todo número n será de la forma


n  4b , n  4b  1, n  4b  2 , n  4b  3  4b  4  1  4(b  1)  1.
Veamos su cuadrado, caso por caso:
 
n  4b  n 2  4b   4 4b 2
2

n  4b  1  n 2  4b  1  16b 2  8b  1  4(4b 2  2b)  1


2

n  4b  2  n 2  4b  2  16b 2  16b  4  4(4b 2  4b  1)


2

Sea cual sea el caso, siempre es de la forma 4k o 4k  1.


Nota: Este resultado es muy útil para resolver muchos problemas de ecuaciones
diofánticas.
3.7 MF
El número de dígitos de 416525 (cuando está escrito en la base 10 usual) es

(A) 31 (B) 30 (C) 29 (D) 28 (E) 27

AHSME 1984 #9

3.8 F
Demuestra que el cuadrado de un número impar es siempre de la forma 8k  1

OPOS BALEARES 2018

3.9 F
El perímetro de un triángulo equilátero excede el perímetro de un cuadrado en 1989 cm.
La longitud de cada lado del triángulo excede la longitud de cada lado del cuadrado en d
cm. El cuadrado tiene perímetro mayor que 0. ¿Cuántos posibles enteros positivos no
son válidos para d?

(A) 0 (B) 9 (C) 221 (D) 663 (E) infinitos

ASHME 40 #17

3.10 F
¿Cuántos números en base 10, N  a b c d satisfacen todas las tres condiciones
siguientes?
(i) 4000  N  6000
(ii) N es múltiplo de 5
(iii) 3  b  c  6

(A) 10 (B) 18 (C) 24 (D) 36 (E) 48

AHSME 1995 #12

3.11 F
La profesora Walter corrige un examen de matemáticas de sus cinco alumnos. Entra en
orden aleatorio las puntuaciones en una hoja de cálculo, que va recalculando la media
de la clase después de cada puntuación (sobre el número de alumnos ya introducidos, no
sobre el total de 5). La profesora se da cuenta de que, después de cada puntuación, la
media es siempre un entero. Las puntuaciones (presentadas en orden ascendente) son
71, 76, 80, 82 y 91. ¿Cuál fue la última puntuación que introdujo la profesora Walter?

(A) 71 (B) 76 (C) 80 (D) 82 (E) 91

AMC12 2000 #9
3.12 F
Determina los valores enteros de x para los cuales la expresión x 2  6 x es un cuadrado
perfecto, es decir, el cuadrado de un entero.
3.13 M
Determina la suma de todos los números enteros positivos b  1000 tal que el número
36b (escrito en base b) es un cuadrado perfecto y el número 27b (también escrito en
base b) es un cubo perfecto.

AIME II 2018 #3

3.14 F
Determina la suma de todos los enteros positivos n tales que n2  85n  2017 es un
entero.

AIME II 2017 #6

3.15 F
Demuestra que ningún número de la forma 11,111,1111,11111,... es un cuadrado
perfecto.

Indicación: Todo número de la forma 111...111 se puede escribir como


111...111  111...108  3  4k  3 .

3.16 MF
¿Para cuantos enteros n entre 1 y 100 el polinomio x 2  x  n factoriza en el producto
de dos factores lineales con coeficientes enteros?

(A) 0 (B) 1 (C) 2 (D) 9 (E) 10

ASHME 1989 #8

3.17 F
Sea S el número de pares ordenados de enteros (a, b) con 1  a  100 y b  0 tales que
el polinomio x2  ax  b pueda ser factorizado como producto de dos (no
necesariamente distintos) factores lineales con coeficientes enteros. Determina el
residuo cuando S se divide entre 1000.

AIME I 2018 #1

3.18 F
Una sucesión pucelana es una sucesión crecientes de dieciséis números impares
positivos consecutivos, cuya suma es un cubo perfecto. ¿Cuántas sucesiones pucelanas
tienen solamente números de tres cifras?

OME 2010 #1

3.19 M
a) Demuestra que el producto de dos números consecutivos es par.
b) Demuestra que el producto de tres números consecutivos es divisible entre 6.
c) Demuestra que n5  n es divisible entre 30.
3.20 F
Determina la suma de todos los números primos entre 1 y 100 tal que sean
simultáneamente 1 mayor que un múltiplo de 4 y 1 menor que un múltiplo de 5

ASHME 1999 #4

3.21 MF
¿Cuántos enteros positivos b existen con la propiedad de que log b 729 sea un entero
positivo?

(A) 0 (B) 1 (C) 2 (D) 3 (E) 4

AMC12 2000 #7

3.22 F
Aplicando el Algoritmo de la división, demuestra que:
a) Un cuadrado perfecto es siempre de la forma 3k o 3k  1 .
b) Un número de la forma 3a 2  1 nunca puede ser un cuadrado perfecto.

3.23 F
N2  7
¿Para cuántos enteros N entre 1 y 1990 la fracción impropia no es irreducible?
N 4
ASHME 1990 #19

3.24 MF
Sea r el residuo cuando 1059, 1417 y 2312 se dividen entre d  1 . Determina el valor de
d r .

AHSME 1976

3.25 F
Demuestra que existen infinitos números enteros n tales que n 2  23 es divisible por 24.

3.26 M
Determina todos los enteros positivos d tales que d divide n 2  1 y (n  1) 2  1 para
algún entero n.

3.27 F
Determina todos los enteros positivos n tales que el número que se obtiene eliminando
el último dígito es un divisor de n .
Divisibilidad y orden.

3.28 Proposición. Divisibilidad implica desigualdad.


a) Si a | b y b  0  a  b .

Y por tanto, si a y b son positivos:

b) a | 1  a  1
c) a | b  a  b
d) a | b y b | a  a  b

3.29 F
Sean 1  d1  d2  d3  ...  dk  n los divisores del entero positivo n.
Encuentra todos los números n tales que n  d 22  d33 .
México 2008

3.30 M
Sea n  231 319 . Determina el número de enteros positivos de n 2 que sean menores que
n y que no sean divisores de n .
AIME 1995 #6

Indicación: Distribución de los divisores de n 2 .


Un dato que puede ser útil es que n es el divisor central de n 2 , es decir, el número de
divisores de n 2 menores que n es igual al número de divisores de n 2 mayores que n.

3.31 D
Encontrar las ternas a, b, n  de números enteros positivos tales que
1 1 n
  .
a b ab

3.32 M
Dado n  IN , calcula la longitud del lado BC en el triángulo que se muestra en la
siguiente figura. ¿Existe algún valor de n para el que la longitud BC sea entero?
3.33 F
Sea n un número de cinco dígitos, y sean q y r el cociente y el residuo, respectivamente,
cuando n se divide entre 100. ¿Para cuántos valores de n se cumple que q  r es
divisible entre 11?

(A) 8180 (B) 8181 (C) 8182 (D) 9000 (E) 9090

AMC12A 2003 #18

3.34 D
Determina todos los enteros n  0 tales que existen enteros a, b con la propiedad
n 2  a  b y n3  a 2  b 2

Romanian Mathematical Olympiad 2004

3.35 MD
Dado un entero n  3 , demuestra que siempre es posible, eliminando a lo sumo dos
elementos del conjunto 1, 2 , ..., n  de forma que la suma de los restantes números sea
un cuadrado perfecto.

Romanian Mathematical Olympiad 2003

3.36 D
Determina todos los enteros positivos a, b, c tales que
ab  bc  ac  abc
4 Máximo común divisor y mínimo común múltiplo.
Concepto de mcd y mcm.
4.1 Definiciones. Máximo común divisor. Números coprimos. Mínimo común múltiplo.
Dados dos números positivos a, b , el conjunto de divisores comunes de ambos no está
vacío, pues 1 es divisor de los dos, y está acotado superiormente, pues todo divisor
común es menor o igual que min( a, b) . Luego tiene sentido definir el máximo común
divisor de a y b como el mayor divisor positivo común a ambos, que denotaremos por
a, b .
Diremos que dos números son coprimos cuando su único divisor común sea 1, es decir,
cuando a, b   1 .

Dados dos números positivos a, b , el conjunto de múltiplos comunes a ambos no está


vacío, pues a b es un múltiplo común, y está acotado inferiormente por max( a, b) .
Luego tiene sentido definir el mínimo común múltiplo de a y b como el menor de los
múltiplos comunes a ambos, y se denotará por a, b .

4.2 Proposición. Algunas propiedades básicas del mcd y del mcm.


a) a, b | a, b | a, b .
b) a, b  min( a, b)  a, b  max( a, b)  a, b.

4.3 Interpretación geométrica del máximo común divisor.


El máximo común divisor de a y b indica el número de puntos ( x, y) con coordenadas
enteras en el segmento que une los puntos ( 0 , 0 ) y ( a , b ) , sin contar el inicial ( 0 , 0 ) .
Por ejemplo, (15 , 6 )  3 , y por tanto el segmento que une los puntos ( 0 , 0 ) y (15 , 6 )
pasa por tres puntos con coordenadas enteras, aparte del propio ( 0 , 0 ) :

15 
 5 ( 5 , 2 )
3  
  ( 5  5 , 2  2 )  (10 ,4 )
6
 2  (10  5 , 4  2 )  (15 ,6 )
3 

4.4 F
Diremos que un punto ( x, y) del plano es “punto entero” cuando sus coordenadas sean
enteras. ¿Cuántos de estos puntos de este tipo hay (incluyendo ambos extremos) en el
segmento que cuyos extremos son ( 3,17 ) y ( 48 , 281) ?

ASHME 1989 #16

4.5 MF
Demostrar que (n, n  k ) | k , independientemente del valor de n . En particular,
(n, n  1)  1
El Teorema de Bezout y sus consecuencias.

4.6 Lema.
(a, b) divide a cualquier combinación lineal de a y b .

Demostración. Sea ax  by una combinación lineal de a y b .


(a, b) | a  (a, b) | ax
  (a, b) | ax  by
(a, b) | b  (a, b) | by 

4.7 Teorema. Teorema de Bezout (TDB).


Dados dos números enteros a, b no ambos cero, el máximo común divisor de a, b se
caracteriza por ser el elemento mínimo del conjunto no vacío

A   ax  by , x, y  Z , ax  by  0 

Luego existe, es único y siempre se puede escribir como combinación lineal de a y b :


Existen enteros x, y tales que (a, b)  ax  by

Demostración. Consideremos el conjunto anterior A   ax  by , x, y  Z , ax  by  0 .


Es un conjunto no vacío pues al menos aa  bb  a 2  b2  0 pertenece a A (estamos en
todo momento suponiendo que a, b no son ambos cero).

Por el Principio de buena ordenación, A tendrá un mínimo, al que llamaremos d .


Vamos a demostrar que d  (a, b) .

Supongamos que d  a x1  b y1  0 para ciertos enteros x1 , y1 .


Por el Algoritmo de la división, existirán enteros q, r tales que a  d q  r con
0r d.

Si r  0 , entonces
0  r  a  dq  a  (a x1  b y1 )q  a  a x1q  b y1q  a(1  x1q)  b y1q , y por tanto, r
pertenece al conjunto A, tomando x2  1  x1q , y2   y1q . Pero r  d , lo cual
contradice la hipótesis de d como elemento mínimo. Luego r  0 y por tanto a  d q , es
decir, d divide al número a.

Con el mismo razonamiento se demuestra que d divide al número b, y por tanto d es


común divisor de a y b. Veamos que es el máximo común divisor.
Sea m otro común divisor de a y b, entonces, aplicando el lema anterior,
m | a x1  b y1  d  m | d  m  d .
4.8 Corolario.
Si x1 ,..., xn son números enteros y a es cualquier número entero positivo,
a x1,..., a xn   a x1,..., xn 
Demostración.
Sean d  a x1,..., a xn  y e  x1 ,..., xn  . Luego e | xi  a e | a xi  a e | d  a e  d .
Por el TDB, e  k1x1  ...  kn xn  a e  k1 ax1   ...  kn axn 
Es decir, ae es combinación lineal de a x1 ,..., a xn , y por tanto es un múltiplo de d ,
luego ae  d . Y, puesto que anteriormente hemos demostrado que a e  d , llegamos a
ae  d .

4.9 Corolario.
(a, b)  1  Existe una combinación lineal ax  by  1

Demostración.  Es el TDB.  (a, b) | ax  by  1  (a, b) | 1  (a, b)  1

4.10 M
21n  4
Demuestra que la fracción es irreducible para todo número natural n.
14 n  3
IMO 1959

4.11 M
Los números de la sucesión 101,104 ,109 ,116, ... son de la forma an  100  n2 ,
n  1, 2, ...
Para cada n , sea d n   an , an1  . Determina max n 1 d n .

AIME 1985

4.12 Teorema.
a a a 
Si d  a1 , a2 , ..., an  entonces  1 , 2 , ..., n   1
d d d 

Demostración.
a a a 
d  a1 , a2 , ..., an   d | ai  ai  ki d . Sea d '   1 , 2 , ..., n  y supongamos que
d d d 
d ' 1.
a a a  a a
d '   1 , 2 , ..., n   d '| i  i  k 'i d '  ai  k 'i d ' d  d ' d | ai  d ' d | d
d d d  d d
Lo cual es imposible suponiendo d ' 1. Luego d ' 1.
Divisibilidad con números coprimos.

4.13 Teorema.
Si (a, b)  1 , entonces:
a | c
a)   ab | c b) a | bc  a | c
b | c
d | ac  d | ac 
c) d |c d)   d | ce
d | bc  d | be 

Demostración.
a)
a | c  c  aa' 

b | c  c  bb' 
(a, b)  1  1  ax  by 
 c  c  1  c(ax  by )  cax  cby  bb' ax  aa' by  ab(b' x  a' y )  ab | c
b)
a | bc  bc  ak1
d | bc  bc  dk2
(a, b)  1  1  k3a  k4b  c  k3ac  k4bc  k3ac  k4 ak1  ak3c  k4 k1   a | c

Por el TDB, (a, b)  1  1  ax  by para ciertos enteros x, y . Luego c  acx  bcy , y


puesto que trivialmente a | acx y por hipótesis a | bcy , se deduce a | acx  bcy  c .
c)
d | ac  ac  dk1
d | bc  bc  dk2
(a, b)  1  1  k3a  k4b  c  k3ac  k4bc  k3dk1  k4 dk2  d (k3k1  k4 k2 )  d | c

d)
d | ac  d | a(ce)
  d | ce aplicando el apartado anterior.
d | be  d | b(ec ) 

4.14 Observación. El apartado a) del teorema anterior se puede generalizar:


Si a1 , a2 ,..., an son enteros tales que (ai , a j )  1 si i  j , entonces
a1 | b , a2 | b , ..., an | b  a1a2 ...an | b

4.15 D
Sean m, n dos enteros positivos diferentes. Demostrar que

(m, n)  (m  1, n  1)  (m  2, n  2)  m  n  1

Indian National Mathematical Olympiad 2019 #3 (parcial)


El algoritmo de Euclides.

4.16 Lema.
(a, ab  c)  (a, c)

Demostración.
Sean n  (a, ab  c) y m  (a, c) .
n  (a, ab  c)  n  xa  y(ab  c)  xa  aby  yc  a( x  by)  yc , luego (a, c) | n .
Y por otro lado,
m | a  m | ab
m  ( a, c )     m | ab  c  m | (a, ab  c)  n .
m | c 
Así pues, n | m y m | n , y por tanto n  m .

4.17 Teorema. Algoritmo de Euclides.


El algoritmo de Euclides es un método efectivo para calcular el máximo común divisor
de dos números a y b que se basa en el algoritmo de la división y en la proposición
anterior:
Si b  a k  c , entonces (a, b)  (a, a k  c)  (a, c)

Suponiendo a  b , podemos dividir a entre b para expresar a  kb  r , con r  b y


así
(a, b)  (bk  r, b)  (r, b)

Este mismo proceso repetiremos una y otra vez, con números más y más pequeños,
hasta que el máximo común divisor se haga evidente.

4.18 Ejemplo.
Calcula  29 , 23  mediante el Algoritmo de Euclides.

Solución.
29  1  23  6  (29,23)  (1  23  6 , 23)  ( 6 , 23)
23  3  6  5  (23,6)  ( 3  6  5, 6 )  ( 5, 6 )
6  1  5  1  (6,5)  (1  5  1, 5 )  (1, 5 )  1
Así pues,  29 , 23   1

4.19 Ejemplo.
Calcula  3456 , 246  mediante el Algoritmo de Euclides.

Solución.
( 3456 , 246 )  (13  246  158, 246 )  (158, 246 )
(158 , 246 )  (158 ,1158  88 )  (158 , 88 )
Y de la misma manera: (158, 88 )  ( 70 , 88 )  (18, 70 )  (16 ,18 )  ( 2 ,16 )  2
Luego  3456 , 246   2
4.20 Proposición.
c | a a | c
a)   c | a, b  b)   a, b | c
c | b b | c

Demostración.
a)  Por el TDB, a, b  ax  by  ca' x  cb' y  c(a' x  b' y)  c | a, b
 Por transitividad: c | (a, b), (a, b) | a  c | a
b)  Sea k  a, b. Supongamos que k  0 , (es decir, que a, b  0 )
Por reducción al absurdo, supongamos que k | c .
Entonces existirán q y 0  r  k tales que c  kq  r .
Luego r  c  kq múltiple común de a y b , pues lo son c y k , pero r  k ,
contradiciendo la hipótesis de k como menor múltiple común de a y b .
 Por transitividad: a, b | c, a | a, b  a | c .

4.21 Corolario.
a |c
  (a, b) | c, d 
b | d

Demostración.
( a, b) | a | c 
  ( a , b ) | ( c, d )
( a, b) | b | d 
Máximo común divisor con números coprimos.

4.22 Proposición.
(a, c)  1  (ab, c)  (b, c)

Demostración.
Por el TDB, (a, c)  1 implica que existen dos enteros x, y tales que 1  ax  cy .
Sea g  (b, c) y h  (ab, c) . Los valores g, h no son negativos.
g  (b, c) | b | ab
  g | (ab, c)  h
g  (b, c) | c 
h  (ab, c) | ab | abx 
  h | abx  cby  b(ax  cy )  b  1  b
h  (ab, c) | c | cby 
h  (ab, c) | c 
  h | (b, c)  g
h|b 
Llegamos a h | g y g | h , y puesto que no son negativos, se deduce que g  h .

4.23 Corolario.
(a, c)  1
  (ab, c)  1
(b, c)  1

Demostración.
 Por la proposición anterior, (a, c)  1  (ab, c)  (b, c)  1.

d | a  d | ab
  d | (ab, c)  1  d  1 , luego (a, c)  1 , y con un razonamiento similar
d |c 
se demuestra que (b, c)  1

Observación. Esta propiedad se puede generalizar fácilmente por inducción:

(a1, c)  (a2 , c)  ...  (an , c)  1  (a1a2 ...an , c)  1

4.24 Teorema. El Lema de Euclides.


a | bc 
a|c
(a, b)  1

Demostración.
a | bc 
  a | (a, bc)
a|a 
Por otro lado, por 4.8, (a, b)  1  (a, bc)  (a, c) .
Luego a | (a, bc)  (a, c) | c , es decir, a | c .
4.25 Corolario.
Si c  0 , (ca, cb)  c(a, b)

Demostración.
c y (a, b) son no negativos, luego c(a, b) es no negativo. Luego, aplicando 4.6a
(a, b) | a  c(a, b) | ca 
  c(a, b) | (ca, cb)
(a, b) | b  c(a, b) | cb 
Por otro lado, aplicando el TDB, existirán enteros x, y tales que (a, b)  ax  by .
(ca, cb) | ca | cax 
  (ca, cb) | cax  cby  c(ax  by )  c(a, b)  (ca, cb) | c(a, b)
(ca, cb) | cb | cby 

Así pues, c(a, b) | (ca, cb) y (ca, cb) | c(a, b) , y puesto que ambos son no negativos,
llegamos a (ca, cb)  c(a, b) .

4.26 Corolario.
Si a y b son enteros positivos, (a, b)  1  (c, a)(c, b)  (c, ab)

Demostración.
Floor and arithmetic functions (Darij Grinberg, 2016) pág. 12
5 Números primos.
5.1 Concepto de número primo.
5.1.1 Definición. Número primo.
Todo número es divisible por sí mismo y por la unidad. Diremos que un número natural
p  1 es primo cuando solo sea divisible por sí mismo y por la unidad. Por ejemplo,
son primos los números 5, 13, 59 o 397. Llamamos compuestos a los números que no
son primos. El número 1 no se considera ni primo ni compuesto.

5.1.2 Proposición. Algunas propiedades de los números primos.


a) Si p es primo y a | p , entonces a  1 ,  p .
b) Si p y q son primos, entonces p | q  p  q .
c) Todos los primos son impares excepto el 2.
d) El 2 y el 3 son los únicos primos cuya diferencia es 1.
e) Si p es primo y a, b  N , entonces p | a b  p | a .

5.1.3 Problema resuelto.


Demuestra que si p  3 es primo, entonces 24 | p 2  1.

Solución.
Por el algoritmo de la división, todo número se puede representar como 6n , 6n  1 o
6n  2 . Si es primo, la única opción aceptable es 6n  1 , pues las otras son divisibles
entre 2 o 3. Luego:
p  6n  1  p 2  6n  1  36n2  12n  1  p 2  1  36n2  12n  12n(3n  1)
2

Está claro que o bien n es par o bien 3n  1 es par, luego 24 | p 2  1.

5.1.4 Problema resuelto.


Determina la suma de todos los números primos entre 1 y 100 que son simultáneamente
1 más que un múltiplo de 4 y 1 menos que un múltiplo de 5.

AHSME 1999 #3
Solución.
p  4a  1
  4a  1  5b  1  5b  4a  2  2a  1
p  5b  1 
 2 | 5b  2 | b  b  2c  p  5(2c)  1  10c  1
Con p  10c  1 ya tenemos un conjunto de candidatos suficientemente pequeño como
para proceder a testearlos, uno por uno:
c  1  p  10  1  1  9 (no es primo) c  2  p  10  2  1  19  4  4  3
c  3  p  10  3  1  29  4  7  1 c  4  p  10  4  1  39  4  9  3
c  5  p  10  5  1  49 c  6  p  10  6  1  59  4  14  3
c  7  p  10  7  1  69 c  8  p  10  8  1  79  4  19  3
c  9  p  10  9  1  89  4  22  1 c  10  p  10  10  1  99
Luego la suma es 29+89=118.
5.1.5 Corolario. Corolario al Lema de Euclides.
Si p es primo y p | ab entonces p | a o p | b .

Demostración.
Basta aplicar el Lema de Euclides teniendo en cuenta que mcd ( p, a)  mcd ( p, b)  1 .

5.1.6 M
Sean x, y enteros. Demostrar que 2 x  3 y es divisible entre 17 si y solo si 9 x  5 y es
divisible entre 17.

5.1.7 Corolario.
a) a es par si y solo si a n es par
b) a es impar si y solo si a n es impar.

Demostración.
 
a)  a  2k  a n  (2k )n  2 2n 1 k n
 Basta aplicar el Lema de Euclides con p  2 .
n
 n
b)  a  2k  1  a n  (2k  1) n  1    2k  impar.
k

k 1  
k
 Por el apartado a, si a es par entonces a n es par, luego si a n es impar,
necesariamente a debe ser par.

5.1.8 Corolario.
Hay infinitos números primos.

Demostración.
Entre otras muchas demostraciones de este resultado, la de Euclides es un ejemplo de
elegancia:
Supongamos, por el contrario, que existe una cantidad finita de números primos. Sean
estos
1  p1  p2  ...  pn
Consideremos el número n  p1  p2  pn  1 .

No puede ser primo, pues n  pn , luego existirá al menos un k tal que pk | n , pero
también se cumple trivialmente pk | n  1  p1  p2  pn , y por tanto
pk | n  (n  1)  1  pk | 1  pk  1

lo cual es imposible. Así pues, no es posible que exista un número finito de primos.
5.2 El Teorema fundamental de la aritmética (TFA).

5.2.1 Teorema. Teorema fundamental de la aritmética (TFA).


a) Todo entero n  1 se puede representar como producto de números primos.
b) Ordenando los pi , obtenemos lo que denomina descomposición canónica del
número n :
n  p1a1 p2a2 ... pkak con p1  p2  ...  pk y 0  ai

Por ejemplo: 18  2  32 , 84  22  3  7 , 4576  2  11  13 , 32716  2  8179

La descomposición canónica de un número es única.

Demostración. a) Sea n  1 . Si n es primo, ya hemos acabado. Supongamos que n es


compuesto y sea p1 su menor divisor. Está claro que p1 tiene que ser primo.
Luego n  p1n1 , para cierto 1  n1  n . De nuevo, si n1 es primo ya hemos acabado.
Supongamos que es compuesto y sea p2 su menor divisor. Está claro que p2 tiene que
ser primo, y n  p1 p2n2 para cierto 1  n2  n1  n .
Este proceso no puede continuar indefinidamente, pues antes de n pasos debemos
encontrar un p x primo, y n  p1 p2 ... ps .

b) Supongamos que n  p1a1 p2a2 ... psas  q1b1 q2b2 ...qtat .


Por el Corolario al Lema de Euclides, todo pi será un q j y todo q j será un pi .
Esto implica que s  t . Puesto que, además, p1  p2  ...  ps y q1  q2  ...  qs ,
tenemos que pi  qi para todo 1  i  s :
n  p1a1 p2a2 ... psas  p1b1 p2b2 ... psas
Veamos que también los exponentes coinciden.
Si ai  bi para cierto 1  i  s , entonces, dividiendo entre pibi ambos lados de la
igualdad, tenemos
p1a1 p2a2 ... piai bi psas  p1b1 p2b2 ... pibi11 pibi11 ... psas

lo cual es absurdo, pues pi divide la parte izquierda pero no la parte derecha de la


igualdad.

Con razonamiento similar demostramos que tampoco se puede dar ai  bi , y por lo tanto
llegamos a la conclusión de que ai  bi para todo 1  i  s , es decir, las dos
descomposiciones canónicas coinciden.
5.2.2 Proposición.
Dado un número en descomposición canónica n  p1a1 p2a2 ... pkak ,
a) Todo divisor de n es de la forma d  p1e1 p2e2 ... pkek , con 0  ei  ai .
b) El número de divisores de n es a1  1a2  1...ak  1 .

Demostración.
a) Todo número de la forma d  p1e1 p2e2 ... pkek , con 0  ei  ai es un divisor de n, y son
todos diferentes por el TFA.
b) Basta aplicar el principio fundamental del recuento.

5.2.3 F
Determina todas las parejas de enteros positivos (m, n) tales que m2n  2020 .

AIME II 2020 #1

5.2.4 F
Existen enteros positivos A, B y C, sin factores comunes mayores que 1, tales que
A log 200 5  B log 200 2  C
Determina A  B  C .
ASHME 1995 #24

5.2.5 M
Demostrar que existe un único número natural n tal que 28  211  2n es un cuadrado
perfecto.

5.2.6 F
Si 1998 se escribe como producto de dos enteros positivos cuya diferencia es lo más
pequeña posible, entonces la diferencia es:

(A) 8 (B) 15 (C) 17 (D) 47 (E) 93

AHSME 1998 #6

5.2.7 M
Determina los tres números naturales consecutivos más pequeños cuya suma es un
cuadrado perfecto y un cubo perfecto de números naturales.

5.2.8 M
Halla todas las sucesiones finitas de n números naturales consecutivos a1 , a2 ,..., an , con
n  3 , tales que a1  a2 ,...  an  2009
OME 2009 #1
5.2.9 MF
En el año 2001, los Estados Unidos acogieron las Olimpiadas Matemáticas. Sean
I , M , O enteros positivos tales que I  M  O  2001. ¿Cuál será el valor más grande
posible de la suma I  M  O ?

(A) 23 (B) 55 (C) 99 (D) 111 (E) 671

AMC12 2000 #1

5.2.10 F
Existen enteros positivos A, B, C , sin factores comunes mayores que 1, tales que
A log 200 5  B log 200 2  C .

¿Cuál es el valor de A  B  C ?

(A) 6 (B) 7 (C) 8 (D) 9 (E) 10

AHSME 1995 #24

5.2.11 M
¿Cuántos conjuntos de tres elementos  a, b, c  de enteros positivos verifican
a  b  c  2310 ?

(A) 32 (B) 36 (C) 40 (D) 43 (E) 45

AHSME 1995 #29

5.2.12 F
Existe un número primo p tal que 16 p  1 es el cubo de un entero positivo. Determina p.

AIME I 2015 #3

5.2.13 F
Cuando los números 702, 787 y 855 son divididos entre el entero positivo m, el resto es
siempre el mismo entero positivo r. Cuando los números 412, 722 y 815 son divididos
entre el entero positivo n, el resto es siempre el mismo entero positivo s  r . Determina
m n r  s.
AIME I 2017 #2

5.2.14 M
Determina el número de polinomios de segundo grado f (x) con coeficientes enteros y
ceros enteros tales que f (0)  2010 .

AIME II 2010 #10


5.2.15 M
Determina todos los primos p para los cuales la ecuación x 2  px  444 p  0 tiene
soluciones enteras.

ASHME 1987 #23

5.2.16 M
Demostrar que en cualquier conjunto de n  1 números entre 1 y 2n siempre podemos
encontrar dos elementos tales que el menor divide al mayor.

5.2.17 F
Determina el número de valores de k sabiendo que 1212 es el mínimo común múltiplo de
66, 88 y k.

AIME 1998

5.2.18 F
Denotamos por [r,s] el mínimo común múltiplo de los enteros positivos r y s. Determina
el número de triplas ordenadas a, b, c tales que [a,b]=1000, [b,c]=2000 y [c,a]=2000.

5.2.19 F
Determina 3x 2 y 2 si x, y son enteros tales que y 2  3x 2 y 2  30 x 2  517

AIME 1987 #5

5.2.20 M
Determina p sabiendo que es un número primo tal que 16 p  1 es el cubo de un
número entero.

AIME I 2015 #3

5.2.21 M
Determina todos los enteros a, b  0 tales que
(a, b)  a, b  a  b  6
5.2.22 Teorema. Descomposición canónica del mcd y del mcm.
Se puede demostrar fácilmente que si a  p1a1 p2a2 ... pnan y b  p1b1 p2b2 ... pnbn
(en donde algunos de los ai y bi pueden ser 0)

Entonces:
a) mcd (a, b)  p1min(a1 ,b1 ) p2min(a2 ,b2 ) ... pnmin(an ,bn )
b) mcm(a, b)  p1max( a1 ,b1 ) p2max( a2 ,b2 ) ... pnmax( an ,bn )
c) ab  mcd (a, b)  mcm(a, b)

Demostración.
c) Basta aplicar el anterior teorema teniendo en cuenta que
x  y  min( x, y)  max( x, y) .

5.2.23 Proposición.
a) Si p | mcm(a, b) y p | a  b entonces p | mcd (a, b) .
b) Si p | mcd a, b entonces p | mcm(a, b) y p | a  b .
c) mcd a, b  mcd a  b, mcm(a, b)

Demostración.
a) Puesto que mcm(a, b)  p1max( a1 ,b1 ) p2max( a2 ,b2 ) ... pnmax( an ,bn ) , si p | mcm(a, b) entonces p | a
o p |b.
Si p | a , puesto que p | a  b , se deduce que p | b , y por tanto p | mcd (a, b) .
Si p | b , con un razonamiento similar llegamos igualmente a p | mcd (a, b) .
b) Ya fue demostrado anteriormente que p | mcd (a, b)  p | a  b .
Puesto que max( x, y)  min( x, y) , está claro que p | mcd (a, b)  p | mcm(a, b)

5.2.24 M
Halla dos enteros positivos a y b conociendo su suma y su mínimo común múltiplo.
Aplícalo en el caso de que la suma sea 3972 y el mínimo común múltiplo 985928.

OME 2008 #1

5.2.25 F
Determina todos los enteros n tales que n  50 y n  50 son ambos cuadrados
perfectos.
5.2.26 Proposición.
Sean a, b números positivos coprimos. Entonces:
a) Los divisores positivos de ab son de la forma dd ' , con d | a , d ' | b , d , d '  0 .
b) Si ab  c n , con c  0 , entonces a  u n y b  v n para ciertos enteros u, v .

Demostración.
a) Puesto que a y b son coprimos, sus respectivas descomposiciones factoriales
canónicas serán de la forma a  p1a1 p2a2 ... prar , b  q1b1 q2b2 ...qsbs , con pi  q j .
Luego la descomposición factorial del producto será
ab  p1a1 p2a2 ... prar q1b1 q2b2 ...qsbs
Por lo tanto, los divisores de ab serán de la forma
  
p1a '1 p2a '2 ... pra 'r q1b'1 q2b'2 ...qsb's  p1a '1 p2a '2 ... pra 'r q1b'1 q2b'2 ...qsb' s con a'i  ai y b' j  b j .
es decir, son el producto de un divisor positivo de a y otro de b.

b) De nuevo, a  p1a1 p2a2 ... prar , b  q1b1 q2b2 ...qsbs , con pi  q j . Por lo tanto
cn  ab  p1a1 p2a2 ... prar q1b1 q2b2 ...qsbs
Supongamos que un primo p tal que p | c . Entonces
p | c  p | c n  ab  p1a1 p2a2 ... prar q1b1 q2b2 ...qsbs , luego p es un pi o un q j , y por tanto c se
escribe como c  p1a '1 p2a '2 ... pra 'r q1b '1 q2b'2 ...qsb ' s .
Luego cn  p1na'1 p2na'2 ... prna'r q1nb'1 q2nb'2 ...qsnb' s , y por la unicidad de la descomposición
canónica, se sigue que na'i  ai y nb' j  b j , y por tanto

a  p1na'1 p2na' 2 ... prna'r  p1a '1 p2a ' 2 ... pra ' r 
n
 n
, y b  q1nb'1 q2nb' 2 ...qsnb' s  q1b '1 q2b ' 2 ...qsb ' s .

5.2.27 Proposición.
Sean a, b, c enteros positivos tales que a n  bnc para algún entero positivo n. Entonces
c  d n para algún entero d . En particular, si c no es la potencia n-ésima de un número
entero, entonces n c no es un número racional.

Demostración.
Si a  p1a1 p2a2 ... prar es la descomposición de a en factores primos, entonces la
descomposición de a n en factores primos será a n  p1na1 p2na2 ... prnar
Puesto que b | a n , la descomposición de b en factores primos será de la forma
b  p1b1 p2b2 ... prbr , con 0  bi  n ai
Y por el mismo motivo tendremos c  p1c1 p2c2 ... prcr , con 0  ci  n ai
Luego an  bnc  p1nb1 p2nb2 ... prnbr p1c1 p2c2 ... prcr  p1nb1 c1 p2nb2 c2 ... prnbr cr
Y por unicidad de la descomposición en factores primos tendremos
nbi  ci  nai  ci  nai  nbi  nai  bi   0  ai  bi
Luego d  p1a1 b1 p2a2 b2 ... prar br es un número entero que verifica d n  c .
Para la última parte, si n
c fuera racional, se podría escribir comoc  a / b , con a, b
n

enteros positivos. Por lo tanto, a  b c , y por el resultado anterior se deduce que c debe
n n

ser una potencia n-ésima de un entero, llegando a contradicción.

Fuente de estos últimos resultados: Apuntes de Teoría Elemental de Números, por Enrique Arrondo.
5.3 Resolución de problemas mediante identidades algebraicas.

Encuentra una tabla de factorizaciones y identidades notables al final del Dossier


de Problemas de Álgebra

5.3.1 Proposición. La primera identidad algebraica fundamental.


La identidad algebraica


x n  y n  x  y  x n 1  x n  2 y  x n 3 y 2  ...  x y n  2  y n 1 
es fundamental en la resolución de todo tipo de problemas de Aritmética.
Su demostración es fácil: Basta con desarrollar el producto de la derecha:

x  y  x n 1  x n  2 y  x n 3 y 2  ...  x y n  2  y n 1  
 x n  x n 1 y  x n  2 y 2  ...  x 2 y n  2  xy n 1  x n 1 y  x n  2 y 2  x n  3 y 3  ...  x y n 1  y n
 xn  y n
En particular, tomando y  1 obtenemos una identidad muy útil:
n1 n2 x n1  1
x x
n
x  ...  x  1 
x 1

De la primera identidad se desprende directamente el siguiente resultado:

a) a  b | a n  b n para cualquier n  1.
Por ejemplo, sin necesidad de ningún cálculo, 87672345  81012345 es divisible entre 666.

Que se puede generalizar para obtener el siguiente resultado:

b) Si d | n con d y n positivos, entonces a d  b d | a n  b n para todo entero a, b .


En efecto, supongamos que n  k d para cierto entero k . Entonces
a n  bn  a k d  bk d  a d    b   a
k d k d
 
 bd a d 1  a d  2b  a d 3b2  ...

5.3.2 La segunda identidad algebraica fundamental.


Otra identidad algebraica muy útil es


Si n es impar, x n  y n  x  y  x n 1  x n  2 y  xn 3 y 2  ...  x y n  2  y n 1 
En efecto, Si n es impar, x n  y n  x n   y  , y basta aplicar la identidad anterior.
n

De esta segunda identidad se desprende de forma directa el siguiente resultado:

c) Si n es impar, a  b | a n  b n
Que se puede generalizar para obtener el siguiente resultado:

e) Si d | n con d y n positivos, y n / d es impar, entonces a d  b d | a n  b n para todo


entero a, b .
5.3.3 Proposición.
Si 2n  1 es primo, entonces n es una potencia de 2.

Demostración. Supongamos, por el contrario, que n no es una potencia de 2, es decir,


que podemos escribir n  k  h , con k  1 impar.

Entonces
  k
 
2n  1  2k h  1  2h  1  2h  1k  2h  1 2h
k
   k 1
 2h k 2
 
 2h
k 3
 ...  2h  1 
Y por tanto nuestro número es divisible entre 2h  1 .

Observación 1: Este resultado será fundamental en el Tema 15, como base para definir
los “primos de Fermat”.

Observación 2: El recíproco no es cierto. Por ejemplo: 641 | 232  1 .

En efecto, utilizando que 641  24  54  5  27  1 ,


  
232  1  2 28 2 4  1  2 28 2 4  54  54  1  2 28 2 4  54  2 28 54  1    
   
 2 28 2 4  54  27  5  1  2 28  641  641  1  1  2 28  641  641  639 640 2  1 
4 4
 
 6412 28
 639640  1
2

En donde hemos utilizado que:



6404  1  6404  14  640  1 6403  6402  640  1  639 6403  6402  641    
  
 639 6402 640  1  641  639 6402  641  641  639  641 6402  1   
(En el Tema 6 volveremos a demostrar este resultado aplicando la aritmética modular)

5.3.4 F
Determina todos los números primos de la forma n3  1 , para enteros n  1 .

5.3.5 M
Demostrar que 2903n  803n  464n  261n es divisible entre 1897 para todo natural n .

EOTVOS 1899

5.3.6 M
Demuestra que n4  4 con n  IN es primo si y solo si n  1 .

5.3.7 M
Determina todos los enteros n  1 para los cuales n4  4n es un número primo.

5.3.8 F
Demuestra que, para todo n  IN , n 2 divide a n  1  1 .
n
5.3.9 F
Demostrar que 1001 divide 11993  21993  31993  ...  10001993

5.3.10 M
Demuestra que 7 divide al número 147  247  347  447  547  647

5.3.11 F
Si al cuadrado de un número de dos dígitos se le resta el cuadrado del número formado
invirtiendo el orden de sus dígitos, entonces el resultado no siempre será divisible por:

(A) 9 (B) El producto de los dígitos. (C) La suma de los dígitos.


(D) La diferencia de los dígitos. (E) 11

ASHME 1957 #24

5.3.12 MD
Demuestra que, para todo entero n  1 , el número n5  n4  1 no es primo.

5.3.13 M
Determina todos los enteros positivos a, b para los cuales a 4  4b4 es primo.

5.3.14 MF
Sean aº y bº los ángulos agudos de un triángulo rectángulo, con a>b y ambos números
primos. ¿Cuál es el menor valor posible de b?

(A) 2 (B) 3 (C) 5 (D) 7 (E) 11

AMC 12B 2020 #4


5.4 Resolución de problemas mediante factorización de polinomios.

Encontraréis una tabla de factorizaciones y identidades notables al final del


Dossier de Problemas de Álgebra

5.4.1 MF
Determina todos los pares de enteros positivos ( x, y) tales que x 2  y 2  23

5.4.2 MF
Tomamos dos números primos diferentes p, q entre 4 y 18. ¿Qué resultado podemos
obtener al restar su suma a su producto?

(A) 23 (B) 60 (C) 119 (D) 180 (E) 231


AMC 12 2000 #6

5.4.3 MF
Sean m, n enteros tales que m2  3m2n2  30n2  517 .
Determina 3m2 n2 .

AIME 1987 #5

5.4.4 M
Existen dos únicos enteros positivos x , y safisfaciendo la ecuación

x 2  84 x  2008  y 2
Determina x  y .

AIME I 2008 #4

5.4.5 F
Expresa el número 1812 como la diferencia de los cubos de dos enteros positivos
consecutivos.

5.4.6 F
Determina los pares ordenados ( x, y) de números enteros positivos que satisfacen la
ecuación
1 1 1 2
  
x y xy 5
6 Problemas de la segunda parte.
6.1 MF
Demuestra que, para todo entero n , 120 divide a n5  5n3  4n .

6.2 MD
Demuestra que n2  3n  5 no es divisible entre 121.

6.3 MD
Demuestra que 34  45 es el producto de dos enteros, cada uno de ellos mayor que
5 6

102002 .

6.4 F
Si a y b son enteros positivos, pruebe que 19 divide a 11a  2b si y solo si 19 divide a
18a  5b .

Olimpiada Matematica Mexicana 1988

6.5 MF
n2  n  1
Demuestra que si n es un entero positivo, entonces es una fracción
n 2  2n
irreducible.

6.6 MF
x3  3x  2
Determina los enteros x para los cuales sea entero.
2x  1

6.7 F
Existe un número primo p tal que 16 p  1 es el cubo de un entero positivo. Determina
p.

AIME I 2015 #3

6.8 MD
Dado un número primo p , determina todos los enteros k  0 para los cuales k 2  pk
es un entero positivo.

OME 1997 #4

6.9 MF
¿Cuántos cuadrados perfectos positivos menores de 106 son múltiplos de 24?

AIME I 2007 #1

6.10 MF
x2
Determina qué valores enteros de x hacen entera la función f ( x) 
x6
6.11 F
La suma de las siete cifras del número aaabbbb es igual al número de dos cifras ab.
¿Cuánto vale la suma a + b?

A) 8 B) 9 C) 10 D) 11 E) 12

CANGURO 2019 N5 #18

6.12 F
Se empaquetan 60 manzanas y 60 peras en cajas con igual número de manzanas y
distinto número de peras en cada caja. ¿Cuál es el mayor número posible de cajas que se
pueden empaquetar de esta manera?

A) 20 B) 15 C) 12 D) 10 E) 6

CANGURO 2019 N5 #19

6.13 M
¿Cuál es el menor número de elementos que tenemos que quitar del conjunto:

{10, 20, 30, 40, 50, 60, 70, 80, 90}

de modo que el producto de los elementos que quedan en el conjunto es un cuadrado


perfecto?

A) 1 B) 2 C) 3 D) 4 E) 5
CANGURO 2019 N5 #28

6.14 M
¿Cuántos de los números del 210 al 213 , ambos inclusive, son divisibles por 210 ?

A) 2 B) 4 C) 6 D) 8 E) 16

CANGURO 2019 N6 #10

6.15 M
¿Cuál es la mayor potencia de 3 que es divisor del número 7! + 8! + 9! ?

A) 32 B) 34 C) 35 D) 36 E) una potencia de 3 mayor que 36

CANGURO 2019 N6 #11

6.16 M
Un entero positivo n se llama “bueno” si su divisor más grande (excluyendo n) es igual
a n  6 . ¿Cuántos enteros positivos “buenos” hay?

A) 1 B) 2 C) 3 D) 6 E) infinitos

CANGURO 2019 N6 #16


6.17 M
Sea a la suma de todos los divisores positivos de 1024 y b el producto de todos los
divisores positivos de 1024. Entonces

A) (a  1)5  b B) (a  1) 5  b C) a 5  b D) a 5 1  b E) a 5 1  b

CANGURO 2019 N6 #21

6.18 M
Los vértices de la red mostrada se numeran del 1 al 10. La suma S de los números de los
cuatro vértices de cada cuadrado es la misma. ¿Cuál es el menor valor posible de S?

A) 18 B) 19 C) 20 D) 21 E) 22

CANGURO 2019 N6 #23

6.19 F
¿Para cuántos enteros n es n 2  2n  3 un número primo?
A) 1 B) 2 C) 3 D) 4 E) infinitos

CANGURO 2019 N6 #26

6.20 F
Este año, el número de chicos en mi clase ha aumentado respecto del curso pasado un
20% y el número de chicas ha disminuido un 20%. Ahora tenemos un estudiante más
que antes. ¿Cuál de los siguientes podría ser ahora el número de estudiantes en mi
clase?

A) 22 B) 26 C) 29 D) 31 E) 34
CANGURO 2019 N6 #12

6.21 D
ab
Para calcular , Sara teclea en la calculadora a  b  c  , y el resultado es 11 (a, b,
c
y c son enteros positivos). Luego teclea b  a  c  , y se sorprende al ver que el
resultado es 14. Se da cuenta de que la calculadora está diseñada para calcular las
ab
divisiones antes que las sumas. ¿Cuál es el resultado correcto de ?
c

A) 1 B) 2 C) 3 D) 4 E) 5

CANGURO 2019 N6 #20


6.22
Sea S el conjunto de todos los enteros positivos N con la siguiente propiedad: Los
últimos cuatro dígitos de N son 2020, y cuando se eliminan estos cuatro dígitos, el
resultado es un divisor de N. Por ejemplo, 42020 pertenece a S porque 4 es divisor de
42020. Determina la suma de todos los dígitos de todos los números de S. Por ejemplo,
el número 42020 contribuye con 4+2+0+2+0=8 a este total.

AIME I 2020 #4

6.23 D
Sean m y n enteros positivos satisfaciendo las siguientes condiciones:
- El máximo común divisor ( m  n , 210 )  1 .
- mm es un múltiplo de n n .
- m no es un múltiplo de n .

Determina el menor valor posible de m  n .

AIME I 2020 #10

6.24 MF
Determina la cantidad de números enteros positivos de cuatro cifras (es decir, enteros
entre 1000 y 9999, inclusive) que tienen solo cifras pares y son divisibles entre 5.

(A) 80 (B) 100 (C) 125 (D) 200 (E) 500

AMC 12A 2020 #4

6.25 F
Para todos los enteros n  9 , el valor de
(n  2)!(n  1)!
n!

(A) es un múltiple de 4 (B) es un múltiple de 10 (C) es un número primo


(D) es un cuadrado perfecto (E) es un cubo perfecto.

AMC 12B 2020 #6

6.26 D
Determina el número de enteros positivos n múltiplos de 5 tales que el mínimo común
múltiplo de 5! y n sea igual a 5 veces el máximo común divisor de 10! y n.

(A) 12 (B) 24 (C) 36 (D) 48 (E) 72

AMC 12A 2020 #21


6.27 D
Sean x, y enteros positivos satisfaciendo el siguiente sistema de ecuaciones:
log10 x  2 log10 Mcd ( x, y )   60

log10 y  2 log10 mcm( x, y )   570

Sea m el número de factores primos en la factorización de x (no necesariamente


diferentes), y sea n el número de factores primos en la factorización de y (no
necesariamente diferentes). Determina 3m  2n .

AIME I 2019 #7

6.28 D
Diremos que un entero positivo n es k-guapo si n tiene exactamente k divisores
positivos y es divisible entre k. Por ejemplo, 18 es 6-guapo. Sea S la suma de todos los
enteros positivos menores de 2019 que sean 20-guapos. Determina S/20.

AIME II 2019 #9

6.29 MF
Consideremos la afirmación "Si n no es primo, entonces n  2 es primo". ¿Cuáles de los
siguientes valores de n es un contraejemplo de esta afirmación?

(A) 11 (B) 15 (C) 19 (D) 21 (E) 27

AMC 12B 2019 #2

6.30 MF
Los caramelos de una tienda cuestan valores enteros en céntimos. Casper tiene
exactamente suficiente dinero para comprar 12 caramelos rojos, 14 caramelos verdes,
15 caramelos azules, o n caramelos púrpura (una de las cuatro opciones). Un caramelo
púrpura cuesta 20 céntimos. Determina el menor valor posible de n .

(A) 18 (B) 21 (C) 24 (D) 25 (E) 28

AMC 12B 2019 #5

6.31 M
Sea S el conjunto de todos los divisores enteros positivos de 100 000. ¿Cuántos
números son el producto de dos elementos diferentes de S?

(A) 98 (B) 100 (C) 117 (D) 119 (E) 121

AMC 10B 2019 #19, AMC 12B 2019 #14


6.32 MF
Determina el número de enteros n (no necesariamente positivos) para los cuales
n
2
4000   
5
es un entero.

(A) 3 (B) 4 (C) 6 (D) 8 (E) 9

AMC 12A 2018 #7

6.33 MF
Sea S un subconjunto de 6 elementos de 1,2,...,12  con la propiedad de que si a y b
son elementos de S y a  b , entonces b no es un múltiple de a . Determina el menor
valor posible de los elementos de S.

(A) 2 (B) 3 (C) 4 (D) 5 (E) 7

AMC 12A 2018 #12

6.34 M
Fijado un entero positivo n , y tres dígitos a, b, c diferentes de cero, sea An el entero de
n dígitos todos ellos iguales a a , sea Bn el entero de n dígitos todos ellos iguales a b ,
y sea Cn el entero de 2n dígitos (no de n dígitos) todos ellos iguales a c . Determina el
mayor valor posible de a  b  c para el que existen al menos dos valores de n tales que
Cn  Bn  An2 .

(A) 12 (B) 14 (C) 16 (D) 18 (E) 20

AMC 12A 2018 #25

6.35 F
Mary toma un número entero par de 4 dígitos n y escribe todos sus divisores en orden
n
creciente, de izquierda a derecha: 1, 2, ..., , n . En un cierto momento, Mary escribe el
2
número 323 como divisor de un cierto n. ¿Cuál es el menor entero divisor de n que
Mary escribirá a la derecha de 323?

(A) 324 (B) 330 (C) 340 (D) 361 (E) 646

AMC 12B 2018 #19


6.36 D
5 p 4
Sean p y q enteros positivos tales que   y siendo q tan pequeño como sea
9 q 7
posible. Determina q  p .

(A) 7 (B) 11 (C) 13 (D) 17 (E) 19


AMC 12B 2018 #17
6.37 MD
Supongamos que la ecuación cuadrática x 2  a x  b  0 tiene dos raíces reales, ambas
múltiples de 3, y que la ecuación x2  bx  9a  0 tiene una raíz doble. Determina a y
b.

6.38 M
Considera el siguiente par de números naturales de 4 cifras:

(m, n) = (2601, 1600) (es decir, m = 2601, n = 1600).

Fíjate que m y n verifican las siguientes propiedades:


1. Son números de 4 cifras (esto es, entre 1000 y 9999).
2. Son cuadrados perfectos.
3.Tienen las mismas cifras en exactamente dos de las cuatro posiciones. (En
nuestro ejemplo, en la segunda y tercera posiciones.)
4. En las dos posiciones en las que las cifras son distintas, la cifra que aparece en
m es igual a la que aparece en n más 1.
Encuentra todos los pares de números naturales que verifican estas cuatro propiedades.
OMEFL Aragón 2021 #2

6.39 F
8n  3
Determinar todos los enteros positivos n para los que la fracción
17n  9
es irreductible.

OMC 2021 #4

6.40 MF
¿Cuál de los siguientes números no es divisible por 3, cualquiera que sea el número
entero n?

A) 5n  1 B) n 2 C) n(n  1) D) 6n  1 E) n3  2

CANGURO 2020 N6 #10


6.41 F
Un número N es divisible por todos los enteros del 2 al 11, excepto por dos de ellos.
¿Cuál de las siguientes parejas de números puede ser la excepción?

A) 2 y 3 B) 4 y 5 C) 6 y 7 D) 7 y 8 E) 10 y 11

CANGURO 2020 N6 #24

6.42 M
Hay 71 canicas en una caja. Un juego consiste en sacar 30 canicas de la caja, si las hay,
o meter 18. Se puede jugar indefinidamente. ¿Cuál es la menor cantidad de canicas que
puede quedar en la caja en algún momento?

A) 1 B) 3 C) 5 D) 7 E) 11
CANGURO 2020 N6 #26
6.43 M
Hay n números primos p1 , p2 , ..., pn diferentes en la fila inferior de na tabla piramidal,
como se ve en la figura.

El producto de dos números contiguos de una fila se coloca en la fila superior, en la


casilla situada exactamente sobre ellos. El número K  p11  p2 2  ...  pn n está en la
casilla de la última fila. Si  2  8 , ¿cuántos números hay en la tabla que sean divisibles
por p4 ?

A) 4 B) 16 C) 24 D) 28 E) 36

CANGURO 2020 N6 #29

6.44 F
Marta puso un signo de multiplicación entre las cifras segunda y tercera del número
2020 y observó que el producto resultante 20 20 es un cuadrado perfecto. ¿Cuántos
números entre 2010 y 2099 (incluido 2020) tienen la misma propiedad?

A) 1 B) 2 C) 3 D) 4 E) 5

CANGURO 2020 N5 #16

6.45 F
Se forma un número de nueve cifras colocando aleatoriamente los dígitos del 1 al 9.
¿Cuál es la probabilidad de que el número resultante sea divisible por 18?

1 4 5 1 3
A) B) C) D) E)
2 9 9 3 4
Nota: En este problema se supone que no se repiten las cifras.

CANGURO 2020 N5 #21

6.46 M
Ocho enteros positivos consecutivos de tres cifras tienen la siguiente propiedad: cada
uno de ellos es divisible por la cifra de las unidades. ¿Cuál es la suma de las cifras del
menor de los ocho enteros?

A) 10 B) 11 C) 12 D) 13 E) 14

CANGURO 2020 N5 #30


6.47 MF
A lo largo de cuatro días, Linda viaja durante una hora, a una velocidad equivalente a
avanzar una milla en un número entero de minutos. Después del primero, su velocidad
disminuye de forma que el número de minutos que necesita para avanzar una milla se
incrementa en 5 minutos respecto del día anterior. En cada uno de los cuatro días la
distancia recorrida es también un número entero de millas. Determina el número total de
millas recorridas después de los cuatro días.

(A) 10 (B) 15 (C) 25 (D) 50 (E) 82

AMC 8 2017 #23

6.48 M
Determina todos los enteros no negativos a, b tales que

a  b  2009

British Math Olympiad 2009 Round 2 #1

6.49 MF
Si
n 1 n 5
  
n 1 n n 1 k

Para ciertos enteros positivos n y k, ¿Cuál es el valor de k?

(A) 1 (B) 5 (C) 24 (D) 25

SAT

6.50 MD
Determina todos los enteros a, b, c con 1  a  b  c para los cuales (a  1)(b  1)(c  1)
divide a abc  1

IMO 1992 #1

6.51 D
Determina las soluciones enteras y positivas a, b, c de la ecuación
 1  1  1 
1  1  1    2
 a  b  c 

British Math Olympiad 1995 #1

6.52 F
Resuelve la siguiente ecuación para x, y, z  IN
1 2 3
  1
x y z

British Mathematical Olympiad 1988 #4


6.53 F
Determina todas las soluciones enteras positivas n, m , con n impar, de la ecuación
1 4 1
 
m n 12

British Mathematical Olympiad 2001 Round 1 #1

6.54 F
Determina todos los pares a, b  de enteros positivos tales que
1 1 3
 
a b 2018
Putnam 2018 A1

6.55 M
El número de pares de enteros ( x, y) con 0  x  y tales que 1984  x  y es:

(A) 0 (B) 1 (C) 3 (D) 4 (E) 7

ASHME 1984 #28

6.56 M
El número n se escribe en base 14 como a b c , se escribe en base 15 como a c b y se
escribe en base 6 como a c a c , con a  0 . Determina el número n en base 10.

AIME I 2018 #2

6.57 F
Determina justificadamente todos los pares de números enteros ( x, y) que verifican la
ecuación x 2  y 4  2009 .

OME 2009 #4

6.58 F
El número de triples a, b, c  de números enteros positivos que satisfacen el sistema

ab  bc  44

ac  bc  23

es
(A) 0 (B) 1 (C) 2 (D) 3 (E) 4

AHSME 1984 #20


6.59 F
Determina el número de 7-tuplas de números positivos (a, b, c, d , e, f , g ) que satisfacen
el siguiente sistema de ecuaciones:
abc  70

cde  71
efg  72

AIME II 2019 #3

6.60 M
Determina 3x 2 y 2 si x, y son enteros que satisfacen y 2  3x 2 y 2  30 x 2  517 .

AIME 1987 #5

6.61 M
4 2
Resuelve la siguiente ecuación diofántica  1
m n
ASHME 1993 #19

6.62 F
Encuentra todos los primos p y q que satisfacen la ecuación p  q  ( p  q)3

Rusia, 2001

6.63 M
Resuelve el siguiente sistema de ecuaciones, con a, b, c enteros.
 a  b  c  19

 ab  c  97

ASHME 1997 #28

6.64 F

Resuelve la ecuación con solución entera x 2  x  1  x2
1

ASHME 1985 #21

6.65 M
Determina todas las tercias de enteros positivos a , b , c tales que abc  a  b  c  1 .

México 2010

6.66 D
Determina todas las soluciones enteras de la ecuación 2 x  1  3 y
6.67 F
Describir todas las soluciones enteras positivas m, n  de la ecuación
8m  7  n2

y dar el primer valor de m (si existe) mayor que 1959.

OMEFL 2017 #1

6.68 D
 
Calcular todos los pares de enteros x, y  tales que 3423 x 2  y 2  x3 y 3 .

OME 2019 #4

6.69 MF
Si x , y son enteros mayores que 1, y se cumple y  x  4 y   7 , determina el valor de x.

SAT Hard Problem


7 Aritmética modular.
El lenguaje de las congruencias nos permite abordar con éxito problemas aparentemente
muy difíciles. Las congruencias es un lenguaje, una técnica, y por tanto solo se aprende
practicando, jugando con ellas durante mucho tiempo.

Concepto de congruencias.
7.1 Definición. Congruencias.
Diremos que a  b (mod n) , y diremos que "a es congruente con b módulo n" cuando
sucede alguna de estas condiciones equivalentes:
a) n | (a  b)
b) a  k n  b para cierto entero k .
c) a y b dejan el mismo residuo cuando son divididos entre n.

Por ejemplo: 24  3 (mod 7) , pues 24  3  7  3 , 34  4 (mod 5) , pues 34  6  5  4


En particular: a  0 (mod n)  n | a

7.2 Ejemplo.
Determina x tal que 5x  6 (mod 8) .

Solución.
Vamos probando, uno por uno:
5 1  5  0  8  5  5 (mod 8) 5  2  10  1 8  2  2 (mod 8)
5  3  15  1 8  7  7 (mod 8) 5  4  20  2  8  4  4 (mod 8)
5  5  25  3  8  1  1 (mod 8)
5  6  30  3  8  6  6 (mod 8)  La solución es x  6 (mod 8)
5  7  35  4  8  3  3 (mod 8)

7.3 Proposición. Propiedades básicas de las congruencias.


Sea n  0 fijo, y a, b, c, d enteros arbitrarios. Entonces se cumple:

a) a  a (mod n) (Propiedad reflexiva).


b) Si a  b (mod n)  b  a (mod n) (Propiedad simétrica).
c) Si a  b (mod n) y b  c (mod n)  a  c (mod n) (Propiedad transitiva).
d) Si a  b (mod n) y c  d (mod n)  a  c  b  d (mod n) y a c  b d (mod n) .
e) Si a  b (mod n)  k a  k b (mod n) para cualquier entero k

f) Si a  b (mod n)  a k  b k (mod n) para cualquier entero positivo k .


7.4 Observación. Qué funciona y qué no funciona con congruencias.
Las propiedades anteriores nos permiten trabajar con congruencias prácticamente igual
a como trabajamos con números, pero no todo lo que hacíamos con números funciona
ahora con congruencias:

a) Cancelación de términos:
No funciona en general la cancelación de términos, el "tachar" de toda la vida.
Por ejemplo: 2  4  2  1 (mod 6) , pero 4  1 (mod 6)
217  21 (mod 7) , pero 216  1 (mod 7)

Aunque existe una Regla de cancelación:


Si (c, n)  1 , entonces ca  cb (mod n)  a  b (mod n)

b) Principio del producto nulo:


No existe tampoco el principio del producto nulo en general. Por ejemplo:
4  3  0 (mod 12) , pero 4  0 (mod 12) y 3  0 (mod 12)

Pero sí se verifica cuando el módulo es un número primo:


Si p es primo, a  b  0 (mod p)  a  0 (mod p) o b  0 (mod p)

7.5 Proposición. Modificaciones en el módulo.


 m
a) Si (a, m)  d  1 , entonces ax  ay (mod m)  ax  ay  mod  .
 d
b) Si a  b (mod n) y d | n  a  b (mod d )
c) a  b (mod n) y a  b (mod m)  a  b  mod n, m  .
d) Si n1 , n2 ,... , nk   1 , a  b (mod ni ) i  1,..., k  a  b (mod n1n2 ...nk )

Demostración.
a) ax  ay (mod m)  m | ax  ay   m | ax  y   | x  y 
m a
d d
b) a  b (mod n m)  n m | (a - b)  n | (a - b)  a  b (mod n)
a  b (mod n)  n | (a  b) 
c)   n, m | (a  b)  a  b  modn, m  )
a  b (mod m)  m | (a  b)
(ver 4.6b)
d) Es un caso particular de c.
Trabajar con congruencias es una técnica muy potente para resolver problemas de
Teoría de Números, como se puede ver en los siguientes ejemplos:

7.6 Problema resuelto.


Demostrar que 41 divide 220  1

Solución. En primer lugar vemos que 25  32  9 (mod 41)


 
Luego 220  25   9 (mod 41)   9  9 (mod 41)  81  81 (mod 41)
4 4 2 2

Pero, por otro lado, 81 (mod 41)  1 (mod 41)


Luego 81  81 (mod 41)  (1)(1) (mod 41)  1 (mod 41)
Finalmente, 220  1  1  1 (mod 41)  0 (mod 41)  41 | 220  1 , tal y como queríamos
ver.

7.7 Problema resuelto.


Determina el residuo al dividir 1! 2! 3! 4! ...  99!100! entre 12.

Solución. Observamos que 4! 24  0 (mod 12) , luego, para todo k  4 ,


k! 4!5  6...  k  0  5  6...  k  0 (mod 12)
luego
1!2!3!4!...  99!100! 1!2!3!4!0  ...  0  0  1!2!3!4! (mod 12)

Y nuestro problema se reduce a encontrar el residuo al dividir 1!2!3!4! 33 entre 12,


que es 9.

Un método efectivo para calcular potencias modulares.


7.8 Proposición. Método efectivo para calcular ab (mod n): “Método de las potencias de 2”
m
Calculamos las potencias a 2 (mod n) sucesivamente y en orden ascendiente. Después
descomponemos nuestra potencia ab (mod n) como producto de las anteriores. Como
tantas cosas en Teoría de Números y en general en matemáticas, lo mejor es observar un
ejemplo práctico:

7.9 Ejemplo.
Calcular 3172 (mod 191) .

Solución.
32  9 (mod 191)   2
34  32  81 (mod 191)
 2
38  34  6561  67 (mod 191) 316  3   4489  96 (mod 191)
8 2

332  3   9216  48 (mod 191)


16 2
364  3   2304  12 (mod 191)
32 2

 3   144 (mod 191)


2
3128 64

Y ahora, puesto que 172  128  32  8  4 ,


3172 (mod 191)  31283323834 (mod 191)  144  48  67  81 (mod 191)  170 (mod 191)
Podemos encontrar otros ejemplos de aplicación de este método en las soluciones de los
problemas #7.22 , #13.1 y #7.24.

7.10 MF
Si n ! denota el producto de todos los números del 1 al n, ¿Cuál es el residuo de
1! 2! 3! ...  n!
al dividirlo entre 9?

7.11 MF
Encuentra un ejemplo que demuestre que a 2  b 2 (mod n) no implica a  b (mod n) .

7.12 F
Determina los residuos cuando 2 50 y 4165 son divididos entre 7.

7.13 F
Utilizando la teoría de congruencias, demuestra que 89 | 244  1 y 97 | 2 48  1 .

7.14 F
Determina el último dígito de 91003  7902  3801 .

7.15 MF
Demuestra que para todo n  IN , el número an  11n  2  122 n 1 es divisible entre 133.

7.16 MF
La cifra de las unidades de 2137 753 es:

(A) 1 (B) 3 (C) 5 (D) 7 (E) 9

AHSME 1961 #28

7.17 F
El dígito de las unidades de 3100171002131003 es:

(A) 1 (B) 3 (C) 5 (D) 7 (E) 9

AHSME 1983 #14

7.18 F
Demuestra que 186  1 (mod 7k ) para k  1,2,3.
7.19 MF
Demuestra que si n es impar, entonces n 2  1 (mod 8)

7.20 F
Determina el número de enteros n , 1  n  25 , tales que n2  3n  2 es divisible entre
6.

7.21 F
Demuestra que 2n  6  9n siempre es divisible entre 7, para todo entero positivo n.

7.22 M
Sea an  6  8 . Determina el residuo cuando a83 se divide entre 49.
n n

AIME 1983 #6

7.23 F
Determina el residuo al dividir 9  99  999  ...  99
...
9 entre 1000.
999 nueves

AIME I 2010 #2

7.24 F
Consideremos el esquema triangular de números 0 ,1, 2 , 3... a lo largo de los lados y con
números interiores obtenidos sumando los dos números superiores de la fila anterior.
Las filas 1 a 6 se muestran en el siguiente esquema:

Sea f (n) la suma de los números de la fila n. ¿Cuál es el residuo cuando dividimos
f (100) entre 100?

AHSME 1995 #27

7.25 M
Sea k  20082  22008. ¿Cuál es el dígito de las unidades de k 2  2k ?

(A) 0 (B) 2 (C) 4 (D) 6 (E) 8

AMC12A 2008 #15, AMC 10A 2008 #24


7.26 F
Determina el menor entero positivo n tal que n y 107n tienen las dos últimas cifras
iguales.

HMMT 2008 #2

7.27 M
Demuestra que si x, y, z son enteros cumpliendo x 2  y 2  3z 2 , entonces x  y  z  0 .

7.28 F
Determina los enteros n tales que n  2 divide n  18 .
2

PUMaC 2007/NT #B2

7.29 MF
Demuestra que para todo natural n , el número 2n  1 no es divisible entre 7.

IMO 1967 #1 (apartado b)

7.30 MF
   ...
7
Determina el último dígito de  77
7 7
en donde aparece 1001 veces el número 7.

7.31 F
Dados cuatro números enteros diferentes a, b, c, d , demuestra que
a  ba  ca  d b  cb  d c  d 
es divisible entre 12
Congruencias con potencias y polinomios.

7.31 Proposición. Congruencias con potencias.


Las propiedades estudiadas en el Tema 4 tienen unas aplicaciones muy importantes en
el estudio de las congruencias:

Dados dos enteros positivos d , k , con d | k , entonces:


a) a  b (mod n)  a k  b k (mod n)
b) a d  b d (mod n)  a k  b k (mod n)
c) a d  b d (mod n) y k / d es impar,  a k  b k (mod n)

Demostración. a) Basta aplicar a  b | a k  b k para todo k.


b) Basta aplicar d | k  a d  b d | a k  b k .
c) a d  bd (mod n)  a d  b d  0 (mod n)  n | a d  bd (*)
Pero si k / d es impar tenemos a d  b d | a k  b k , luego
(*)  n | a k  bk  a k  b k  0 (mod n)  a k  bk (mod n)

7.32 Problema resuelto.


Aprovechando que 641  24  54  5  27  1 , demostrar que 641 | 232  1 .

641  2 4  54  2 4  54  0 (mod 641)  54  2 4 (mod 641)


 54 2 28  2 4 2 28 (mod 641)  54 2 28  232 (mod 641)

 5  27 
4
 232 (mod 641)  641  1  232 (mod 641)
4

  1  232 (mod 641)  1  232 (mod 641)  232  1  0 (mod 641)


4

 641 | 232  1

7.33 Teorema.
Dado un polinomio con coeficientes enteros p( x)  cm x m  cm1 x  ...  c1 x  c0 ,
entonces:
a  b (mod n)  p(a)  p(b) (mod n)

Demostración. a  b (mod n)  a k  bk (mod n)  ck a k  ck bk (mod n) 


m m
p(a)   ck a k  p(b)   ck b k (mod n)
k 0 k 0

7.34 F
Dado cualquier número positivo n, y sea S la suma de sus cifras, demuestra:
a) n  S es divisible entre 9.
b) n es divisible entre 9 si y solo si S es divisible entre 9.

En este problema justificamos el “criterio de divisibilidad del nueve”: Un número es


divisible entre nueve si y solo si las suma de sus cifras es divisible entre 9.
7.35 F
Demuestra el “criterio de divisibilidad del once”: Un número es divisible entre 11 si y
solo si la suma alternada de sus cifras es múltiplo de 11.

7.36 Proposición.
Dado un polinomio con coeficientes enteros p( x)  cm x m  cm1 x  ...  c1 x  c0 , diremos
que a es una solución de la congruencia p( x)  0 (mod n) si p(a)  0 (mod n) .
Si a es una solución de la congruencia p( x)  0 (mod n) y b  a (mod n) entonces b
también es una solución de la congruencia p( x)  0 (mod n) .

Demostración. a  b (mod n)  0  p(a)  p(b) (mod n)  0  p(b) (mod n)


Teorema del inverso modular. Lema de Gauss.

7.37 Teorema. Teorema del inverso modular.


Sean a, b enteros, se cumple (a, b)  1 si y solo si existe un entero x tal que
a x  1 (mod b) , y este valor es único módulo b. A x le llamaremos inverso de a
módulo b.

Demostración. Aplicando el TDB, si (a, b)  1 existirán x, y tales que a x  b y  1 , y


por tanto a x  1  by  ax  1 (mod b) .
Supongamos que existen dos valores x, x' tales que a x  1 (mod b) y a x'  1 (mod b) .
Entonces
x'  1x'  a xx'  a x' x  1x  x (mod b)

Recíprocamente, supongamos que a x  1 (mod b) , es decir, a x 1  yb para cierto


entero y ,
y por tanto a x  yb  1  a x  ( y)b  1 . Aplicando el TDB llegamos a (a, b)  1 .

7.38 Corolario. Lema de Gauss.


Si a, b, c son enteros tales que a | bc y (a, b)  1 , entonces a | c .

Demostración. (a, b)  1 , luego existirá un x tal que bx  1 (mod a) . Pero por otro lado,
a | bc  0  bc  xbc  cbx  c1  c (mod a)  a | c .

7.39 Corolario.
a) (a, b1 )  (a, b2 )  ...  (a, bn )  1   a , b1  b2  ...  bn   1
 
b) En particular, (a, b)  1  a , bk  1 para todo k  1 .

Demostración. a) (a, b1 )  (a, b2 )  ...  (a, bn )  1, luego existirán x1 , ..., xn tales que
xibi  1 (mod a) . Multiplicando tenemos que x1x2 ...xn b1b2 ...bn   1  1  ...  1  1 (mod a) ,
y por tanto  a , b1  b2  ...  bn   1 por el recíproco del Teorema del inverso modular.
b) Basta tomar en el apartado anterior b1  b2  ...  bk  b

7.40 Corolario.
a n | bn  a | b

Demostración. Si a  0 o b  0 está claro que a | b . Supongamos que a, b  0 .


Sea d  (a, b)  a  du , b  dv con (u, v)  1 .
Luego a n | bn  du  | dv   d nu n | d nv n  u n | v n  u | v n
n n

Por el corolario anterior, (u, v)  1  (u, v n )  1 , y por tanto u  1 , y en consecuencia


a  d , y por lo tanto divide a b  dv .
7.41 Corolario.
Si a | c y b | c con (a, b)  1 , entonces ab | c

Demostración. a | c  c  a d para cierto d .


b | ad 
  b | d por el Lema de Gauss, y por tanto a b | a d  c , como queríamos ver.
(a, b)  1

7.42 Teorema.
Sean a, b enteros positivos coprimos, tales que su producto es una potencia de grado n ,
es decir, ab  c n para cierto entero positivo c . Entonces a y b también son ambos
potencias de grado n .

Demostración. Sea d  (a, c) , y escribimos a  du y c  dv para ciertos u, v coprimos.


a  du 
n
 dub  (dv) n  d nv n  ub  d n 1v n (*)
ab  c 

De la igualdad anterior se desprende que u | d n 1v n , pero por hipótesis,


u
(u, v)  1  (u, v n )  1 , y por tanto u | d n 1 . Así pues, v n  n 1 b , es decir, b | v n .
d
Pero de la igualdad (*) también se deduce que v | ub , y puesto que (u, v n )  1 ,
n

llegamos a v n | b , y finalmente:
b | v n 
b v
n

v | b
n

Sustituyendo en (*) se deduce u  d n 1  a  d n , con lo que se concluye la


demostración.

7.43 Problema resuelto.


Demuestra que el producto de tres números consecutivos nunca puede ser una potencia
perfecta.

Solución. Supongamos que (n  1)n(n  1)  a k , para ciertos enteros a, k  1.


Entonces podemos escribir n(n2  1)  a k . Puesto que (n, n2  1)  1 podemos aplicar el
teorema anterior: n  bk , n2  1  c k para ciertos enteros b, c  1 .
  2
  k
 
n2  1  c k  1  n2  c k  bk  c k  b2  c k  b2  c b2( k 1)  ...  c k 1 
Lo cual es imposible pues b2  c  1 y b2( k 1)  ...  ck 1  k  1.
El problema de las monedas.

Dados dos números positivos m, n , queremos estudiar las combinaciones lineales


a m  bn

con coeficientes enteros no negativos a, b  0 .

Este problema se llama problema de las monedas de Frobenius o simplemente problema


de Frobenius, pues este matemático alemán estudió las posibles sumas de dinero que se
podían obtener acumulando ciertos tipos de moneda.

7.43 El teorema “Chicken McNugget”.


Dados dos números positivos coprimos m, n , el mayor entero que no puede
representarse como combinación lineal a m  bn con a, b enteros no negativos es
mn  m  n

Una consecuencia de este teorema es que existen exactamente


m  1 n  1
2
enteros positivos que no se pueden representar de dicha forma.

Se dice que el origen de este teorema y de su curioso nombre viene de cuando


McDonalds ofrecía sus nuggets en paquetes de 9 y de 20, y algunos aficionados a las
matemáticas se preguntaron por la cantidad máxima que no se podían comprar en cajas
completas (son 151).

7.44 MF
En un pueblo llamado Hamlet hay 3 personas por cada caballo, 4 ovejas por cada vaca,
y 3 patos por cada persona. ¿Cuál de los siguientes valores no puede ser la suma de
personas, caballos, ovejas, vacas y patos en Hamlet?

(A) 41 (B) 47 (C) 59 (D) 61 (E) 66


AMC 10B 2015 #15

7.45 D
Determina todos los posibles valores de enteros positivos n para los cuales 91 céntimos
es el mayor valor que no se puede formar disponiendo de una infinita cantidad de sellos
de 5, n y n  1 céntimos.

AIME II 2019 #14


8 Congruencias y sistemas de congruencias lineales.
Congruencias lineales.
8.1 Definición. Congruencia lineal.
Llamamos congruencia lineal a toda ecuación de la forma a x  b (mod n)
Diremos que el entero x0 satisface la congruencia lineal a x  b (mod n) cuando
a x0  b (mod n)

O equivalentemente: a x0  b (mod n)  a x0  ny0  b  a x0  n y0  b

Es decir, buscamos soluciones x0 , y0  de la ecuación lineal diofántica


a x0  n y0  b

8.2 Teorema. Existencia de soluciones de una congruencia lineal.


La congruencia lineal a x  b (mod n) tiene solución si y solo si d | b , donde
d  (a, n) , en cuyo caso existen d soluciones diferentes (aquí se entiende diferentes
como mutuamente incongruentes), todas ellas de la forma

n n n n
xo , x1  x0  , x2  x0   2 , x3  x0   3 , ... , xd 1  x0   (d  1)
d d d d

Donde x0 es una solución particular de la ecuación.

8.3 Ejemplo.
Resuelve la congruencia 18x  30 (mod 42)

Solución. d  (18,42)  6 , y 6 | 30 , luego la ecuación anterior tiene seis soluciones.


Por tanteo, vemos que 18  4  72  42  1  30  4 es una solución de la ecuación.
Luego las soluciones serán:
x0  4
x1  4  (42 / 6)  1  4  7  11, efectivamente: 18  11  198  4  42  30
x2  4  (46 / 6)  2  4  14  18 , efectivamente: 18  18  324  7  42  30
x3  4  (46 / 6)  3  4  21  25 , efectivamente: 18  25  450  10  42  30
x4  4  (46 / 6)  4  4  28  32 , efectivamente: 18  32  576  13  42  30
x5  4  (46 / 6)  5  4  35  39 , efectivamente: 18  39  630  16  42  30

Las soluciones son:  4 ,11,18 , 25 , 32 , 39 

8.4 F
Resuelve la congruencia 9 x  21 (mod 30)

8.5 F
Resuelve la congruencia lineal 3x  7 (mod 10)
8.6 Definición. Inversos modulares.
Diremos que a y b son inversos módulo n si a b  1 (mod n) , o equivalentemente,
diremos que b es el inverso de a módulo n.
La congruencia lineal a x  1 (mod n) tiene solución si y solo si (a, n) | 1 , es decir,
cuando (a, n)  1 , así pues, existirá el inverso multiplicativo de a si y solo si (a, n)  1 , y
será único modulo n.

8.7 Ejemplos.
El inverso de 3 módulo 4 es 3 porque 3  3  9  1 (mod 4) .
El inverso de 3 módulo 5 es 2 porque 3  2  6  1 (mod 5) .

8.8 F
Determina el inverso de 9 módulo 82.

Problema resuelto.
Determina todas las parejas de inversos módulo 20.

Sabemos que serán todos los números coprimos con 20, es decir:
1, 3, 7 , 9 ,11,13,17 ,19 
Ahora solo nos queda agruparlos por parejas (o consigo mismo)
El 1 es inverso de sí mismo, pues 1  1  1  1 (mod 20)  11  1 (mod 20)
Vemos que 3  7  21  1 (mod 20)  31  7 (mod 20) , 71  3 (mod 20)
Vemos que 17  13  221  1 (mod 20)  171  13 (mod 20) , 131  17 (mod 20)
El 11 es inverso de sí mismo, pues 11  11  121  1 (mod 20)  111  11 (mod 20)
El 9 es inverso de sí mismo, pues 9  9  81  1 (mod 20)  91  9 (mod 20)
Finalmente, el 19 también es inverso de sí mismo, pues
19  19  381  1 (mod 20)  191  19 (mod 20)
Otra manera de verlo es observar que 19  1 (mod 20)  192  (1)2  1 (mod 20)

Problema resuelto. Aplicación del Teorema de Bezout para determinar inversos.


Determina 341 mod 143

Solución. Sabemos que, puesto que 34 , 143  1 , existirán enteros x, y tales que
34 x  143 y  1 , y por tanto 34 x  1  143 y  34 x  1 mod 143 .

El Algoritmo de Euclides (4.17) nos permite determinar esta combinación lineal:


143  34  4  7  143,34  34  4  7,34  7,34
34  4  7  6  7,34  7,4  7  6  7,6  1

Luego
1  7  6  7  34  4  7   7  34  4  7  34  5  7  34  5  143  34  4 
 34  5  143  34  4  5  5  143  21  34
Y por tanto 341  21  122 mod 143
Problema resuelto.
Demostrar que las seis últimas cifras de 7 9999 son 857143.

Solución. Las claves de este problema son demostrar que


7  79999  710000  1 mod 106  
y observar que
6000001 6  106
857143 
7

7
 7  857143  1 mod 106  

 
Demostrar que 710000  1 mod 106 se puede hacer "por fuerza bruta", o mediante el
Binomio de Newton: 74  2401  24 102  1 , luego
 
710000  7 4
2500

 24  10 2  1 
2500

 2500   2500   2500 
 1    24  10 2    24  10 2
  
2 3
 24  10 2  ... 
 1   2   3 
 2500  3
2
 1  2500  24  10 2  1250  2499 24  10 2    
 24  10 6  ...
 3 
 1  3  2 7  5 6  2 7  33  5 4  7 2  17  10 4  ...
 1  3  2  10 6  2 5  33  5 2  7 2  17  10 6  ...
 
 1  3  2  2 5  33  5 2  7 2  17  ...  10 6  710000  1 mod 10 6  
6000001 6  106
Por otro lado, observamos que 857143  
7 7
Con lo cual:
 
710000  1 mod 106  710000  k  106  1  k  106  6  106  6  106  1 
 (k  6)  106  6  106  1

 
(k  6)  106  710000  6  106  1 ambos múltiples de 7, luego k  6 será también
múltiplo de 7, es decir: 710000  7k '106  6  106  1

Y por tanto, finalmente:


710000 7k '106  6  106  1
79999 
  k '106  857143
7 7
tal y como queríamos ver.

Fuente: Jerónimo Vega Guillén en Facebook.

Observamos que

7  7 9999  1 mod 106 
 
 7 9999  857143 mod 106  
6 

7  857143  1 mod 10  
Es consecuencia del teorema anterior, puesto que 7 ,106  1  
Propiedad. Simplificación de congruencias lineales.
na x  nb mod nc  a x  b mod c

Problema resuelto.
Resuelve la congruencia
140 x  56 mod 252

Solución. Esta congruencia tiene solución pues d  140,252  28 y 28 | 56 .


Vemos que se puede simplificar pues todos los números involucrados son múltiples de
28:
140 x  56 mod 252  5x  2 mod 9

Para resolver esta última congruencia vemos que el inverso de 5 módulo 9 es 2:


5  2  10  1 mod 9

Luego
5x  2 mod 9  x  51  2  2  2  4 mod 9

Las 28 soluciones módulo 252 son, por tanto:

4 , 4  9 , 4  18 , 4  27 , 4  36 , ...
Sistemas de congruencias lineales.

8.9 Sistemas de congruencias lineales (caso particular).

Queremos resolver ahora un sistema de congruencias lineales:


a1 x  b1 mod m1 
a x  b mod m 
 2 2 2

 ...
ar x  br mod mr 

En donde vamos a suponer que los módulos mi son todos coprimos entre ellos.
Evidentemente, el sistema tendrá solución cuando cada ecuación la tenga
individualmente, y por tanto
d k | b k , donde d k  ak , mk  para todo 1  k  r

8.10 Ejemplo. Resolución de s.c.l. mediante el método interactivo.


 x  3 (mod 4)

Resuelve el sistema  x  1 (mod 5)
 x  2 (mod 3)

Solución.
(Más adelante, mediante el Teorema chino del residuo, se verá que la solución existe y
es única mod 60, pues (4,5)  (5,3)  (4,3) )

x  3 (mod 4)  x  4a  3 
x  1 (mod 5)  4a  3  1 (mod 5)  4a  2 (mod 5)  2  5 (mod 5)  3 (mod 5)
4a  3 (mod 5)  4  4a  4  3 (mod 5)  16a  12 (mod 5)  a  2 (mod 5) 
a  5b  2  x  4a  3  45b  2  3  20b  11

Luego:
x  2 (mod 3)  20b  11  2 (mod 3)  20b  9 (mod 3)  0 (mod 3)
 b  0 (mod 3)  b  3c

Finalmente:
x  20b  11  20  3c  11  11  60c (mod 60) . En efecto:
11  4  2  3 , 11  5  2  1 , 11  3  3  2
8.11 Teorema. Teorema chino del residuo.
Sean m1 , m2 , ..., mr enteros positivos tales que mi , m j   1 si i  j . Entonces el sistema
de congruencias lineales
a1 x  b1 mod m1 
a x  b mod m 
 2 2 2

...
ar x  br mod mr 

Tiene una única solución (módulo el entero m1  m2  ...  mr ).

Y se obtiene siguiendo los siguientes pasos:


Paso 1: Sea N  m1  m2  ...  mr ,
Paso 2: Sean N1  N / m1 , N2  N / m2 , ... , Nr  N / mr .
Paso 3: Resolver las congruencias lineales:
N1 y1  1 mod m1  , N 2 y2  1 mod m2  , ... , N r yr  1 mod mr 

Paso 4: x  N1 y1 b1  N 2 y2 b2  ...  N r yr br (mod N ) es la única solución del


sistema.

8.12 Ejemplo. El problema de Sun-Tsu.


El Teorema chino del residuo debe su nombre en honor al siguiente problema del siglo I
DC: Determina un número cuyos residuos son 2, 3 y 2 al dividirlo entre 3, 5 y 7,
respectivamente.

Nota: Este mismo problema aparece en las Introductio Arithmeticae del matemático
griego Nicómano de Gerasa, alrededor del 100 DC.

Solución. Se trata de resolver el sistema de congruencias lineales


 x  2 (mod 3)

 x  3 (mod 5)
 x  2 (mod 7)

Paso 1: N  3  5  7  105
105 105 105
Paso 2: N1   35 , N 2   21 , N3   15
3 5 7
Paso 3: Las congruencias lineales 35 y1  1 (mod 3) , 21y2  1 (mod 5) y 15 y3  1 (mod 7)
tienen soluciones y1  2 , y2  1 y y3  1 .

Paso 4: x  35  2  2  21 1  3  15  1 2  233 será solución del sistema módulo 105, y


233  23 (mod105)

Y por tanto 23 es la única solución del sistema (módulo 105).


En efecto: 23  7  3  2 , 23  4  5  3 , 23  3  7  2
8.13 Ejemplo resuelto.
 x  2 (mod 4)
Resolver el sistema de congruencias 
 x  7 (mod 9)

Solución.
Claramente (4,9)  1 y por tanto el sistema tiene solución.
Paso 1: N  4  9  36
36 36
Paso 2: N1   9 , N2   4.
4 9
Paso 3: Resolvemos las ecuaciones 9 y1  1 (mod 4) y 4 y2  1 (mod 9)
Puesto que 9  1  2  4  1 , y 4  7  28  3  9  1 , tenemos que y1  1 , y2  7 son
soluciones.

Paso 4: La solución es x  9  1  2  4  7  7  214 (mod 36) , es decir, 34.

Efectivamente, 34  4  8  2 , y 34  9  3  7 ,
Sistemas de congruencias lineales (caso general).
Se pueden resolver sistemas de congruencias incluso cuando sus módulos no son
necesariamente coprimos. El criterio es similar al de las ecuaciones diofánticas lineales.

8.14 Teorema.
Dado el sistema
 x  a1 (mod m1 )

 x  a2 (mod m2 )

Si m1,m2  no es divisor de a1  a2 , el sistema no tiene solución.


En caso contrario, existe una única solución mod a1 , a2 

Observamos que el Teorema chino del residuo sería un caso particular de este teorema
cuando m1, m2   1 , pues entonces garantizamos que el sistema tenga solución, y
a1, a2   m1 m2 .
8.15 Ejemplo.
Resolver el sistema
 x  5 (mod 12)

 x  11 (mod 18)

Solución.
Puesto que (12,18)  6 , y 6 | (11 - 5) , existirá una única solución. La vamos a obtener
con el método interactivo.

x  5 (mod12)  x  12a  5
x  11 (mod18)  12a  5  11 (mod18)  12a  11  5 (mod18)  12a  6 (mod18)

Esta última congruencia se puede simplificar: 6 divide a 12 y a 6, y además


mcd (6,18)  6 , luego podemos simplificarla:

12a  6 (mod18)  2a  1 (mod 3)  a  2 (mod 3)  a  3k  2


x  12a  5  123k  2  5  36k  29 mod(36)

Donde hemos tenido en cuenta que 12,18  36

Efectivamente, 29  12  2  5 , y 29  18  1  11
8.16 Ejemplo. Como calcular congruencias cuando el módulo no es primo.
Calcular 20182018 (mod 26) .

Solución.
Puesto que 26 no es primo, no podemos aplicar directamente el PTF. Puesto que
26  2  13 ,
vamos a calcular por separado 20182018 (mod 2) y 20182018 (mod 13) , y después
aplicaremos el Teorema Chino del Residuo para determinar el resultado del enunciado.

Está claro que 2018  0 (mod 2) y por tanto 20182018  02018  0 (mod 2) .

2018  13  155  3 , luego 13 | 2018 , y por tanto podemos aplicar el PTF:


201812  1 (mod13) .

Por otro lado, 2018  12  168  2 , luego:


 
20182018  201812168 2  201812 20182  1 20182  20182 (mod 13)
168 168

2018  13  155  3  2018  3 (mod13)  20182  32  9 (mod13)

De todo lo anterior tenemos:



2018
2018
 0 (mod 2)


2018
2018
 9 (mod 13)

Y aplicamos el Teorema Chino del Residuo:


N  2  13  26
N1  13
N2  2
No hace falta resolver la congruencia 13 y1  1(mod 2) pues b1  0 .
Resolvemos la congruencia 2 y2  1(mod13)  y2  7 ,
Luego 20182018  13  y1  0  2  7  9  126 (mod 26)  22 (mod 26)

8.17 Observación.
Para calcular potencias elevadas con módulos no primos disponemos de dos técnicas:
El “método de las potencias de dos” (ver 7.8) y el método que acabamos de ver:
descomponer el módulo y aplicar el Teorema Chino del Residuo. Es importante
dominar estas dos técnicas, pues son la clave para resolver muchísimos problemas de
Aritmética. Se propone resolver el siguiente problema mediante las dos técnicas
anteriores:

8.18 F
Determina los dos últimos dígitos de 10321032.

HMMT 2009
8.19 Congruencias lineales que se resuelven mediante sistemas de congruencias lineales.

El siguiente resultado nos puede ser útil para resolver congruencias lineales:

a x  b (mod n m)  a x  b (mod n) y a x  b (mod m)

En efecto: a x  b (mod n m)  a x  k nm  b  km n  b  a x  b (mod n)

Existe un recíproco: a x  b (mod n) y a x  b (mod m)  a x  b mod n, m 

8.20 Ejemplo.
Resolver la congruencia lineal 17 x  9 (mod 276) .

Solución.
Puesto que 276  4  3  23 , la ecuación anterior es equivalente a resolver el sistema de
congruencias
17 x  9 (mod 3)  x  0 (mod 3)
 
17 x  9 (mod 4) o equivalentemente:  x  1 (mod 4)
17 x  9 (mod 23) 17 x  9 (mod 23)
 

x  0 (mod 3) equivale a decir que x  3a , luego sustituyendo en la segunda ecuación y


multiplicando por 3 ambos lados:
3a  1 (mod 4)  9a  3 (mod 4)  a  3 (mod 4)  a  4b  3  x  3a  12b  9

Sustituyendo en la tercera ecuación:


17 x  9 (mod 23)  1712b  9  9 (mod 23)  204b  153  9 (mod 23)
 204b  144 (mod 23)  20b  17 (mod 23)  3b  6 (mod 23)
 3b  6 (mod 23)  b  2 (mod 23)  b  23k  2
 x  12(23k  2)  9  276k  24  9  276k  33  x  33 (mod 276)

Efectivamente, 17  33  561  2  276  9


9 Congruencias cuadráticas.
9.1 Ejemplo.
Resuelve la ecuación x 2  1 (mod 144)

Solución.
Puesto que 144  16  9 , y (16,9)  1 , podemos descomponer la ecuación anterior en el
sistema no lineal
 x 2  1 (mod 16)
 2
 x  1 (mod 9)
x 2  1 (mod 16) tiene 4 soluciones: x  1 o  7 (mod 16)
x 2  1 (mod 9) tiene 2 soluciones: x  1 (mod 9)
Luego tenemos ocho alternativas:
i) x  1 (mod16) y x  1 (mod 9)
ii) x  1 (mod16) y x  1 (mod 9)
iii) x  1 (mod16) y x  1 (mod 9)
iv) x  1 (mod16) y x  1 (mod 9)
v) x  7 (mod16) y x  1 (mod 9)
vi) x  7 (mod16) y x  1 (mod 9)
vii) x  7 (mod16) y x  1 (mod 9)
viii) x  7 (mod16) y x  1 (mod 9)

Podemos ir resolviendo cada caso mediante el TCR.


Independientemente del caso, (16,9)  1 , luego todos los ocho sistemas tienen solución.
Además: N1  9 , N2  16 , 9 y1  1 (mod16)  y1  9 , 16 y2  1 (mod 9)  y2  4
i) x  9  9  1  16  4  1  145 (mod144)  1 (mod144)
ii) x  9  9  1  16  4  (1)  17 (mod144)
iii) x  9  9  (1)  16  4  1  17 (mod144)
iv) x  9  9  (1)  16  4  (1)  145 (mod144)  1 (mod144)
v) x  9  9  7  16  4  1  631 (mod144)  55 (mod144)
vi) x  9  9  7  16  4  (1)  503 (mod144)  71 (mod144)
vii) x  9  9  (7)  16  4  1  503 (mod144)  73 (mod144)  71 (mod144)
viii) x  9  9  (7)  16  4  (1)  631 (mod144)  55 (mod144)

9.2 F
Calcula los tres últimos dígitos de 200511  200512  ...  20052006

Senior Hanoi Open MO 2006

9.3 F
Demostrar que si x es un número impar no divisible entre tres, entonces x 2  1 mod 24 .
Ejemplo resuelto.
Resuelva la congruencia x 2  4 (mod 77)

Solución:


 x  4 (mod 7)  x  2 (mod 7)
2

x 2  4 (mod 77)   2

 x  4 (mod 11)  x  2 (mod 11)

Luego tenemos cuatro casos:


a) x  2 (mod 7) , x  2 (mod 11)
b) x  2 (mod 7) , x  2 (mod 11)
c) x  2 (mod 7) , x  2 (mod 11)
d) x  2 (mod 7) , x  2 (mod 11)

Cada uno de estos casos se resuelve por el TCR

a)
N  77
N1  11  11y1  1 (mod 7)  y1  2
N 2  7  7 y2  1 (mod 11)  y2  8
x  11  2  2  7  8  2  156  77  2  2  2 (mod 77)
Y por tanto la primera solución es x  2 (mod 77)

Para ahorrarnos trabajo, podemos deducir el resto de los casos directamente de la última
igualdad:

b) x  11 2  (2)  7  8  2  68 (mod 77)


c) x  11 2  2  7  8  (2)  68  68  77  9 (mod 77)
d) x  11 2  (2)  7  8  (2)  156  156  77  3  75 (mod 77)

Así pues, las soluciones son x  2 , 9 , 68 , 75 (mod 77)

Fuente: “PROFMAT MA14 12.2” en Youtube.


Definición. Residuo cuadrático módulo n.
Sean a, n enteros con n  0 y (a, n)  1. Diremos que a es un residuo cuadrático
módulo n si
x2  a mod n
tiene solución.

Resolución de congruencias cuadráticas.


Supongamos que queremos resolver la congruencia

ax2  bx  c  0 mod p  para cierto primo p y con p | a .

ax 2  bx  c  0 mod p    c 
ax 2  bx  c mod p    4a 
4a 2 x 2  4abx  4ac mod p    b 2 
4a 2 x 2  4abx  b 2  4ac  b 2 mod p  
(2ax  b) 2  b 2  4ac mod p 
Luego tendrá solución si y solo si b2  4ac es un residuo cuadrático módulo p, o bien
p | b2  4ac , en cuyo caso tendríamos la congruencia
(2ax  b)2  0 mod p   2ax  b  0 mod p 

Veamos este método en el siguiente ejemplo resuelto.

Ejercicio resuelto.
Resolver la congruencia
x 2  3x  5  0 mod 7

Solución. Siguiendo los pasos anteriores llegamos a la congruencia equivalente


2 x  32  1 mod 7

Y vemos que 1 es un residuo cuadrático módulo 7. En efecto: 12  1 mod 7 y


62  36  1 mod 7 . Luego nuestro problema se reduce a resolver las siguientes
congruencias lineales:
2 x  3  1 mod 7 y 2 x  3  6 mod 7

2 x  3  1 mod 7  2 x  2 mod 7  x  1  6 mod 7

2 x  3  6 mod 7   2 x  3 mod 7   x  21  3 mod 7  


x  4  3 mod 7   x  12 mod 7   x  5 mod 7 
En donde hemos utilizado que 2  4  8  1 mod 7  21  4 mod 7

Fuente: “Michael Penn Math” https://youtu.be/oPZbKUwBh4s


10 Ecuaciones diofánticas.
10.1 Ecuaciones diofánticas lineales.
Definición. Ecuaciones diofánticas lineales.
Son las que tienen la forma a x  b y  c , con a, b, c enteros, y a, b no ambos cero.

Por ejemplo, la ecuación 3x  6 y  18 tiene infinitas soluciones:


x  4 , y  1  3  4  6  1  18
x  6 , y  6  3  (6)  6  6  18
x  10 , y  2  3  10  6  (2)  18
En general, cualquier valor x  2k , y  3  k  3(2k )  6(3  k )  6k  18  3k  18

Sin embargo, la ecuación 2 x  10 y  17 no tiene solución: Para cualquier x, y , la parte


izquierda de la ecuación será par, mientras que la parte derecha es un impar.

Teorema fundamental de las ecuaciones diofánticas lineales.


Una ecuación diofántica lineal a x  b y  c tendrá solución si y solo si (a, b) | c .

Si  x0 , y0  es una solución particular de esta ecuación, entonces todas las soluciones


son de la forma
b a
x  x0    k , y  y0    k para cualquier k entero.
d  d 

Ejemplo resuelto.
Resuelve la ecuación diofántica 172 x  20 y  1000

Solución.
d  (172,20)  4 , y 4 | 1000 , luego esta ecuación tiene solución.

Probando números vemos que 172  5  20  7  1000 , luego x0  5 , y0  7 es una


solución particular de la ecuación.
Por el Teorema anterior, las soluciones de esta ecuación son todas las parejas de la
forma
 20   172 
x  5    k  5  5 k  5(1  k ) , y  7    k  7  43 k , con k  Z
 4   4 
o equivalentemente, tomando q  1  k ,
x  5q , y  7  43(q  1)  7  43q  43  50  43q , con q  Z .

10.1.1 F
Resuelve la ecuación diofántica 7 x  9 y  3
10.1.2 F
Un cliente compra una docena de piezas de fruta, manzanas y naranjas, por 1.32€. Si
una manzana cuesta 3 céntimos más que una naranja, y se compraron más manzanas
que naranjas, cuantas piezas de cada fueron compradas?

Resolución de ecuaciones diofánticas lineales mediante el Algoritmo de Euclides.


El Algoritmo de Euclides para el cálculo del máximo común divisor de dos números
mediante sucesivas divisiones nos permite resolver ecuaciones diofánticas lineales. Lo
veremos con el siguiente ejemplo:

Resuelve la ecuación diofántica 2173x  2491y  53

Solución. Aplicamos el algoritmo de Euclides:


2491  1  2173  318
2173  6  318  265 
  (2173,2491)  53 , y claramente 53 | 53 , luego hay solución.
318  1  265  53 
265  5  53  0 
Deshaciendo los pasos del algoritmo de Euclides:
53  318  1  265 

265  2173  6  318   53  318  1  265  2491  1  2173  1  2173  6  318 
318  2491  1  2173
 2491  1  2173  2173  6  2491  1  2173 
 2491  1  2173  2173  6  2491  6  2173 
 7  2491  8  2173
Luego x  8 , y  7 es una solución de la ecuación diofántica del enunciado.
El conjunto de soluciones de la ecuación serán las parejas de la forma:
2491 2173
x  8  k  8  47k , y  7  k  7  41k
53 53

10.1.3 F
Resuelve la ecuación diofántica 858x  253 y  33 mediante el algoritmo de Euclides.

10.1.4 F
Resuelve la ecuación diofántica 258x  147 y  369

10.1.5 F
Resuelve la ecuación diofántica 60 x  33 y  9

10.1.6 F
Las medidas (en grados) de los ángulos interiores de un hexágono convexo forman una
sucesión aritmética de enteros positivos. Sea mº la medida del mayor de los ángulos
interiores de este hexágono. El mayor valor posible de mº es

(A) 165º (B) 167º (C) 170º (D) 175º (E) 179º

AHSME 1991 #12


10.1.7 F
Determina un número que, cuando se divide entre 10, deja un residuo de 9, cuando se
divide entre 9 deja un residuo de 8, entre 8 el residuo es 7, y así sucesivamente, hasta
que, finalmente, cuando se divide entre 2, deja un residuo de 1.

ASHME 1951 #37


10.2 Ternas pitagóricas.

Definición. Terna Pitagórica.


Definimos por Terna Pitagórica las ternas ( x, y, z ) de números enteros solución de la
ecuación diofántica
x2  y 2  z 2

Las ternas pitagóricas son un caso particular de la llamada Ecuación de Fermat:


xn  y n  z n

Que fue resuelta por Wiles en 1994, más de 350 años después de que este problema
fuera propuesto, demostrando que para n  2 no existen soluciones que no sean la
trivial x  y  z  0 .

Lema.
Si ( x, y, z ) es una terna pitagórica y d  mcd ( x, y, z ) , entonces:
a) d  mcd ( x, y)  mcd ( y, z)  mcd ( x, z)
b) Si escribimos x  d x' , y  d y' , z  d z ' , entonces ( x' , y' , z' ) también es una terna
pitagórica.

Demostración.
a) Demostraremos la primera igualdad. Está claro que
d | x,d | y,d | zd | x,d | y
Por otro lado:

d | x  d 2 | x2 
2
 d 2 | x2  y 2  z 2  d 2 | z 2  d | z
d| yd | y 
2

b) Es trivial.

Definición. Terna pitagórica primitiva.


Una terna pitagórica ( x, y, z ) se dice que es primitiva si mcd ( x, y, z) , o
equivalentemente mcd ( x, y)  mcd ( y, z)  mcd ( x, z)  1

Lema.
Sea ( x, y, z ) una terna pitagórica primitiva. Entonces uno y solo uno de los tres números
es par, y siempre es x o y .

Demostración.
x par  x 2 par  x imp  x 2 imp 

  z  x  y imp ,   z  x  y imp
2 2 2 2 2 2

y imp  y imp 
2
 y par  y par 
2

Recordemos que todo cuadrado perfecto es 0 o 1 módulo 4. Luego
x imp  x 2 imp  x 2  1 (mod 4) 

  z  x  y  2 (mod 4) absurdo.
2 2 2

y imp  y imp  y  1 (mod 4)


2 2

Puesto que estamos suponiendo ternas pitagóricas primitivas, el caso x,y ambos pares
no se puede dar.
Teorema. Caracterización de las ternas pitagóricas.
Las ternas pitagóricas primitivas son todas las ternas de enteros que se pueden escribir
de la forma

 x  m2  n2  x  2mn
 
 y  2mn  y  m  n (con x par)
2 2
(con y par) o bien
 z  m2  n2  z  m2  n2
 

Donde m  n  0 son números coprimos con paridad diferente.

Demostración.
Que las ternas de enteros anteriores son pitagóricas es fácil de comprobar:
  
x2  y 2  z 2  m2  n2  2mn  m2  n2
2 2
 2

Simplificando:
m      
 n2  2mn  m2  n2  m2  n2  4m2n2  m2  n2 , que es una
2 2 2 2 2
2

aplicación directa de la igualdad (a  b)2  4ab  (a  b)2 .

Veamos que si una terna es pitagórica, necesariamente debe ser como las anteriores.
Vamos a demostrar la columna de la izquierda, es decir, cuando x es impar y y es par.

x2  y 2  z 2 se puede escribir como


y 2  z 2  x 2  z  x z  x 
Y por tanto:
y2 z  x z  x
2
 y
    
2 4 2 2
zx zx zx zx
Teniendo en cuenta que   z,   x , y mcd ( x, z )  1 ,
2 2 2 2
llegamos a
 zx zx 
mcd  ,  1
 2 2 

zx zx
Y por tanto, aplicando 5.2.26b, y son cuadrados perfectos, es decir:
2 2
zx zx
 n2 y  m2 para ciertos enteros n y m.
2 2
Se cumple claramente que m  n y son enteros coprimos.
También se cumple que m y n tienen diferente paridad, puesto que z  m2  n2 es impar.
Y se cumple x  m2  n2 , y  2mn , z  m2  n2 , tal y como queríamos ver.

10.2.1 D
Calcula todos los números enteros a, b y c tales que a 2  2b2  3c 2
OMEFL 2011 #4
10.3 La ecuación diofántica x2  y 2  k .

Lema.
Un elemento importante para resolver la ecuación x 2  y 2  k es observar que, por cada
posible factorización a  b  k ,
x  y  n
x 2  y 2  k  ( x  y )( x  y )  n  m  k  
x  y  m

Y el sistema de ecuaciones de la derecha tiene como única solución


nm
Sumando las dos ecuaciones: 2 x  n  m  x 
2
nm
Restando las dos ecuaciones: 2 y  n  m  y 
2

Aunque no todas las soluciones en x, y serán aceptables por no ser enteras: Está claro
que n  m y n  m deben ser ambos pares, luego n, m deben tener la misma paridad.

Ejemplo 1.
Determina todas las soluciones enteras de la ecuación
x 2  y 2  108

Solución.
x2  y 2  108  ( x  y)( x  y)  22 33
Y por tanto
 x  y  2 3
a b

 para ciertos 0  a  2 , 0  b  3 .
 x  y  22  a33b
Luego
 2a3b  22  a33 b
 x   2a 13b  21 a33 b
2
 2  a 3b
 y  2 3  2 3  2a 13b  21 a33 b
a b

 2
El único valor de a para el que x, y son enteros es a  1 , y en este caso:

x  y  2  3
b

x  3  3
b 3 b

 

x  y  2  3
3 b

y  3  3
b 3 b

Dando valores a n aparecen las cuatro soluciones posibles con x  0 :


b  0  x  28 , y  26
b  1  x  12 , y  6
b  2  x  12 , y  6
b  3  x  28 , y  26
Las soluciones son ocho:
( x, y)  (  28 ,  26 ) , ( x, y)  (  12 ,  6 )
Ejemplo 2.

Determina todas las soluciones enteras de la ecuación


x2  y 2  600

Solución.
x 2  y 2  600  ( x  y)( x  y)  23  3  52
Y por tanto

x  y  2  3  5
a b c

 para ciertos 0  a  3 , 0  b  1 , 0  c  2
 3 a 1 b
x  y  2  3  5
2c

Luego
 2a  3b  5c  23 a  31b  52  c
 x   2a 1  3b  5c  22  a  31b  52  c
2
 3 a 1 b 2c
 y  2  3  5  2  3  5  2a 1  3b  5c  22  a  31b  52  c
a b c

 2

Luego a  1 o a  2 . En total tenemos las siguientes combinaciones:


a=1 , b=0 ,c=0, x= 151, y= -149 ; a=1 , b=0 ,c=1, x= 35, y= -25
a=1 , b=0 ,c=2, x= 31, y= 19 ; a=1 , b=1 ,c=0, x= 53, y= -47
a=1 , b=1 ,c=1, x= 25, y= 5 ; a=1 , b=1 ,c=2, x= 77, y= 73
a=2 , b=0 ,c=0, x= 77, y= -73 ; a=2 , b=0 ,c=1, x= 25, y= -5
a=2 , b=0 ,c=2, x= 53, y= 47 ; a=2 , b=1 ,c=0, x= 31, y= -19
a=2 , b=1 ,c=1, x= 35, y= 25 ; a=2 , b=1 ,c=2, x= 151, y= 149

Que dan lugar a las siguientes 24 soluciones:


( x, y)  (151,149), (35,25), (31,19), (53,47), (25,5), (77,73)

Ejemplo 3.
Determina todas las soluciones enteras de la ecuación
x2  y 2  294

Solución.
x 2  y 2  294  ( x  y)( x  y)  2  3  72
Y por tanto

x  y  2  3  7
a b c

 para ciertos 0  a  1 , 0  b  1 , 0  c  2
 1 a
x  y  2  3  7
1 b 2c

Luego
 2a  3b  7c  21 a  31b  7 2  c
 x   2a 1  3b  7c  2 a  31b  7 2  c
2
 1 a 1 b 2c
 y  2  3  7  2  3  7  2a 1  3b  7c  2 a  31b  7 2  c
a b c

 2

Y vemos que no existe ningún valor 0  a  1 para el cual x, y sean enteros. Por lo
tanto, esta ecuación no tiene ninguna solución entera.
Ejemplo 4.
Determina todas las soluciones enteras de la ecuación
x 2  y 2  189

Solución.
x 2  y 2  189  ( x  y)( x  y)  33  7

Y por tanto

x  y  3  7
a b

 para ciertos 0  a  3 , 0  b  1
 3 a
x  y  3  7
1 b

Luego
 3a  7b  33 a  71 b
 x 
2
 3 a 1 b
y  3 7  3 7
a b

 2

Las soluciones son:


a=0 , b=0, x= 95, y= -94 ; a=0 , b=1, x= 17, y= -10
a=1 , b=0, x= 33, y= -30 ; a=1 , b=1, x= 15, y= 6
a=2 , b=0, x= 15, y= -6 ; a=2 , b=1, x= 33, y= 30
a=3 , b=0, x= 17, y= 10 ; a=3 , b=1, x= 95, y= 94

Que dan lugar a las siguientes 16 soluciones:


( x, y)  (95,94), (17,10), (33,30), (15,6)

Ejemplo 5.
Determina todas las soluciones enteras de la ecuación
x 2  y 2  441

Solución.
x2  y 2  189  ( x  y)( x  y)  32  72

Y por tanto

x  y  3  7
a b

 para ciertos 0  a  2 , 0  b  2
 3 a
x  y  3  7
1 b

Luego
 3a  7b  32  a  7 2  b
 x 
2
 2a 2 b
y  3 7  3 7
a b

 2
Las soluciones son:
a=0 , b=0, x= 221, y= -220 ; a=0 , b=1, x= 35, y= -28
a=0 , b=2, x= 29, y= 20 ; a=1 , b=0, x= 75, y= -72
a=1 , b=1, x= 21, y= 0 ; a=1 , b=2, x= 75, y= 72
a=2 , b=0, x= 29, y= -20 ; a=2 , b=1, x= 35, y= 28
a=2 , b=2, x= 221, y= 220

En este caso observamos que el número de soluciones con x  0 es impar.


Las soluciones a la ecuación son las siguientes 18:
( x, y)  (221,220), (35,28), (29,20), (75,72), (21,0)
De las cuales son positivas las cinco siguientes:
( x, y)  (221,220), (35,28), (29,20), (75,72), (21,0)

Teorema 1.
El número r de soluciones enteras ( x, y) de la ecuación x 2  y 2  k queda
determinado por su descomposición en factores primos k  p11 p2 2 ... pn n .
a) Si p1  2 y 1  1 , es decir, el número k es de la forma k  2q con q impar, no hay
solución posible: r  0 .

b) Si con p1  2 , es decir, el número k es impar, entonces la fórmula


n

 
i 1
i  1

genera todas las soluciones de la forma ( x, y) , ( x, y) , con x  0 , luego


n
r  2  i  1
i 1

c) Si p1  2 y 1  1 , es decir, el número k es de la forma k  4q , entonces la fórmula


n
(1  1)  i  1
i 2
genera todas las soluciones de la forma ( x, y) , ( x, y) , con x  0 , luego
n
r  2(1  1)  i  1
i2

Demostración.
a) x2  y 2  2q  ( x  y)( x  y)  2q
Vemos que 2 y q son coprimos, y es imposible encontrar factorizaciones 2q  n  m con
la misma paridad.

b) Si k  p11 p2 2 ... pn n con p1  2 , entonces


 p11 p2 2 ... pn n  p11  1 p2 2   2 ... pn n   n
 x 
2 con 0  i  i
 n
1  2
 y  p1 p2 ... pn  p1
 1  1  2   2
p2 ... pn n   n
 2
Y todas las combinaciones posibles son válidas pues los numeradores son siempre
pares, pues son sumas y restas de números impares.

c) Si k  21 p2 2 ... pn n , entonces


 21 p2 2 ... pn n  21  1 p2 2   2 ... pn n   n
 x   21 1 p2 2 ... pn n  21  1 1 p2 2   2 ... pn n   n
2
 n
1  2
 y  2 p2 ... pn  2
 1  1  2   2
p2 ... pn n   n
 21 1 p2 2 ... pn n  21  1 1 p2 2   2 ... pn n   n
 2

con 0  i  i

Pero para 1  0 y 1  1 los valores obtenidos de x, y no serán enteros, luego hay


que descartarlos. Por lo tanto el exponente 1 debe ser tratado de forma diferente.

Observación 1.
Si queremos determinar únicamente las soluciones no negativas, debemos diferenciar
los siguientes casos:
a) Si k no es un cuadrado perfecto, es decir, algún  i es impar y por tanto algún  i  1
es par, el número de soluciones enteras será un número par (la mitad positivas y la
mitad negativas). En particular:
n
a1) Si p1  2 , es decir, k es impar, la fórmula  
i 1
i  1 genera todas las soluciones

( x, y) , ( x, y) con x  0 , luego el número de soluciones positivas será


1 n
s    i  1
2 i 1
a2) Si p1  2 y 1  1 , entonces el número de soluciones positivas será
n
s  (1  1)  i  1
1
2 i 2

b) Si k es un cuadrado perfecto, es decir, todos los  i son pares y por tanto todos los
 i  1 son impares, aparecerá la solución x  k , y  0 :
k  k 0  k 0 
Esta solución hará que el número total de soluciones sea impar.

Por ejemplo, en el Ejemplo 4,


1 n

x 2  y 2  189  33  7  i  1  1 (3  1)(1  1)  8  4
2 i 1 2 2
Que son las cuatro soluciones ya encontradas anteriormente:
( x, y)  (95,94), (17,10), (33,30), (15,6)
Ejemplo 6.
Determina el número de soluciones enteras de la ecuación x 2  y 2  9 . ¿Cuántas de
ellas no son negativas?

Solución.
n
Aplicando la fórmula del teorema anterior, 9  32    i  1  2  1  3
i 1

Luego hay 3  2  6 soluciones enteras, de las cuales 3 / 2  1.5  2 no son negativas.


Concretamente son: x  3, y  0 ; x  5, y  4

Ejemplo 7.
Determina el número de soluciones enteras de la ecuación x 2  y 2  400 . ¿Cuántas de
ellas no son negativas?

Solución.
n
400  24  52  (1  1)  i  1  3  3  9 .
i2

Luego hay 9  2  18 soluciones enteras, de las cuales 9 / 2  4.5  5 no son


negativas.

Observación 2.
En el apartado 20.1 se introduce el número de divisores positivos de k ,  (k ) , y
aplicando la fórmula de 20.4 se deduce que, si k es impar, se cumple:
1
 (k )  r
2
Fuente: “On the Number of Solutions of the Diophantine Equation x2 − y2 = N” , J.F.T. Rabago
INTERNATIONAL JOURNAL OF MATHEMATICS AND SCIENTIFIC COMPUTING
(ISSN: 2231-5330), VOL. 2, NO. 2, 2012 13

10.3.1 D
Dado un número entero positivo n , definimos λ(n) como el número de soluciones
enteras positivas de la ecuación x 2  y 2  n . Diremos que el número n es “olímpico”
si λ(n) = 2021. ¿Cuál es el menor entero positivo que es olímpico? ¿Y cuál es el menor
entero positivo impar que es olímpico?

OME 2021 #2

10.3.2 M
Suponiendo que las raíces de la función f ( x)  x 2  a x  2a son valores enteros,
determina la suma de todos los posibles valores de a .

(A) 7 (B) 8 (C) 16 (D) 17 (E) 18

AMC 10A 2015 #23


10.4 La técnica del descenso infinito de Fermat.
Supongamos que queremos resolver una ecuación diofántica, y encontramos que, dada
una solución x1  0 , entonces también será solución x2  0 , con x1  x2  0 .

Obtenemos así una sucesión infinita y decreciente de soluciones x1  x2  ...  xn  0 , lo


cual es imposible pues trabajamos con números naturales, llegando a contradicción.

Así pues, esta técnica nos permite demostrar que una determinada ecuación diofántica
no tiene solución posible, o solo tiene la solución trivial x  0 .

Una definición más formal de esta técnica sería la siguiente: Supongamos que queremos
demostrar una propiedad P para los enteros. Si la hipótesis de que se cumple para un
cierto número entero positivo n0 implica que también se cumple para otro entero
positivo más pequeño n1  n0 , entonces ningún entero positivo satisface P.

Esta técnica se justifica formalmente en el principio de que todo conjunto finito de


números naturales tiene un mínimo. Sea S el conjunto de números naturales que
satisfacen P. Entonces S tiene un mínimo n , y la existencia de m  n satisfaciendo P,
es decir, m  S nos lleva a contradicción.

Este método se asocia a Fermat (1601-1665) porque este matemático fue el primero en
utilizarlo explícitamente en sus razonamientos. Es intersante reseñar que Fermat, pese a
ser proclamado padre de la Teoría de Números moderna, siempre se consideró a sí
mismo un matemático amateur, y nunca se preocupó de publicar ninguno de sus
resultados matemáticos, contentándose con enviarlos a su amigo el matemático Marin
Mersenne (1588-1648).

Podemos leer en Cut the Knot que este método fue el utilizado por Euclides en Los
Elementos VII.31 para demostrar que todo número compuesto tiene un divisor primo.

10.4.1 F
Determine todas las soluciones enteras de la ecuación
x 2  y 2  3z 2

10.4.2 M
Determine todas las soluciones enteras de la ecuación
a 2  b2  c 2  a 2b2
USAMO 1976 #3
11 Problemas de la tercera parte.
11.1 MF
Sea n un número natural. Probar que si la última cifra de 7 n es 3, la penúltima es 4.

OMEFL 2018 #5

11.2 MF
Demuestra que 22225555  55552222 es múltiplo de 7.

OMEFL #4

11.3 M
Determina el número de valores distintos de la expresión
n2  2
n2  n  2

Donde n  1, 2 , 3,...,100 

OME 2017 #1

11.4 F
Probar que para todo entero positivo n, n19  n7 es divisible por 30.

OMEFL 2009 #4

11.5 D
Probar que hay infinitos números primos cuyo resto al dividirlos entre 3 es 2.

OMEFL 2017 #5

11.6 M
Determinar razonadamente si el número
n  3n2  2n  2

es irracional para todo entero no negativo n .

OME 2012 #1

11.7 M
Los números naturales 22, 23, y 24 tienen la siguiente propiedad: los exponentes de los
factores primos de su descomposición son todos impares:
22  21  111 , 23  231 , 24  23  31

¿Cuál es el mayor número de naturales consecutivos que pueden tener esa propiedad?
Razónese la contestación.
OMEFL 2006 #5
11.8 F
Encontrar todas las soluciones enteras posibles, x e y, de la ecuación:
p x  y   x y
siendo p un cierto número primo.

OMEFL 2007 #2

11.9 F
a) Demostrar que
x2 y2 z2
  1
( x  1) 2 ( y  1) 2 ( z  1) 2

Para todos los números reales x, y, z , todos diferentes de 1 y cumpliendo xyz  1 .

b) Demostrar que la igualdad acontece para infinitas ternas de números racionales


x, y, z , todos diferentes de 1 y cumpliendo xyz  1 .

IMO 2008 #2

Nota: Solo el apartado (b) es un problema de Teoría de Números. El apartado (a) es un problema propio
de Desigualdades.

11.10 MD
Demuestra que para cada entero n existe un entero de n dígitos, todos impares, divisible
por 5n .

USAMO 2003 #1

11.11 D
Demostrar que para cualquier par de enteros positivos k y n , existen k enteros
positivos m1 , m2 , ..., mk (no necesariamente distintos) tales que
2k  1  1  1   1 
1  1   1   ...1  
n  m1   m2   mk 

IMO 2013 #1

11.12 MD
Determine todas las parejas de enteros ( x, y) tales que

1  2 x  22 x 1  y 2

IMO 2006 #4

11.13 D
Sea n un entero positivo y sean a1 ,..., ak ( k  2 ) enteros distintos del conjunto {1, ...,n},
tales que n divide a ai ai 1  1 , para i  1,..., k  1 . Demostrar que n no divide a
ak a1  1 .

IMO 2009 #1
11.14 D
Determinar un número de cinco cifras tal que su cuadrado termine en las mismas cinco
cifras colocadas en el mismo orden.

OME 1984 #2

11.15 D
Determina la suma de todos los enteros positivos n tales que, cuando
13  23  33  ...  n3 se divide entre n  5 , el residuo es 17.

AIME II 2020 #10

11.16 M
Determinar todos los números de cuatro cifras n  abcd tales que al insertar un dígito 0
en cualquier posición se obtiene un múltiplo de 7.

OMEFL Aragón 2021 #3

11.17 F
a) Demuestra que 2n  1 y 2n  1 son coprimos.
b) Demuestra que, si n es un número par, entonces n 2  1 y 3n  1 son primos entre sí.

11.18 D
Determina todos los x, y, z  IN tales que 3x  4 y  5z .

IMO 1991 Shortlist


12 El pequeño Teorema de Fermat.
12.1 Teorema. Pequeño Teorema de Fermat (PTF).
Si p es primo,
a) a p  a (mod p) para cualquier entero a .
b) Si p | a , entonces a p1  1 (mod p)

Nota: El recíproco no es cierto: a n1  1 (mod n) para cierto entero a 


 n primo. Esto
se estudiará detenidamente en una observación más adelante.

Demostración.
a) Aunque en el próximo tema veremos que este teorema es un caso particular de
aplicación de la función Phi de Euler, vamos a presentar aquí una demostración directa.

Caso 1: Si a  0 . a  0  a p  0  a y está claro que entonces a p  a (mod p) .

Caso 2: Si a  0 . Vamos a demostrarlo por inducción en a :


Si a  1 , a p  1p  1 y está claro que entonces a p  a (mod p) .
Supongamos cierto a p  a (mod p) , queremos ver que entonces es cierto para a  1 .
Aplicando el Teorema del Binomio:
a  1p  a p    a p 1  ...  
p p 
 a  1
1  p  1
 p  p p!
Pero p |   para todo 1  k  p (por la definición de    al ser p primo,
k  k  k!( p  k )!
estará en el numerador, pero no en el denominador) , y por tanto
a  1p  a p  1 (mod p) .
Finalmente, aplicando la hipótesis de inducción:
a  1p  a p  1 (mod p)  a  1 (mod p) ,
tal y como queríamos ver.

Caso 3: Si a  0 . Si p  2 , entonces
a 2  (a)2  a (mod 2)  2 | a 2  a  2 | a 2  a  2a  a 2  a  a 2  a (mod 2)
Si p  2 , entonces p es impar, y por tanto: a p  (a) p  (a) (mod p)  a (mod p)
En donde hemos aplicado el “Caso 1” pues  a es positivo.

 
b) Aplicando el apartado anterior, a p  a (mod p)  p | a p  a  a a p 1  1 .
Aplicando el Lema de Euclides, puesto que, por hipótesis, p | a , deducimos que
p | a p 1  1 , o equivalentemente, a p 1  1  0 mod p   a p 1  1 mod p 
El PTF se puede aplicar al cálculo de congruencias con potencias de números, como en
los siguientes ejemplos:

12.2 Ejemplo resuelto.


Demostrar que 538  4 (mod 11) .

Solución.
Aplicamos el PTF para garantizar que 510  1 (mod 11) , luego:
  3
538  5310  8  510 58  1358 (mod11)  58 (mod11)  (*)

Por otro lado 52  25  3 (mod11) , y por tanto:


  4
(*)  52 (mod11)  34 (mod11)  81 (mod11)  4 (mod11)

Donde hemos aplicado que 81  7  11  4

12.3 Ejemplo resuelto.


Calcular 7121 (mod 13)

Solución.
Puesto que 13 | 7 podemos aplicar el PTF para garantizar que 712  1 (mod13) .
 
Puesto que 121  12  10  1 7121  712101  712
10
 7  110  7  7 (mod13)

El PTF se puede utilizar también para testear si un número n es primo o no. Si


encontramos un entero a tal que a n  a (mod n) , entonces seguro que n no es primo
(pues en caso contrario contradeciría el PFT). Veamos esto en el siguiente ejemplo:

12.4 Ejemplo resuelto.


Demostrar que 117 no es primo.

Solución.
Tomamos a  2 . Sabemos que 27  128  11 (mod117) , y que 121  4 (mod117) ,
luego:
2 
7 16
  8
 8
 1116  112  1218  48  22  216 (mod117)
2117  2716 5 2  2  2 2 (mod117)  2 (mod117)
7 16 5 16 5 21

 2   11  121  11  4  11  44 (mod117)
3
Pero 221 7 3

Con lo que, finalmente llegamos a 2117  44  2 (mod117) , y por tanto 117 debe ser un
número compuesto (de hecho: 117  13  9 ).
12.5 Proposición. Descomposición de módulos.
Si p y q son primos diferentes tales que a p  a (mod q) y a q  a (mod p) , entonces:
a p q  a (mod p q)

a 
q p
 a q (mod p) por el Corolario al PTF, y a q  a (mod p) por hipótesis, luego:
Luego a p q  a q p
 a q (mod p)  a (mod p) , es decir: p | a p q  a

Con un razonamiento similar llegamos a q | a p q  a , y por tanto:


p q | a p q  a , o equivalentemente: a p q  a  0 (mod p q)

12.6 Ejemplo resuelto.


Demostrar que 2340  1 (mod 341) .

Solución.
Aquí 341  11  31 , y por otra parte, 210  1024  31  33  1, luego 210  1 mod(31) , y por
tanto:
211  2  210  2  1  2 (mod 31)

Por otro lado, 210  1024  11  93  1  210  1 (mod11) , y por tanto:


  3
  3
231  2 210  2 210  2  13  2 (mod11)

Y aplicando la proposición anterior: 2341  21131  2 (mod 341) , y cancelando un factor 2


llegamos al resultado deseado: 2340  1 (mod 341)

12.7 Observación. Los números pseudoprimos y el recíproco del PTF.


El PTF dice que n primo  2n  2 (mod n) , y acabamos de ver que 2341  2 (mod 341)
y sin embargo 341  11  31 no es primo, es decir un contraejemplo para el recíproco del
PTF.

Este dato es curioso porque n  341 es el primer compuesto tal que 2n  2 (mod n) , es
decir, sirve como contraejemplo del recíproco del PTF, es decir:

n primo  2n  2 (mod n) para 2  n  340

Esta es la razón por la que los matemáticos chinos antiguos creyeron, equivocadamente,
que 2n  2 (mod n) caracterizaba los números primos.

Los números n tales que 2n  2 (mod n) , es decir n | 2n  2 se denominan


pseudoprimos. Hay infinitos pseudoprimos y los más pequeños son 341, 561, 645 y
1105.
Aplicación del PTF a la resolución de congruencias no lineales.

12.8 Ejemplo resuelto.


Determina una solución de la congruencia x103  4 (mod 11)

Solución.
Puesto que, aplicando el PTF, x10  1 (mod11)
  3
x103  x1010 3  x10 x3  x3 (mod11)
Luego hemos reducido nuestro problema a resolver la congruencia x3  x (mod11)

Probando valores x  1 , x  2 , x  3 ,... llegamos a 53  4 (mod11) , y por tanto:


x  5 (mod11)

12.9 Ejemplo resuelto.


Determina una solución de la congruencia x86  6 (mod 29) .

Solución.
Aplicando el PTF, sabemos que x 28  1 (mod 29) para todo x .
 3
Luego x86  x328 2  x 28 x 2  x 2 (mod 29) , luego hemos reducido nuestro problema a
resolver la congruencia x 2  6 (mod 29) .
Vamos probando valores x  1 , x  2 , x  3 ,... hasta llegar a
x  8  x  64  29  2  6  6 (mod 29)
Luego una solución es x  8 (mod 29)

Nota: Existe otra solución: x  21  x 2  441  29  15  6  6 (mod 29)


Para encontrar estas soluciones se puede hacer el siguiente planteamiento:
6  64 (mod 29) , y por tanto, la ecuación x 2  6 (mod 29) es equivalente a
x 2  64 (mod 29) ,
y ahora:
x  8
x 2  64 (mod 29)  x 2  64  0 (mod 29)  x  8x  8  0 (mod 29)  
 x  8
y finalmente:  8  21 (mod 29)

12.10 F
Aplicando el PTF, determina:
a) 331 (mod 7) b) 235 (mod 7) c) 128129 (mod 17)

12.11 F
Dividimos el número 21000 entre 13. ¿Cuál es el residuo?

AHSME 1972 #31


12.12 F
Utilizando el PTF, demuestra que 17 divide a 11104  1

12.13 F
Demuestra que si (a,35)  1, entonces a12  1 (mod 35)

12.14 F
Sea a1  4 , an  4an1 , n  1 . Determina el residuo cuando a100 se divide entre 7.

12.15 F
Demuestra que, si (a,42)  1, entonces 168  3  7  8 | a6  1 .

12.16 F
Determina 220  330  440  550  660 (mod 7)

12.17 D
En los años 60, tres matemáticos americanos demostraron que una de las conjeturas de
Euler era falsa al encontrar un entero positivo n tal que

1335  1105  845  275  n5

Determina n.

AIME 1989 #9

Nota: Se presentan dos soluciones, pero ninguna de las dos es completa: Son argumentos que justifican
que un cierto n es el mejor candidato posible.

12.18 F
Determina los números primos p para los cuales 29 p  1 es múltiplo de p.

12.19 D
Determine todos los enteros positivos coprimos con todos los términos de la siguiente
sucesión:
an  2n  3n  6n  1 , n  1

IMO 2005 #4
13 La función Phi de Euler.
13.1 Definición. La función Phi de Euler.
Dado un número natural n  1 , la función Phi de Euler,  (n) , es la que indica el
número de números naturales menores que n y coprimos con n.

Por ejemplo,  (30)  8 porque el número de naturales coprimos con 30 son 1, 7, 11, 13,
17, 19, 23 y 29. De la misma manera vemos que
 (1)  1,  (2)  1,  (3)  2,  (4)  2,  (5)  4,
 (6)  2,  (7)  6...
Observamos que  (1)  1 porque mcd (1,1)  1 , sin embargo, si n  1 , mcd (n, n)  n  1 .
Observamos p es primo si y solo si  ( p)  n  1 .

Con Mathematica: "EulerPhi[n]"

13.2 Proposición.La función Phi de Euler mediante el TFA.


Si la descomposición en factores primos de n es n  p1k1 p2k 2 ... prk r , entonces:
 1
 (n)   p1k  p1k 1  p2k  p2k ... p 
r
1 1 2 2 1 kr
r  prk r 1  n   1  
i 1  pi 

Ejemplos:
a)  (360)  96 porque
 1  1  1 
360  23325   (360)  3601  1  1    96
 2  3  5 
b)  (1001)  720 porque
 1  1  1
1001  7  11  13   (1001)  10011  1  1    720
 7  11  13 
13.3 Propiedad de la función Phi de Euler.
Fijemos un número n y un divisor suyo: Por ejemplo, n  30 y d  5 .
Estudiemos el conjunto Sd   m  n , (m, n)  d . En la siguiente tabla vemos que
S5   5 , 20 

1 m  n (m,30) (m,30)  5  (6)  2


1 1
2 2
3 3
4 2
5 5 5  5 1 1
6 6
7 1
8 2
9 3
10 10 10  5  2 2
11 1
12 6
13 1
14 2
15 15 15  5  3 3
16 2
17 1
18 6
19 1
20 10 20  5  4 4
21 3
22 2
23 1
24 6
25 5 5  55 5
26 2
27 3
28 2
29 1
30 30 30  5  6 6

Consideremos ahora n / d  6 y  (6) . El conjunto de números menores que 6 y


coprimos con 6 es  1 , 5 , luego  (6)  2 .
Vemos que existe una biyección perfecta entre S5 y los coprimos de 6, y por tanto
S d   (n / d )
Por otro lado, todo número m  n pertenece a algún S d con d | n , y por tanto:
n   S d   ( n / d )
d |n d |n

d
Existe también una biyección d | n  | n y por tanto llegamos a la siguiente
n
propiedad:
n    (d )
d |n
13.4 Teorema. Teorema de Euler.
a ( n )  1 (mod n) si (a, n)  1

Observación: La condición (a, n)  1 es necesaria, pues si (a, n)  1 , ya vimos en el


capítulo 8 que la congruencia a x  1 (mod n) no tiene solución, y por tanto no puede
existir ningún k tal que a k  1 (mod n) , pues en ese caso x  a k 1 sería solución de la
congruencia a x  1 (mod n) .

Nota histórica. La primera demostración del PTF , a p 1  1 (mod p) si p | a , fue dada


por Euler en 1736. El propio Euler presentó en 1760 este teorema, que es una
generalización del PTF porque si n es primo entonces  (n)  p  1 , y (a, p)  1  p | a .

13.5 Ejemplo.
Tomando n  30 y a  11 ,  (30)  8 y 118  1 (mod 30)
En efecto, 112  121  1 (mod 30)  118  112  4
 14  1 (mod 30)

13.6 Definición. Orden de un entero.


El Teorema de Euler indica que si (a, n)  1 , la secuencia
 
a , a 2 , a3 , a 4 ,...
siempre alcanza el 1 (y por lo tanto se vuelve periódica), y lo alcanza en a (n ) .
Naturalmente,  (n) no es necesariamente el primer número k para el cual a k  1 . El
menor exponente k para el cual a k  1 se denomina orden de a (módulo n) y se
estudiará en el apartado siguiente.

El Teorema de Euler y el PTF nos permiten reducir potencias muy grandes módulo n.

13.7 Problema resuelto.


Determina los dos últimos dígitos de la expresión decimal de 3256 .

Solución.
Está claro que este problema implica estudiar 3256 (mod100) , y aquí nos puede ayudar
el Teorema de Euler:
   1  1 
 (100)   2252  1001  1    40 , y puesto que (3,100)  1 , podemos aplicar el
 2  5 
Teorema de Euler: 3  1 (mod100)
40

  6
Luego 3256  340616 (mod100)  340 316 (mod100)  316 (mod100) .
38  6561  61 mod(100)  316  3838  61  61  3721  21 mod(100)
Por lo tanto, el número 3256 acaba en "21".
13.8 F
¿Para cuántos enteros i , cumpliendo 1  i  1000 , existe un entero j , cumpliendo
1  j  1000 , tal que i es un divisor de 2 j  1 ?

13.9 M
Determina los ocho últimos dígitos de la expansión binaria de 271986 .

13.10 D
1
... 2
20072006
Determina los tres últimos dígitos de 2008
1
... 2
20072006
Nota: 2008 se define recursivamente:
a1  1 , a2  2a1 , a3  3a2 , ... , a2008  2008a2007

PuMAC (Princeton University Mathematics Competition)

13.11 D
xx
Definimos f ( x)  x x . Determina los dos últimos dígitos de
f (17)  f (18)  f (19)  f (20)

PuMAC 2008

Nota: f (x) se define recursivamente: f1 ( x)  x , f 2 ( x)  x f1 ( x ) ,..., f ( x)  f 4 ( x)  x f3 ( x ) .

Indicación: Si (a, n)  1 , entonces ab  ab mod ( n) (mod n) y podemos reducir el cálculo


a determinar b mod  (n) .

13.12 F
Encontrar las tres últimas cifras del número 7 2014

OMEFL 2014 #2
14 Orden de un entero.
14.1 Definición. Orden de un entero.
El concepto de orden de un entero se introdujo en 13.6:
Si a es cualquier número entero y n es un número entero n  1 , (a, n)  1 , definimos el
orden de a módulo n, ordn (a) , como el entero positivo más pequeño d tal que
a d  1 (mod n)

Ejemplos:
31  3, 32  2 , 33  6 , 34  4 , 35  5 , 36  1 (mod 7)  ord7 (3)  6 .
ord5 (2)  4 pues 21  2 , 22  4 , 23  3 , 24  16  1 (mod 5) .
ord12 (5)  2 pues 51  5 , 52  25  2  12  1  1 (mod12) .

Con Mathematica:

14.2 Observación.
No todos los números tienen un definido un orden. Por ejemplo, si n | a , entonces
a k  0 (mod n) para todo k .

La siguiente proposición nos indica cuando un entero tendrá un orden asociado.

14.3 Proposición.
Si (a, n)  1 , entonces existe un k  n tal que a k  1 (mod n) .

Demostración.
 
Consideremos el conjunto a, a 2 , a3 , ..., a n1 mod n . Consta de n  1 números y solo
existen n residuos mod n , luego por el Principio de las Casillas, dos de estas potencias
deben ser iguales módulo n. Es decir, existirán 1  s  t  n  1 tales que
a s  at (mod n) .
Ahora, 1  t  s  n , y por tanto:
a s  at (mod n)  at s a s  at s at (mod n)  at  at s at (mod n)
Puesto que (a, n)  1  (at , n)  1 , podemos cancelar el factor a t , con lo que llegamos a
1  a ts (mod n) , tal y como queríamos ver.
14.4 Corolario.
Dado un entero n  1 , a  Z tendrá orden módulo n si y solo si (a, n)  1 .

Demostración. Si (a, n)  1 , por la proposición anterior existirá un 1  k  n tal que


a k  1 (mod n) , luego por el Principio de la buena ordenación, existirá un mínimo k
cumpliendo tal condición, es decir, el número a tendrá un orden asociado.

Supongamos ahora que existe un entero positivo m tal que a m  1 (mod n) . Entonces
existirá un entero s tal que
a m  sn  1  a  a m 1  sn  1

Esta última expresión es una combinación lineal de a y n , luego


(a, n) | a  a m 1  sn  1  (a, n)  1

14.5 Teorema.
Si (a, n)  1 , entonces, para cualquier entero t, at  1 (mod n)  ordn (a) | t .

14.6 Problema resuelto.


Determina todos los enteros positivos n tales que 2n  1 sea divisible entre 7.

IMO 1967 #1 (apartado a)

Solución: 7 | 2n  1  2n  1  0 (mod 7)  2n  1 (mod 7)


Puesto que (2,7)  1 , podemos aplicar el Teorema anterior:
2n  1 (mod 7)  ord7 (2) | n
21  2 , 22  4 , 23  8  1 (mod 7)  ord7 (2)  3
Luego, finalmente, 7 | 2n  1  3 | n , es decir, para todos los múltiplos de 3.

14.7 Corolario.
Dados p primo y a entero con p | a , entonces ordn (a) | p  1

Demostración. Puesto que p es primo y p | a , podemos aplicar el PTF para garantizar


que a p 1  1 (mod p) , y aplicar ahora el Teorema anterior.

14.8 Ejemplo.
Determinar ord11(8) .

Solución.
Puesto que 11 | 8 , aplicar el corolario anterior para asegurar que ord11(8) | 10 .
ord11(8)  2 no puede ser pues 82  64  9  1 (mod11) .
ord11(8)  5 tampoco puede ser:
82  9 (mod11) y 83  512  6 (mod11) , luego
85  8283  9  6  54 (mod11)  10 (mod11)
Luego solo nos queda ord11(5)  10 .
14.9 Proposición. Propiedad de la secuencia de potencias de un número.
Dados a y n cumpliendo (a, n)  1 , sea k  ordn (a) .
 
En la secuencia a , a 2 , a3 , a 4 ,..., a k  1 no hay números repetidos (módulo n), pues si
ai  a j con 1  i  j  k , entonces podemos cancelar términos pues (a, n)  1 , y deducir
que a j i  1 (mod n) , con j  i  k contradiciendo la definición de orden.

En general:
a j  1 (mod n)  ordn (a) | j

En efecto, por el Algoritmo de la división, j  k  q  r con 0  r  k , pero entonces:


  q
a j  a kq  r  a kq a r  a k a r  1q a r  a r (mod n) . Si 0  r  k se contradeciría la
definición de k como índice de a, por lo tanto r  0 y j es un múltiplo del orden.

En particular, podemos aplicar este hecho a la función Phi de Euler: ordn (a) |  (n)

14.10 Ejemplo resuelto.


Determinar ord13 (2)

Solución.  (13)  12  ord13(2)  1, 2 , 3, 4 , 6 ,12 


21  2 , 22  4 , 23  8 , 24  16  3 (mod 13) , 26  32  12 (mod 13) , luego solo puede
ser ord13 (2)  12 .

14.11 Problema resuelto.


Determina el menor factor primo impar de 20198  1

AIME I 2019 #14

Solución.
Buscamos el menor número primo p tal que p | 20198  1
p | 20198  1  20198  1  0 (mod p)  20198  1 (mod p)
Entonces, elevando al cuadrado ambos lados, 201916  1 (mod p)
Pero 201916  1 (mod p)  ord p (2019)  1, 2 , 4 , 8 ,16 
Sin embargo, ord p (2019)  1, 2 , 4 , 8   20198  1 (mod p) y no -1, como queríamos,
luego deducimos que ord p (2019)  16 .
Puesto que ord p (2019) |  (p) ,  (p) será un múltiplo de 16.
 1
Puesto que por hipótesis p es primo,  ( p)  p  1    p  1
 p
Y por tanto p  1 (mod16) . Los dos primeros primos que cumplen p  1 (mod16) son
17 y 97.
Sin embargo, 20198  1 (mod17) , pero 20198  1 (mod 97) , luego la solución es 97.

Fuente de esta solución: artofproblemsolving.com


14.12 Problema resuelto.
Determina el menor entero positivo n tal que, cuando ese escribe 3n en base 143, sus
últimas dos cifras en dicha base son 01.

AIME I 2018 #11

Sabemos que decir que la expresión de 3n en base 143 es ...d3d 2 d1 01143 es equivalente a

3n  1  0  143  d1  1432  d2  1433  ...  1  1432 d1  d2  143  ...  3n  1 mod 1432 
Así pues, en este problema nos piden resolver la congruencia 3n  1 mod 1432 con  
solución n mínima, es decir, determinar el órden de 3 módulo 1432 .

Es importante remarcar que nos piden n mínimo, pues el Teorema de Euler (13.4) nos
   
garantiza que 3 (143 )  1 mod 1432 puesto que 3,1432  1 , pero en este caso
2

 
 1432  17160 y veremos que no es el valor mínimo posible.

 
143  11  13  1432  112  132 , y puesto que 112 ,132  1 , para resolver la congruencia
  
3n  1 mod112  132 será suficiente resolver las congruencias 3a  1 mod112 y 
 
3b  1 mod132 por separado.

Paso 1. Resolvemos la congruencia: 3a  1 mod112 


Esta es sencilla. Se puede encontrar por tanteo:
31  3  3 mod121 32  9  9 mod121
33  27  27 mod121 34  81  81 mod121
35  243  2  121  1  1 mod121

El valor mínimo es a  5 .

 
Paso 2. Resolvemos la congruencia: 3b  1 mod132 . Esta es mucho más complicada.

Paso 2.1. Por tanteo ("bash"). Si lo hacemos por tanteo nos vamos a pasar un buen rato
calculando postencias hasta llegar al exponente 39 para encontrar 339  1 mod132 : 
1→3 2→9 3→27 4→81 5→74
6→53 7→159 8→139 9→79 10→68
11→35 12→105 13→146 14→100 15→131
16→55 17→165 18→157 19→133 20→61
21→14 22→42 23→126 24→40 25→120
26→22 27→66 28→29 29→87 30→92
31→107 32→152 33→118 34→16 35→48
36→144 37→94 38→113 39→1
Paso 2.2. Aplicando el Teorema de Euler. Una indicación nos la puede dar la función
 
Phi de Euler:  132  2  3  13 , luego el orden de 3 módulo 156 será un divisor de
156  2  3  13 . Probando divisores encontramos la solución 39  3  13 .

Paso 2.3. Aplicando el Teorema del binomio.


 
Observamos que 3b  1 mod132  3b  1 mod13 .
 
En efecto: 3b  1 mod132  3b  132 k  1  1313k   1  3b  1 mod13

Y la congruencia 3b  1 mod13 tiene fácil solución: 33  27  2  13  1  1 mod13 .


Así pues, cualquier solución de 3b  1 mod13 será múltiple de 3.
Llegados a este punto podríamos aplicar la estrategia 2.2 anterior para llegar a la
solución, pero en su lugar vamos a aplicar el desarrollo binomial.

El número b buscado será múltiplo de 3: b  3c para cierto entero c , con lo que la


congruencia se transforma en:

   c

33c  1 mod132  33  1 mod132  27c  1 mod132   
Ahora aplicamos el desarrollo binomial:
27  2  13  1  27 c  2  13  1 
c

c c  c  c


  2  13 10   2  13 11  ...   2  1311c 1   2  1301c
c c 1

0 1  c  1 c

Y observamos que, trabajando módulo 132 , cualquier potencia 13c con c  2 será
(congruente con) cero.

Así pues, a todos los efectos prácticos:


 c  c
27c   2  13 1c 1   2  13 1c  c  26  1
1 0

 c  1 c


Y por tanto, la congruencia exponencial 27c  1 mod132 se convierte en la 
congruencia lineal
26c  1  1 mod132  
Que se resuelve fácilmente:

  
26c  1  1 mod132  26c  0 mod132  2  13c  0 mod132   
Para que esta última congruencia se cumpla, es necesario y suficiente que c sea
múltiplo de 13.

Así pues, finalmente llegamos a un resultado múltiplo de 3 y múltiplo de 13, y el valor


mínimo posible es c  3  13  39 .
Paso 3. Juntamos las dos congruencias:

   
Hemos obtenido 35  1 mod 112 y 339  1 mod 132 , luego tomando el mínimo común
múltiplo de ambos exponentes: 5,39  5  39  195 , tendremos, aplicando 7.5d:
3195  3539  35 39
 
 139  1 mod 112 

3195  3395  3 
39 5
  2 

 1  1 mod 13   3195  1 mod 112  132
5

11 ,13   1
2 2 


Que será el valor mínimo posible pues en todos los pasos hemos obtenido los valores
mínimos posibles.

La solución del problema es 159.

Fuentes:
https://artofproblemsolving.com/wiki/index.php/2018_AIME_I_Problems/Problem_11

https://youtu.be/e7JGgykuK3w Analyzing the Expression in Mod 143² (2018 AIME I Prob 11)
"LetsSolveMathProblems"

https://youtu.be/b_Z_OGfyJyw 2018 AIME I Problem #11 "Osman Nal"


15 Congruencias exponenciales.
Definición. Raíz primitiva.
Dado un entero m , diremos que el entero g es una raíz primitiva módulo m si, para
cualquier entero x coprimo con m , siempre existe un entero j tal que x  g j mod m .

Toda raíz primitiva módulo m es siempre coprima con m . En efecto, si g es una raíz
primitiva módulo m , entonces existe un entero n tal que g n  1 mod m , pero
entonces todo divisor común de g y m deberá dividir a 1, es decir, g y m son
comprimos.

Lema.
Sea m un entero positivo y sea x un entero coprimo con m . Para todo par j, k de
enteros positivos, se cumple:

x j  xk mod m  j  k mod d  , donde d  ordm x 

Ejercicio modelo 1.
Resuelve la siguiente congruencia:
17 x  10 mod 27

Solución.
En primer lugar, vemos que, puesto que 27  33 , existirá una raíz primitiva módulo 27.
De hecho, se demuestra que 2 es una raíz primitiva módulo 27.
Para aplicar el lema anterior, debemos escribir 17 y 10 como potencias de la misma
base.
21  2 mod 27 2 2  4 mod 27 23  8 mod 27
24  16 mod 27 25  32  5 mod 27 26  10 mod 27
27  20 mod 27 28  13 mod 27 29  26  1 mod 27
210  2  25 mod 27 211  4  23 mod 27 212  8  19 mod 27
213  11 mod 27 214  22 mod 27 215  44  17 mod 27
 
Luego 17 x  10 mod 27  215  26 mod 27  215x  26 mod 27
x

Y ahora aplicamos el lema anterior:


 
215x  26 mod 27  15x  6 mod 18 , pues  (27)   33  18
Con lo que hemos reducido una congruencia exponencial a una congruencia lineal.
15x  6 mod 18  5x  2 mod 6 dividiendo entre tres toda la congruencia.
Esta última congruencia la podemos resolver viendo que 51  5 mod 6 y por tanto
5x  2 mod 6  x  51  2  5  2  10  4 mod 6
Puesto que nosotros buscamos soluciones módulo 16, el conjunto total de soluciones
será
4 , 4  6 , 4  12 mod 18  4 , 10 , 16 mod 18
Ejercicio modelo 2.
Resuelve la siguiente congruencia:
9x  10 mod 13

Solución.
En primer lugar, vemos que 2 es una raíz primitiva módulo 13.

Para poder aplicar el lema anterior, debemos escribir 9 y 10 como potencias de 2:


21  2 mod 13 2 2  4 mod 13 23  8 mod 13
24  16  3 mod 13 25  6 mod 13 26  12  1 mod 13
27  2  11 mod 13 28  4  9 mod 13 29  18  5 mod 13
210  10 mod 13

 
Luego 9x  10 mod 13  28  210 mod 13  28 x  210 mod 13
x

y aplicando el lema anterior:


28 x  210 mod 13  8x  10 mod 12 , pues  (13)  12

Pero vemos que esta congruencia lineal no tiene solución (8,12)  4 y 4 | 10 .

Así pues, la congruencia del enunciado no tiene solución.

Ejercicio modelo 3.
Resuelve la siguiente congruencia:
11x  17 mod 18

Solución.
Sabemos que 5 es una raíz primitiva módulo 18.
Para poder aplicar el lema anterior, debemos escribir 11 y 17 como potencias de 5:
51  5 mod 18 52  25  7 mod 18 53  35  1  17 mod 18
54  5  13 mod 18 55  25  11 mod 18

Luego
 
11x  17 mod 18  55  53 mod 18  55 x  53 mod 18
x

y aplicando el lema anterior:


55 x  53 mod 18  5x  3 mod 6 , pues  (18)  6

Para resolver esta congruencia lineal aprovechamos que 5 y 6 son coprimos, luego 5
tendrá inverso módulo 6, en efecto, es el propio 5, luego
5x  3 mod 6  x  51  3  5  3  15  3 mod 6
16 Problemas de la tercera parte.

16.1 MD
Demuestra que existen infinitas parejas a, b  de números enteros positivos distintos y
relativamente coprimos a  1 y b  1 tales que ab  ba es divisible entre a  b .

USAMO 2017 #1
17 Números factoriales.
17.1 Definición. Función suelo, o parte entera.
Definimos la parte entera de un número real x , y la denotaremos por x  , como el
mayor entero n menor o igual que x . Por ejemplo, 2.4  2 , 1.99  1 ,  3.15  4

El Tema 9 de PA está dedicado íntegramente a la función parte entera. Se


recomienda practicar con ella realizando los problemas que allí se proponen.

17.2 M
Calcula la suma
 20   21   22   21000 

3 3 3   ...   3 
       

All Russian MO 2000

17.3 F
¿Cuál es la probabilidad de que un entero del conjunto  1 , 2 , 3 , ... , 100  pueda ser
divisible entre 2 y no divisible entre 3?

AMC10A 2003 #15

Orden p-ádico de un número. La fórmula de Polignac.


17.4 Definición. Orden p-ádico de un número.
Dado un número entero n , su valuación p-ádica o orden p-ádico es la potencia
máxima de p que divide a n :
v p (n)  k  p k | n y p k 1 | n
o equivalentemente:

v p (n)  max k , p k | n 
o equivalentemente:
Si n  p1a1 p2a2 ... piai ... pnan es la descomposición canónica de n , entonces
v pi (n)  ai .

En Mathematica: IntegerExponent[n,p]

Antes de presentar la fórmula de Polignac, propondremos un par de ejemplos para


entender por qué aparecen los elementos que la constituyen.
17.5 Problema resuelto.
¿Cuál será el exponente del factor 5 en la factorización en números primos de 1005! ?

Solución.
1005 ! 1  2  3...  1004  1005

De todos los factores de la derecha, uno de cada cinco es múltiplo de 5, luego habrán
1005 5
0 201
201 múltiplos de 5, que contribuirán en una unidad al exponente de 5.

De los 201 anteriores, uno de cada cinco será múltiplo de 25, luego habrán
201 5
1 40
40 múltiplos de 25, que contribuirán con una unidad más al exponente de 5.

De los 40 anteriores, uno de cada cinco será múltiplo de 53 , luego habrán


40 5
0 8
3
8 múltiplos de 5 , que contribuirán con una unidad más al exponente de 5.

De los 8 anteriores, uno de cada cinco será múltiplo de 54 , luego habrá


8 5
3 1
4
un único múltiplo de 5 , que contribuirá con una unidad más al exponente de 5.

Está claro que no existirán múltiplos de 55 ,56 ,...


Luego el exponente de 5 será 201  40  8  1  250

17.6 Teorema. La fórmula de Polignac.


Estos mismos cálculos los podríamos haber realizado con cualquier número primo y con
cualquier factorial. Para expresarlos en una única fórmula con el lenguaje algebraico
observamos lo siguiente:
1005 / 5  201, 1005 / 5   40 , 1005 / 5   8 , 1005 / 5   1
2 3 4

Luego la suma anterior se puede expresar mediante la función “parte entera”,


justificando la denominada fórmula de Polignac:

La mayor potencia de un número primo p que divide a n! , es decir, el orden p-adico de


n! viene dado por

n
v p n!    k 
k 1  p 

Esta fórmula también es conocida como "fórmula de Legendre" (1806).


Observación: Esta serie es en realidad una suma finita, pues para k suficientemente
n
altos se cumple n  p k y por tanto  i   0 para todo i  k .
p 
17.7 Ejemplo resuelto.
¿Cuál será el exponente del factor 2 en la factorización en números primos de 1005! ?

Solución.
En 1005! uno de cada dos factores es par, luego
1005 2
1 502

hay 502 números pares, de los cuales, uno de cada dos será múltiplo de 2 2 :
502 2
0 251

De estos 251, la mitad serán múltiplos de 23 :


251 2
1 125

De estos 125, la mitad serán múltiplos de 24 :


125 2
1 62

De estos 62, la mitad serán múltiplos de 25 :


62 2
0 31

De estos 31, la mitad serán múltiplos de 26 :


31 2
1 15

De estos 15, la mitad serán múltiplos de 27 :


15 2
1 7

De estos 7, la mitad serán múltiplos de 28 :


7 2
1 3

De estos 3, la mitad serán múltiplos de 29 :


3 2
1 1

Y ya no hay múltiplos de 210 , 211 , 212...

El exponente de 2 en la factorización será 502+251+125+62+31+15+7+3+1=997


17.8 Problema resuelto.
Determina el mayor n tal que 10n divide a 1005 !

AHSME 1977

Solución.
Puesto que 10  2  5  10n  2n 5n , el número n quedará determinado por el número de
parejas 2  5 que podemos hacer en la descomposición factorial de 1005! , es decir, el
mínimo entre a y c
donde 2a  3b  5c...  1005! es la descomposición factorial de 1005!

Acabamos de ver que a  997 y c  250 , luego el resultado será min 250,997  250 .

O, escrito en la forma de la fórmula de Polignac:



1005 
El exponente de 2 será   k   502  251  125  62  31  15  7  3  1  997
k 1  2 

1005 
El exponente de 5 será   k   201  40  8  1  250
k 1  5 

17.9 Problema resuelto.


¿Cuántos ceros hay a la derecha de la expresión decimal de 300 ! ?

Solución.
El número de ceros vendrá dado por el número de veces que 10 divide a 300! .
Puesto que hay más factores de 2 que de 5 en 300! , el número de ceros vendrá dado la
mayor potencia de 5 en 300!, y aquí aplicamos la fórmula de Polignac:

 300   300   300   300 


 51   60 ,  52   12 ,  53   2 , y si k  4  5k   0 .


 300 
Por lo tanto,   5
k 1
k 

 60  12  2  74 .

17.10 M
1000 
Demostrar que 7 no divide a   .
 500 

17.11 F
 200 
¿Cuál es el mayor factor primo de dos dígitos del entero n    ?
 100 
AIME 1983 #8
17.12 MD
Determina todos los pares (k , n) de enteros positivos tales que

   
k! 2n  1 2n  2 2n  4 ... 2n  2n 1 
IMO 2019 #4

17.13 MF
Determina el mayor entero positivo x tal que 236  x divide 2000 !

17.14 Corolario de 17.6.


n 1
a) Si n  p k , con p primo, v p n ! 
p 1

En particular:
p 1
b) Si n  p , con p primo, v p  p !   1.
p 1
p2 1
 
c) Si n  p 2 , con p primo, v p p 2 ! 
p 1
 p  1.

Demostración.
n  n   n   n  pk 1 n 1
v p n !      2   ...   k 1    k   p k 1  p k 2  ...  p  1  
 p  p  p  p  p 1 p 1

17.14 MD
Determina el número de enteros n entre 1 y 50, inclusive, tales que
n2 1 !  
n!n
es un entero (Recuerda que 0! 1 ).

(A) 31 (B) 32 (C) 33 (D) 34 (E) 35

AMC 12A 2019 #24


18 Números combinatorios.

El Tema 4 de PC está dedicado íntegramente a los números combinatorios. Se


recomienda practicar con ellos realizando los problemas que allí se proponen.

18.1
Demuestra que la expresión
(m, n)  n 
 
n  m 
es un entero para todo par de números enteros n  m  1 .

Putnam 2000

18.2 Proposición.
 p
a) Si p es primo, p divide   para todo 0  k  p .
k
b) Si p es primo, p | 2  2 .
p

Demostración.
 p p!  p p!
a)     k!    p( p  1)( p  2)...( p  k  1)
 k  k!( p  k )!  k  ( p  k )!

 p
Está claro que p | p( p  1)( p  2)...( p  k  1) , luego p | k! 
k
 p
Puesto que k  p , es imposible que p | k! , luego p |  
k

 p  p  p
b) 2 p        ...   
0 1  p
 p  p  p  p 
Y puesto que       1 , tenemos 2 p  2     ...   
 0   p 1  p  1
 p
Aplicando el apartado anterior, p |   para todo 0  k  p , luego p | 2 p  2 .
k
Números de Catalan.

18.3 Definición. Número de Catalan de orden n.


Definimos el número de Catalan de orden n como
1  2n 
Cn   
n  1  n 

Los primeros diez números de Catalan son 1, 2, 5, 14, 42, 132, 429, 1430, 4862, 16796.

18.4 Proposición.
Todos los números de Catalan son enteros.

Demostración.
2n  1  2n   2n  1
Vamos a aplicar la identidad    
n  1  n   n  1 
Puesto que 2n  1 y n  1 son coprimos, y la parte de la derecha de la identidad anterior
 2n 
es un entero, está claro que n  1 divide a   .
n
19 Números y primos de Fermat y de Mersenne.
Números y primos de Fermat.
19.1 Definición. Números de Fermat. Primos de Fermat.
Queremos estudiar los números primos de la forma 2n  1 . Ya vimos en el Tema 4 que
si es primo entonces n debe ser una potencia de 2.

Definimos el n-ésimo número de Fermat como Fn  22  1 , n  0 .


n

F0 = 3, F1 = 5, F2 = 17, F3 = 257, F4 = 65537, F5 = 4294967297

El propio Fermat conjeturó que Fn era primo para todo n, sin embargo Euler demostró
que 641 | F5 . El razonamiento se encuentra en el Tema 4.

Así pues, no todo número de Fermat es primo. Llamaremos “primos de Fermat” a


aquellos números de Fermat que sean primos. Es un problema abierto la existencia o no
de infinitos primos de Fermat. Se conjetura que Fn es compuesto para n  5 .

19.2 Proposición.
Los números de Fermat son todos coprimos entre ellos:  Fn , Fm   1 si n  m .

Demostración.
Supongamos que m  n , y sea d un divisor común de Fn y Fm . En primer lugar vemos
que d debe ser impar, pues lo son todos los números de Fermat.
d | Fn  22  1  22  1  0 (mod d )  22  1 (mod d )
n n n

Pero entonces
22  22
m n m n
2
 22 n 2 m n
  1
2 m n
 1 (mod d )

Pero, por otro lado, d | Fm  22  1  22  1  0 (mod d )  22  1 (mod d ) , con lo


m m m

cual llegamos a 1  1 (mod d )  2  0 (mod d )  d | 2 , y puesto que d es impar, solo


puede ser d  1 .

Nota: Un argumento alternativo podría ser aprovechar la identidad


Fn  2  F0 F2 ...Fn 1

Que se deduce de
22  1  2  12  122  1... 22
m
 n 1

1

Y por tanto, si d divide Fn y Fm , con m  n , también dividirá a 2  Fn  F0 ...Fn 1 , y


puesto que d es impar, llegamos a d  1
Números y primos de Mersenne.

19.3 Números de Mersenne. Primos de Mersenne.


Definimos los números de Mersenne como aquellos de la forma M n  2n  1 , n  1.

Está claro que si n es compuesto, también lo será M n :


  b
  
Si n  ab  M n  2ab  1  2a  1b  2a  1 2a1  2a2  ...  2  1  2a  1 | M n

Por lo tanto, si M n es primo, también lo será n.

Sin embargo, existen primos p para los cuales M p es compuesto. Por ejemplo:
47 | M23, 167 | M83, 263 | M13

Llamaremos “primos de Mersenne” a los números de Mersenne que sean primos. Es


un problema abierto la existencia o no de infinitos primos de Mersenne.

19.4 Los mayores primos de Mersenne descubiertos hasta ahora.


El mayor número primo de Mersenne que se conoce es 282589933 1 , descubierto en
diciembre del 2018.

Los primeros 38 primos de Mersenne son los siguientes:

22−1 , 23−1 , 25−1 , 27−1 , 213 – 1 , 217−1 , 219−1 , 231−1 , 261−1 , 289 −1 , 2107−1 ,
2127−1 , 2521−1 , 2607−1 , 21279−1 , 22203−1 , 22281−1 , 23217−1 , 24253−1 , 24423−1 ,
29689−1 , 29941−1 , 211213−1 , 219937−1 , 221701−1 , 223209−1 , 244497−1 , 286243−1 ,
2110503−1 , 2132049−1 , 2216091−1 , 2756839−1 , 2859433−1 , 21257787−1 , 21398269−1 ,
22976221−1 , 23021377−1 , 26972593 −1.
18 Números combinatorios.

El Tema 4 de PC está dedicado íntegramente a los números combinatorios. Se


recomienda practicar con ellos realizando los problemas que allí se proponen.

18.1
Demuestra que la expresión
(m, n)  n 
 
n  m 
es un entero para todo par de números enteros n  m  1 .

Putnam 2000

18.2 Proposición.
 p
a) Si p es primo, p divide   para todo 0  k  p .
k
b) Si p es primo, p | 2  2 .
p

Demostración.
 p p!  p p!
a)     k!    p( p  1)( p  2)...( p  k  1)
 k  k!( p  k )!  k  ( p  k )!

 p
Está claro que p | p( p  1)( p  2)...( p  k  1) , luego p | k! 
k
 p
Puesto que k  p , es imposible que p | k! , luego p |  
k

 p  p  p
b) 2 p        ...   
0 1  p
 p  p  p  p 
Y puesto que       1 , tenemos 2 p  2     ...   
 0   p 1  p  1
 p
Aplicando el apartado anterior, p |   para todo 0  k  p , luego p | 2 p  2 .
k
Números de Catalan.

18.3 Definición. Número de Catalan de orden n.


Definimos el número de Catalan de orden n como
1  2n 
Cn   
n  1  n 

Los primeros diez números de Catalan son 1, 2, 5, 14, 42, 132, 429, 1430, 4862, 16796.

18.4 Proposición.
Todos los números de Catalan son enteros.

Demostración.
2n  1  2n   2n  1
Vamos a aplicar la identidad    
n  1  n   n  1 
Puesto que 2n  1 y n  1 son coprimos, y la parte de la derecha de la identidad anterior
 2n 
es un entero, está claro que n  1 divide a   .
n
19 Números y primos de Fermat y de Mersenne.
Números y primos de Fermat.
19.1 Definición. Números de Fermat. Primos de Fermat.
Queremos estudiar los números primos de la forma 2n  1 . Ya vimos en el Tema 4 que
si es primo entonces n debe ser una potencia de 2.

Definimos el n-ésimo número de Fermat como Fn  22  1 , n  0 .


n

F0 = 3, F1 = 5, F2 = 17, F3 = 257, F4 = 65537, F5 = 4294967297

El propio Fermat conjeturó que Fn era primo para todo n, sin embargo Euler demostró
que 641 | F5 . El razonamiento se encuentra en el Tema 4.

Así pues, no todo número de Fermat es primo. Llamaremos “primos de Fermat” a


aquellos números de Fermat que sean primos. Es un problema abierto la existencia o no
de infinitos primos de Fermat. Se conjetura que Fn es compuesto para n  5 .

19.2 Proposición.
Los números de Fermat son todos coprimos entre ellos:  Fn , Fm   1 si n  m .

Demostración.
Supongamos que m  n , y sea d un divisor común de Fn y Fm . En primer lugar vemos
que d debe ser impar, pues lo son todos los números de Fermat.
d | Fn  22  1  22  1  0 (mod d )  22  1 (mod d )
n n n

Pero entonces
22  22
m n m n
2
 22 n 2 m n
  1
2 m n
 1 (mod d )

Pero, por otro lado, d | Fm  22  1  22  1  0 (mod d )  22  1 (mod d ) , con lo


m m m

cual llegamos a 1  1 (mod d )  2  0 (mod d )  d | 2 , y puesto que d es impar, solo


puede ser d  1 .

Nota: Un argumento alternativo podría ser aprovechar la identidad


Fn  2  F0 F2 ...Fn 1

Que se deduce de
22  1  2  12  122  1... 22
m
 n 1

1

Y por tanto, si d divide Fn y Fm , con m  n , también dividirá a 2  Fn  F0 ...Fn 1 , y


puesto que d es impar, llegamos a d  1
Números y primos de Mersenne.

19.3 Números de Mersenne. Primos de Mersenne.


Definimos los números de Mersenne como aquellos de la forma M n  2n  1 , n  1.

Está claro que si n es compuesto, también lo será M n :


  b
  
Si n  ab  M n  2ab  1  2a  1b  2a  1 2a1  2a2  ...  2  1  2a  1 | M n

Por lo tanto, si M n es primo, también lo será n.

Sin embargo, existen primos p para los cuales M p es compuesto. Por ejemplo:
47 | M23, 167 | M83, 263 | M13

Llamaremos “primos de Mersenne” a los números de Mersenne que sean primos. Es


un problema abierto la existencia o no de infinitos primos de Mersenne.

19.4 Los mayores primos de Mersenne descubiertos hasta ahora.


El mayor número primo de Mersenne que se conoce es 282589933 1 , descubierto en
diciembre del 2018.

Los primeros 38 primos de Mersenne son los siguientes:

22−1 , 23−1 , 25−1 , 27−1 , 213 – 1 , 217−1 , 219−1 , 231−1 , 261−1 , 289 −1 , 2107−1 ,
2127−1 , 2521−1 , 2607−1 , 21279−1 , 22203−1 , 22281−1 , 23217−1 , 24253−1 , 24423−1 ,
29689−1 , 29941−1 , 211213−1 , 219937−1 , 221701−1 , 223209−1 , 244497−1 , 286243−1 ,
2110503−1 , 2132049−1 , 2216091−1 , 2756839−1 , 2859433−1 , 21257787−1 , 21398269−1 ,
22976221−1 , 23021377−1 , 26972593 −1.
20 Número de divisores de un entero.
20.1 Definición. Número de divisores de un entero.
Dado un número entero n, definimos  (n) como el número de enteros positivos
divisores de n.

Ejemplos:
Los divisores positivos de 6 son  1 , 2 , 3 , 6 , luego  (6)  4 .
Los divisores positivos de 20 son  1 , 2 , 4 , 5 , 10 , 20 , luego  (20)  6 .
 (1)  1,  (2) = 2,  (3) = 2,  (4) = 3,  (5) = 2,  (6) = 4,  (7) = 2,  (8) = 4,  (9) = 3,
 (10) = 4
Con Mathematica:

20.2 Proposición.
Para cualquier entero positivo n,
a)  (n)  1 y  (n)  1  n  1 .
b) Para todo n  2 ,  (n)  2 y  (n)  2  n es primo.
c)  (n)  2 n

Demostración.
a) y b) se deducen de la propia definición de  (n) .
c) Cada divisor positivo a de n tiene asociado su divisor complementario b  n / a .
n
Puesto que a  b  a  n , se tiene que cumplir a  n o b  n . Así pues, existirán
a
como mucho 2 n divisores de n.

20.3 Teorema.
La función d (n) es multiplicativa, es decir, si n  a b , con (a, b)  1 , entonces
 (n)   (a) (b)

20.4 Corolario.
a) Si n  p a para cierto p primo, entonces los divisores de n son

1 , p , p 2 , p 3 , ... , p a 
y por tanto  (n)  a  1

b) Si n  p1a1 p2a2 ... prar es la descomposición en factores primos de n, entonces los


divisores d de n son todos los números que se pueden escribir como
d  p1b1 p2b2 ... prbr con 0  bi  ai ,
y
 (n)  a1  1a2  1...ar  1

Por ejemplo:  (2904)  d (23  3 112 )  (3  1)(1  1)(2  1)  24


20.5 F
Demostrar que  (n) es impar si y solo si n es un cuadrado perfecto.

20.6 F
Demostrar que  (n) es primo si y solo si n  p q1 con p, q primos.

20.7 MF
Si tomamos aleatoriamente un divisor de 1099 , ¿Cuál es la probabilidad de que sea
múltiplo de 1088 ?

AIME 1998 #5

20.8 F
Determina todos los enteros positivos n tales que  (n)  6 .

20.9 F

Determina el número de enteros positivos que son divisores de al menos uno de los
siguientes números: 1010 , 157 , 1811 .

AIME II 2005 #4

20.10 D
Sea  (n) el número de enteros positivos de n. Determina la suma de los seis enteros
positivos n más pequeños tales que
 (n)   (n  1)  7

AIME 1 2019 #9

20.11 F
¿Cuántos divisores positivos de 20042004 son divisibles por exactamente 2004 enteros
positivos?

AIME II 2004 #8

20.12 F
Joey, Chloe y su hija Zoe cumplen años el mismo día. Joey es un año mayor que Chloe
y Zoe cumple hoy su primer año. Hoy es el primero de los nueve aniversarios en los que
la edad de Chloe será múltiple de la edad de Zoe. Determina la suma de los dos dígitos
de la edad de Joey que tendrá la próxima vez que sea múltiple de la edad de Zoe.

(A) 7 (B) 8 (C) 9 (D) 10 (E) 11


AMC 12B 2018 #14
20.13 F
Sea N un entero positivo. Supongamos que existen exactamente 2005 pares ordenados
x, y  de enteros positivos tales que
1 1 1
 
x y N

Demuestra que N es un cuadrado perfecto.

BMO 2005 Round 2 #1

20.14 Proposición. Producto de los divisores de un entero.


Una propiedad interesante de  (n) es que el producto de todos los divisores de n es
igual a la raíz cuadrada de n (n ) :
d 
d |n
n ( n )

Demostración.
La clave está en observar que los divisores de un número van por parejas: Dado un
divisor d , tenemos su divisor complementario d ' n / d , de forma que d  d '  n .
Sean d1 , d 2 , ..., d ( n ) todos los divisores de n. Si multiplicamos dos veces dichos
divisores, los factores se pueden reorganizar agrupando  (n) parejas cuyo resultado sea
n, luego:
2 2
   
  d    d   d  d1.d 2 ...d ( n )  d1.d 2 ...d ( n )  n ( n )    d   n ( n )
   
 d |n  d |n d |n  d |n 

20.15 Corolario.
De la proposición anterior se deduce que n (n ) es un cuadrado perfecto. Esto es obvio si
 (n) es par, pero no estaba tan claro si  (n) es impar. Pero en el problema 20.1 anterior
ya demostramos que si  (n) es impar, n es un cuadrado perfecto, y por tanto también lo
es n (n ) .

20.16 Ejemplo.
El producto de todos los divisores de 16 es
 d | 16 
d |n
16 ( n )  165  45  1024

En efecto, 1 2  4  8 16  1024 .


21 Suma de los divisores de un entero.
21.1 Definición. Suma de los divisores de un entero.
Dado un número entero n, definimos  (n) como la suma de todos los enteros positivos
divisores de n, incluyendo 1 y el propio n.
 ( n)   d
d |n

Está claro que  (n)  n  1, y  (n)  n  1 si y solo si n es primo.

Ejemplos.
Los divisores positivos de 6 son  1 , 2 , 3 , 6 , luego  (6)  1  2  3  6  12 .
Los divisores positivos de 20 son  1 , 2 , 4 , 5 , 10 , 20 , luego
 (20)  1  2  4  5  10  20  42 .

21.2 Teorema.
La función  (n) es multiplicativa.

21.3 Proposición.
a) Si n  p a para cierto primo p , entonces  (n)  1  p  p 2  ...  p a .
b) Si n  p1a1 p2a2 ... prar es la descomposición en factores primos de n, entonces

 (n)  1  p1  p12  ...  p1a 1  p2  p22  ...  p2a ...1  pr  pr2  ...  pra  
1 2 r

p1a1 1  1 p2a 2 1  1 pra r 1  1


  ...
p1  1 p2  1 pr  1

Demostración.
a) Aplicando la propia definición de  (n) .
b) Basta tener en cuenta que  (n) es multiplicativa y aplicar la serie geométrica:
x n1  1
x n  x n1  x n2  ...  x  1 
x 1

Por ejemplo:
23  1 33  1 52  1 7 26 24
180  22  32  5   (180)        7 13  6  546
2 1 3 1 5 1 1 2 4
21.4 Problema resuelto.
Sea n un entero positivo tal que 24 | n  1 . Demuestra que entonces se cumple
24 |  (n) .
PUTNAM 1969

Solución.
n  1  0 (mod 3)  n  -1 (mod 3)
24 | n  1  
n  1  0 (mod 8)  n  -1 (mod 8)

Observamos que los divisores de un número van en parejas: Si d1 es divisor entonces su


n
complementario d 2  también es divisor.
d1
Por un lado, haciendo módulo 3:
n
d1  d 2  d1  n  1 (mod 3) , y las únicas opciones son:
d1
d1  1 (mod 3) y d 2  2 (mod 3) o viceversa.
En todo caso se cumple d1  d 2  0 (mod 3) .

Y por otro lado, haciendo módulo 8:


n
d1  d 2  d1  n  1 (mod 8) , y las únicas opciones son:
d1
d1  1 (mod 8) y d 2  7 (mod 8) o viceversa.
d1  3 (mod 8) y d 2  5 (mod 8) o viceversa.
En todo caso se cumple d1  d 2  0 (mod 8) .

Así pues, d1  d 2  0 (mod 3) y d1  d 2  0 (mod 8) , luego d1  d 2  0 (mod 24) , y por


tanto el sumatorio  (n) se puede reordenar por parejas divisibles entre 24, y por tanto:
 (n)   d será divisible entre 24.
d |n

21.5 F
Los únicos factores primos de un número n son 2 y 3. Si la suma de todos los divisores
de n (incluyendo el propio n) es 1815, determina n.

PUMaC 2011/NT #A1


Números perfectos.

21.6 Definición de número perfecto.


Los antiguos griegos clasificaban los enteros positivos en tres clases, en función de la
suma de sus divisores:
“Números perfectos” si  (n)  2n
“Números abundantes” si  (n)  2n
“Números deficientes” si  (n)  2n

Los primeros números perfectos son: 6 , 28 , 496 , 8128 .

Si utilizamos la notación clásica en la que se cuentan solo los divisores propios, es


decir, todos menos el propio n, tiene más sentido la idea de “número perfecto” como
aquel que es igual a la suma de todos sus divisores (propios).

21.7 Teorema. Caracterización de los números pares perfectos.


Un número par n es perfecto si y solo si existen números primos p, q tales que
n  2 p1 q con q  2 p  1

Así pues, cada número par perfecto está asociado a un primo de Mersenne, que fueron
introducidos en el Tema 19.

Demostración.
 Supongamos que n se puede escribir de esta forma.
Está claro que q es impar , 2n  2 p q y que q  1  2 p .
p 11
1
 2    2  1 ,  q   q  1
2
p 1 p

2 1
Luego  (n)   2   q   2  1q  1  2
p 1 p p
q  2 p  (q  1)  2n
Y por tanto n es perfecto.

 Supongamos ahora que n es perfecto, es decir, que  (n)  2n . Puesto que además
estamos suponiendo que n es par, podemos escribir n  2k 1 m con m impar y k  2 .

   
Luego 2k m  2n   (n)   2k 1 m   2k 1  m  2k  1  m  
2k 11  1
Donde hemos utilizado que  2  k 1
  2  1  2k  1 .

 
De la igualdad 2k m  2k  1  m se deduce que 2k  1 | 2k m , y puesto que
 
2k  1 , 2k  1 , el Lema de Euclides implica que 2k  1 | m , y por tanto

 
m  2k  1 l para cierto entero l impar.
  
2 2  1 l  2k m  2n   (n)   2k 1 (2k  1)l   2k 1  (2k  1)l 
k k
   
 2 k
 1 (2 k
 
 1)l  2k l   (2k  1)l  (*)
 
Si l  1 entonces 1, l , (2k  1)l son factores diferentes de 2k  1 l , y por tanto, por la
 
definición de  , tenemos que 1  l  2k  1 l   2k  1 l  
   
Luego 2k l   2k  1 l  1  l  2k  1 l  1  l  2k l  l  1  2k l absurdo.
 
Así pues, l  1, y n  2k 1 2k  1 . Ahora solo falta demostrar que k es primo.

 
De (*) tenemos que 2k   2k  1  1  2k  1  d  d  0
d |( 2 k 1) d |( 2 k 1)
1 d  2 k 1 1 d  2 k 1

Es decir, 2k  1 no tiene divisores propios, es decir, es un número primo. Ya vimos en el


Tema 15 que si un número de Mersenne es primo entonces k es primo, y con esto
acabamos la demostración.

21.8 Observación.
Acabamos de ver que todo número perfecto par está asociado a un primo de Mersenne.
Es un problema abierto la existencia o no de infinitos números perfectos pares, y apenas
sabemos nada de los números perfectos impares, ni tan solo si existe alguno.
Soluciones.
1.2
Supongamos que N  n  0  n  1  n  2  n  3  ...  n  99  100n  (1  2  3  ...  99)
99(99  1)
Sabemos que 1  2  3  ..  99   4950
2
Luego N  100n  4950 , luego será un número que acabe en "50", y el único candidato
aceptable es (A).

1.3
N  9  99  999  9999  ..  99
 99 
...
321cifras

      
 10  1  10  1  10  1  ...  10321  1
1 2 3

 
 10  10  10  ...  10   1
1 2 3

321
 ...
1  1  10  10
1 2

 103  ...  10321  321  (*)
 321 
10  10  10  ...  10  10  100  1000  ..  100
1 2 3 321
...   111
000
 ...111
0
321 321

(*)  111
 0  321  111
...111 ....
111
0789
321 321 3 318
Las cifras de este número suman 1 318  0  7  8  9  342

1.5
Vamos a ir observando la pauta sn  1  2  3  ...  n
s1  1  d1  1
s2  1  2  3  d 2  3
s3  1  2  3  6  d3  3
s4  1  2  3  4  10  d 4  0
s5  1  2  3  4  5  15  d 5  5
s6  1  2  3  4  5  6  21  d 6  1
s7  1  2  3  4  5  6  7  28  d 7  8
Las sucesivas sn siguen de la siguiente manera:36, 45, 55, 66, 78, 91, 105, 120, 136, 153,171,
190,210,231,253,276,300,325,351,378,406,435,465,496,528,561,595,630,666,703,741,
780,820,861,903...
Vemos que los primeros d1  d2  ...  d19  70 .
y vemos que, a partir de la posición d19 sigue una pauta repetitiva de 20 números:
0, 0, 1, 3, 6, 0, 5, 1, 8, 6, 5, 5, 6, 8, 1, 5, 0, 6, 3, 1
que suman 70.

Entre d19 y d 2017 hay 20 bloques completos y 19 elementos "sueltos" finales:


2017  18  1999  99  20  19
Los 19 primeros elementos de la pauta anterior suman 69, luego
2017

d
n 1
n  70  99  70  69  7069 . El residuo de 7069 entre 1000 es 69.
1.5
Vamos calculando pacientemente la sucesión:
a3  4(8  5  2)  60  5 , a4  4(5  8  5)  76  6 ,
a5  10 , a6  7 , a7  4 , a8  7 , a9  6 , a10  2 , a11  5 , a12  8

Vemos que los tres últimos elementos coinciden con los primeros, por lo que podemos asegurar
que los elementos se van repitiendo en grupos de diez.

2018  201  10  8  a2018  a8  7 



2020  202  10  0  a2020  a0  2  a2018  a2020  a2022  7  2  8  112
2022  202  10  2  a2022  a2  8

1.6
2  4  6  8  384 , luego la multiplicación de cuatro números, acabados en 2, 4, 6 y 8 acabará en
4.
La multiplicación de números acabados en 4 va alternando: Acabados en 6 y acabados en 4:
42  4  4  16, 43  16  4  64, 44  64  4  256,... luego 410 acabará en 6.
2  4  6  8 12 14 16 18  22  24  26  28  ...  92  94  96  98 la podemos entender como la
multiplicación de diez grupos, cada uno de ellos acabado en 4, luego el resultado será el mismo
que 410 , es decir, 6.

1.7
Supongamos que nuestra sucesión es de la forma xn  a n  b para ciertos enteros a, b .
Supongamos que para cierto n se cumple xn1  a n1  b  m2 .
Entonces
m  a2  m2  a 2  2am  an1  b  a 2  2am  an1  a  2m  b
Y por tanto el término an2 con n2  n1  a  2m  n1 es también un cuadrado perfecto. Esta
sucesión de cuadrados perfectos la podemos hacer tan grande como queramos.

1.8
N  a  112  b  11  c  1  83  b  82  c  8  a , con 0  a, b, c  7 .

0  1  83  b  82  c  8  a  a 112  b 11  c 
   
0  1  83  82  11 b  8  1c  1  112 a 
0  512  53b  7c  120a 
120a  53b  7c  512

Luego
120a  53b  7c  512  512  a  512 / 120  4

Con a  4 nos quedamos cortos, luego a  5 .

Supongamos que a  5 . La ecuación queda de la forma:


600  53b  7c  512 
600  53b  7c  512 
88  53b  7c

Con b  0 la ecuación resultante 88  53  0  7c  7c no tiene solución entera.


Con b  1 la ecuación resultante 88  53 1  7c tiene solución entera:
88  53  1  7c  88  53  7c  35  7c  c  5

Luego hemos encontrado la solución a  4, b  1, c  5 , y como en todo momento hemos


trabajado con valores mínimos, esta será la solución mínima que nos pide el enunciado.

1.10
a 2  b2
Supongamos, por el contrario, que existe un k  , con a, b enteros positivos que no es
1  ab
un cuadrado perfecto.
Por ser a, b simétricos en la expresión, podemos suponer que a  b .
Podemos suponer, además, que b es el entero positivo más pequeño posible.

En primer lugar, vemos que a  b , pues


a 2  b2 2a 2
abk    2  k  1 , pero 1 es un cuadrado perfecto: 1  12 , llegando a
1  ab 1  a 2
contradicción.

Así pues, a  b .
a 2  b2
k  1  ab k  a 2  b 2  k  abk  a 2  b 2
1  ab
 a 2  b 2  k  abk  0  b 2  akb  a 2  k  0

Esta última expresión la podemos considerar como una ecuación de segundo grado en b, con lo
que existirán dos soluciones, b,b1 , tales que:
ak  b  b1
a 2  k  b b1

Puesto que a, b  0 , la hipótesis b1  0 contradice la condición


1  ab1 k  a 2  b12
Puesto que k no es un cuadrado perfecto, la hipótesis b1  0 también contradice la condición
anterior:
1  a  0k  a 2  02 . Así pues, b1  0 .
Por otro lado:
b1b  b1b  b1 ak  b1   a 2  k  akb1  b12  a 2  k  akb1  k  a 2  b12
a 2  b12
 k ab1  1  a 2  b12  k 
ab1  1
Y también:
a 2  k b2  k
b1   b
b b

a 2  b12
Luego hemos encontrado un nuevo número b1  b que también satisface k  , cuando
ab1  1
habíamos supuesto que b era el menor número posible, llegando a contradicción. Por lo tanto,
k tiene que ser un cuadrado perfecto.

Fuente de la solución: Elementary Number Theory Notes (David A. Santos, 2004) , pág. 3.

2.1
Basta realizar la suma “en columnas”:

1 5 7 2 8
2 2 A 0 4
 C D 3 3 1
5 7 2 6 3

De izquierda a derecha vamos deduciendo A=2, D=9 y C=1 (B)

2.2
b  2a  1
  c  2(2a  1)  1  4a  2  1  4a  3
c  2b  1
Se cumple c  1000 , luego a  250 , luego a  2N 2 con N  5 o bien a  1M 1 .

Probamos las diferentes posibilidades:


a  242  b  2a  1  485 no cumple las condiciones del enunciado.
a  232  b  2a  1  465 no cumple.
a  222  b  2a  1  445 no cumple.
a  212  b  2a  1  425 no cumple.
a  202  b  2a  1  405 no cumple.
a  101  b  2a  1  203 no cumple.
a  111  b  2a  1  223 no cumple.
a  121  b  2a  1  243 no cumple.
a  131  b  2a  1  263 no cumple.
a  141  b  2a  1  283 no cumple.
a  151  b  2a  1  303 sí cumple, pero c  2b  1  607 no cumple.
a  161  b  2a  1  323 sí cumple, pero c  2b  1  607 no cumple.
a  171  b  2a  1  343 sí cumple, pero c  2b  1  687 no cumple.
a  181  b  2a  1  363 sí cumple, y c  2b  1  727 también cumple.
a  191  b  2a  1  383 sí cumple, y c  2b  1  767 también cumple.

Luego cumplen dos números (C).

2.3
Basta realizar la suma en columnas:
7 2 4 3
2 1 A 7
 C D 2 6
1 1 1 2 6

Para deducir que A=5, D=7 y C=1 (B)

2.4
2019  224  9  3 , luego con 224 dígitos seguro que nos quedamos cortos. Con 225 dígitos, el
más pequeño será 39999  ... cuya primera cifra es 3 (B)
999
223

2.5
Para llegar a T , R o P o S necesita realizar un número par de pasos. Para llegar a Q necesita un
número impar de pasos. El número 2019 es impar, luego solo a Q. (C)

2.6
Analizando detenidamente la larga cadena de operaciones involucradas en este problema,
podemos especular que g (n)  10 , el primer valor que no se puede representar en base 16 con
los dígitos  0 ,1, 2 , ... , 9 .

En base 8 disponemos de los dígitos del 0 al 7, y por lo tanto, el valor mínimo para que la suma
sea 10 será 3+7, es decir, el número 378  3  8  7  31 .

Finalmente, buscamos un número cuya suma de dígitos en su representación en base 4 sea 31.
En base 4 tenemos los dígitos  0 ,1, 2 , 3 . 31  3 10  1 , luego el número buscado será
 
n  133333333334  1 411  3  410  3  49  ...  3  4  3  40  411  3 410  49  ...  4  1

Aplicando la fórmula de la serie geométrica:


1  411 411  1
410  49  ...  4  1  
1 4 3

Para calcular 411  222 a mano vamos calculando potencias de 2:


210  1024
220  1024 1024  1048576
411  222  220  4  4194304

Con lo que, finalmente:


 411  1 
n  411  3   2097151
 3 
2.7
 10   9   8   7   6   5   4   3   2   1  0 
1  2  3  4  5  6  7  8  9  10  11  12  13  14 
 11  12  13  14  50
Luego en cada fila, columna y diagonal la suma será 10 (C)
2.8
Interpretando esta ecuación como una ecuación de segundo grado en y, tenemos
2  4  4 1 x 2020 2  4(1  x ) 2  2 1  x 2020
2020
y 2  2 y  x 2020  0  y    
2 2 2
 1  1  x 2020
x  1
Que tendrá solución siempre que 1  x 2020  0  
x  0
x  1  y  1  0  1 una solución
x  0  y  1  1  0,1 dos soluciones
En total, hay 3 soluciones (C)

2.9
Vemos que log a es un entero positivo si y solo si a  10 c  a  10 c   2
 10 2c para cierto
entero positivo c .
Luego  
log a  log 102c  2c y debe ser un entero, luego c  2e2 para cierto entero e , y
 
log a  log 102c  2c  2e
Por otro lado, log a  c  2e2 .

De la misma forma vemos que b  10d  b  10d  2


 102d para cierto entero positivo d.
Luego  
log b  log 102 d  2d debe ser un entero, luego d  2 f 2 para cierto entero f , y
log b  2d  2 f
Por otro lado, log b  d  2 f 2 .

Así pues, la ecuación del enunciado se transforma en


100  log a  log b  log a  log b  2e  2 f  2e2  2 f 2 
50  e  f  e2  f 2  e(e  1)  f ( f  1)

Buscamos posibilidades: 1  2  2 , 2  3  6 , 3  4  12 , 4  5  20 , 5  6  30 , 6  7  42
50  2  48 , 50  6  44 , 50  12  38 , 50  20  30
50  32  20 , 50  42  8

Las únicas combinaciones aceptables son e  4, f  5 y e  5, f  4 .

Luego
 
a  10c
2
 102c  1022e  104e 
2 2

 ab  104e  104 f  104e  4 f  104 e  f  


2 2 2 2 2 2


b  10   102 d  1022 f  104 f 
d 2 2 2

 104 4 52   10164 ( D)


2

Observación: En las soluciones oficiales


https://artofproblemsolving.com/wiki/index.php?title=2019_AMC_12A_Problems/Problem_15&oldid=122937 se presentan otros desarrollos
alternativos.
2.10
Haciendo el listado exhaustivo de todos los valores que vamos obteniendo para valores
pequeños de n , vemos que los conjuntos


Sn  an  3n  an 1  3n 1  ...  a1  31  a0  30 ,  1  ai  1 
Constan de 3n enteros, sin que se produzcan repeticiones, y se cumple 0  Sn , y hay simetría:
x  Sn   x  Sn .
Así pues, en particular, para el caso n  7 , tendremos 38  34  34  81 81  6561 números
diferentes: 3280 negativos, 3280 positivos y el cero. El total de elementos no negativos es, por
tanto, 3281 (D).

2.11
Fijando un k  1,2,...,2450 , los triangulos ABC y APk Qk son semejantes por Tales, ya que
Pk Qk // BC .

Las regiones separadas por segmentos tienen la misma área


ABC  .
2450
Los trapecios se pueden unir de forma que APk Qk   k
ABC 
2450
Es decir, la razón de proporcionalidad entre las áreas de los triángulos es:
APk Qk   k  k
ABC  2450 2  52  72
Sabemos que las áreas están en proporción el cuadrado de la razón de semejanza de las
longitudes de los lados, luego la razón de proporcionalidad de las longitudes es:

k k /2
 
25 7
2 2
57

Y por tanto
k /2 1 3k 1 3  3k 3 k
Pk Qk  BC    5 3  5 3    que será un número racional
57 7 2 7 3 2 7 6
k
si y solo si es racional, es decir, para todos los k tales que k / 6 sea un cuadrado perfecto:
6
k / 6  p2  k  6 p2

p  1  k  6  12  6 , p  2  k  6  22  24 , ... , p  20  k  6  202  2400 ,


Con p  21  k  6  212  2646 ya nos pasamos luego hay 20 valores de k aceptables.

2.12
PR #1.16

2.13
Para resolver este problema vamos a aplicar la siguiente identidad notable:
a 2
 
 b2 c 2  d 2  ac  bd   ad  bc 
2 2

Esta identidad se demuestra fácilmente mediante números complejos, tomando z1  a  bi y


z2  c  di , vemos que se reduce a la conocida identidad
z1 z2  z1 z2  z1 z2  z1 z2
2 2 2

Volviendo a nuestro problema, multiplicamos por 5  12  22 ambos lados de la igualdad y


aplicamos la identidad anterior:

732  342  5  1297  732  342 12  22  52  1297   
73  1  34  2  73  2  34  1
2 2
 5  1297 
2

5  180  5  1297 
2 2 2

  2
52  22  32  5  52  1297 
5  5 2  3   5  1297 
2 2 2 2 2 2

1  2  3   1297 
2 2 2

12  362  1297

2.14
Basta con multiplicar esta expresión por 1  (3  2) y aplicar la identidad “suma por
diferencia”:
(a  b)(a  b)  a 2  b2
En nuestro caso:
(3  2)2  3  32  22
32
 22 22  32   32   22   34  24
2 2

34
 24 24  34  34   24   38  28
2 2

Para ir cancelando, uno por uno, todos sus términos hasta quedar 3128  2128 (C)

2.15
Analizando la tercera columna vemos que acaba en 4, y analizando la primera columna vemos
que el resultado es 26 contando el acarreo, luego la suma será 24. (C)

2.16
En primer lugar podemos suponer a  1 es decir mínimo, para buscar el máximo de b.
Vemos que para b  c  1000 se cumple a  b  c  1000000 , y que para cualquier número
mayor, por ejemplo b  c  1001 entonces a  b  c  1000000 . (D)

2.17
El 0 se puede obtener tomando a  0 , b  0 , c  0 .
El 2 se puede obtener tomando a  1 , b  0 , c  0 .
El 8 se puede obtener tomando a  2 , b  0 , c  0 .
El 6 se puede obtener tomando a  1 , b  1 , c  0 .
Luego solo nos queda la solución 1. (B)
2.18
(B)

2.19
Puesto que Par+Par=Par, Impar+Par=Impar, Impar+Impar=Par, la sucesión sigue la siguiente
pauta: I I P I I P I I P I I P...
2020  3  673  1 , luego habrán 673 números pares. (A)

2.20
Basta aplicar la propiedad conmutativa en la columna de la derecha: 79  2  158 (B)

2.21
Por tanteo se llega a  1  0  1  2  2 (C)

2.22
El próximo será 2121, y transcurrirán 101 años (B)

2.23
Se deduce que es 2 (B)

2.24
Sea x el valor central. Sumando los elementos de dos formas diferentes llegamos a la ecuación
40  4 x  13  4 cuya solución es x  3 (A)

2.25
95  55  150 km y no llegaría con los 14 litros de combustible. La anterior está en el kilómetro
75  55  130 y le faltarían recorrer 520  130  390 kilómetros con lo que necesitaría 39 de
los 40 litros de gasolina disponibles (D).

2.26
Empezamos por el número 7 que solo puede ser 1  2  4 . Luego, por tanteo, llegamos a la
siguiente combinación aceptable:

Cuyas caras suman 7, 8, 10, 9 y 13. Luego la solución es 13 (C)

2.27
Observando las sumas parciales que aparecen y por tanteo, llegamos a esta solución aceptable:

Con lo que la respuesta correcta es 7 (C)


2.28
En primer lugar vemos que
1 1 1 1 1 1 3 3
5abc       
a b c 5 5 5 5 4
Y no se podrá cumplir la igualdad del enunciado. Luego nos podemos reducir a los casos
1 a  4

Primer caso: a  1 .
1 1 3 1 1 3 1
1      1  lo cual es imposible con b, c positivos.
b c 4 b c 4 4

Segundo caso: a  2
1 1 1 3 1 1 3 1 1
       
2 b c 4 b c 4 2 4

1 1 2 1
Si 8  b  c     , y la igualdad requerida no se podrá cumplir. Luego nos
b c 8 4
reducidos a 2  b  8 .
1 1 1 bc 1
     4(b  c)  bc  bc  4c  4b  c(b  4)  4b
b c 4 bc 4
4b
c
b4
4b
De aquí se deduce claramente que c  0b4
b4
Vamos probando casos:
45
b 5c   20
54
46
b6c   12
64
4  7 28
b7c  no aceptable
74 3
48
b 8c  8
84

Segundo caso: a  3
1 1 1 3 1 1 3 1 5
       
3 b c 4 b c 4 3 12
1 1 2 5
Si b, c  5     pues 24  2  12  5  5  25 . Luego nos podemos reducir a los
b c 5 12
casos 1  b  4
1 1 5 bc 5
     12(b  c)  5bc  12b  12c  5bc  12c  5bc  12b
b c 12 bc 12
12b
 c(12  5b)  12b  c 
5b  12
Los únicos valores aceptables son
12  3 12  4
b 3 c  3 , b4c  6
15  12 20  12

Tercer caso: a  4
1 1 1 3 1 1 3 1 1
       
4 b c 4 b c 4 4 2

1 1 2 1
Si b, c  4     y no se puede dar la igualdad, luego la única opción posible es
b c 4 2
1 1 1
b, c  4   
b c 2
Que efectivamente cumple la condición requerida.

Luego las soluciones son a  2, b  5, c  20 ; a  2, b  6, c  12 ; a  2, b  8, c  8 ;


a  3, b  3, c  12 ; a  3, b  4, c  6 ; a  4, b  4, c  4 .

3.7
  16
416525  22 525  2216525  232525  27 225525  128  (2  5) 25  128  1025 
 1.28  1027
Un número que tiene 28 cifras decimales.

3.8
Si n es impar, n  2k  1 para cierto k entero, luego
n2  2k  1  4k 2  4k  1  4k (k  1)  1
2

Ahora bien, el producto de dos números consecutivos es siempre par, pues o bien k es par o
bien k  1 es par, y por tanto 4k (k  1) es múltiplo de 8, tal y como queríamos ver.

3.9
Sea x la longitud del lado del cuadrado.
Entonces x  d es la longitud del lado del triángulo equilátero y
3( x  d )  4 x  1989 
3x  3d  4 x  1989 
3d  x  1989 
3d  1989  x

Puesto que suponemos x  0 , se tiene que cumplir 3d  1989  0 y esto solo pasa si d  663 .
Luego existen 663 casos (contando d  0 ) que no son válidos para d.

3.10
La condición (i) equivale a a  4,5
La condición (ii) equivale a d  0,5
La condición (iii) equivale a a b 34,35,36,45,46,56
Luego el número posible de enteros es 2  6  2  24

3.11
Una vez introducida la cuarta puntuación, el resultado tiene que ser múltiplo de 4.
El 80 es solución al problema. En efecto, 71  76  82  91  320 , que es múltiplo de 4.
Cualquier otra posibilidad no es aceptable:

71  400  71  329 , 76  400  76  374 , 82  400  82  378 , 91  400  91  389

Ninguno de estos números es múltiplo de 4.

3.12
Tenemos que resolver la ecuación x 2  6 x  n2 con x, n enteros.
x2  6 x  n2  x2  6 x  n2  0 

x  

 6  62  4  1  (n 2 )  6  36  4n 2  6  4 n 2  9


2 2 2
 6  2 n2  9
  3  n 2  9
2

Luego n2  9 debe ser un cuadrado:


n 2  9  m2  9  m2  n 2   m  n  m  n 
Por otro lado, m  n  m  n , luego la posibilidad 9  3  3 queda descartada, y la única
m  n 1
posibilidad es  m  n  m  n   1  9  
m  n  9

Resolvemos el sistema anterior:



m  n 1 m 1 n
 1 n  n  9  1 2n  9  n  4  m 1 4  5
m  n  9

Así pues, las soluciones son:


n  4, m  5  (m  n)(m  n)  (1)  (9)  9
n  4, m  5  (m  n)(m  n)   9 1  9
n  4, m  5  (m  n)(m  n)  (9)  (1)  9
2
En todo caso, n 2  m2  9  25  9  16  x  3  16  9  3  25  3  5  
 8
3.13
Primera versión.
27b  2b  7 es un cubo perfecto, es decir, 2b  7  m3 para cierto entero m.

b  1000  m3  2b  7  2007

Puesto que hay menos cubos perfectos que cuadrados perfectos, listamos todos los cubos:

23  8, 33  27, 43  64, ..., 123  1728


Puesto que 2b  7 es siempre impar, podemos eliminar de la lista todos los cubos pares, y
quedarnos solo con
33 , 53 , 73 , 93 ,113

36b  3b  6 es un cuadrado perfecto, es decir, 3b  6  n2 para cierto entero n.

n2  3b  6  3b  2  3 | n2  3 | n  n  3k  n2  (3k )2  9k 2

Luego nos podemos quedar solo con aquellos cubos divisibles entre 9: 33 , 93

Ya solo nos queda comprobar si estos candidatos se adaptan a nuestras condiciones:


33  27  2b  7  b  10  3b  6  36  62 , luego la base 10 es aceptable.
93  729  2b  7  b  361  3  361  6  1089  332

Luego el resultado es 361  10  371.

Segunda versión.
36b  3b  6  3(b  2)  n2 es un cuadrado perfecto, luego n  3k  n2  9k 2 .
3(b  2)  9k 2  b  2  3k 2  b  3k 2  2

El valor de k está limitado superiormente: b  1000  3k 2  2  1000  k  18

Sustituyendo en la segunda ecuación:


   
m3  2b  7  2 3k 2  2  7  6k 2  4  7  6k 2  3  3 2k 2  1

y vemos que 3 es divisor de m3 , y por tanto 3 es divisor de m .


También vemos que es impar:
 
2k 2 par  2k 2  1 impar  3 2k 2  1 impar

Y que está acotado superiormente:


b  1000  m3  2b  7  2  1000  7  2007

y 132  2197 , luego m  13

Los cubos que cumplen las condiciones anteriores son dos: 33 , 93 , y como en la versión
anterior, solo nos queda comprobar que, efectivamente, satisfacen las condiciones del
enunciado.

Fuente de estas versiones: www.artofproblemsolving.com

3.14
n2  85n  2017 es un cuadrado perfecto 
n2  85n  2017  m2 para cierto m  Z 

n 2  85n  2017  m2  0  n 

 85  852  4  1  2017  m2


 85  4m2  843

2 1 2 1
4m2  843 es un cuadrado perfecto 
4m2  843  p 2  4m2  p 2  843   2m  p  2m  p   843  3  281

Las dos posibilidades son:


a)

 2m  p  3 
2 m  p  3
   m  71, p  139
 2m  p  281 
 2 m  p  281

p  139  4m 2  843  p 2  1392 


 85  4m 2  843  85  1392  85  139 27
n   
2 1 2 1 2 1  112
b)

 2m  p  1 
2 m  p  1
   m  211, p  421

 2 m  p  843 
2 m  p  843

p  421  4m 2  843  p 2  4212 


 85  4m 2  843  85  4212  85  421 168
n   
2 1 2 1 2 1  253

Comprobamos estas cuatro soluciones:


n  27  n2  85n  2017  5041  712
n  112  n2  85n  2017  5041  712
n  168  n2  85n  2017  44521  2112
n  253  n2  85n  2017  44521  2112

Finalmente, nos piden la suma de las soluciones positivas: 27  168  195 .

3.15
Seguimos la indicación propuesta.
Todo número n acabado en "08" es múltiplo de 4. En efecto, se podrá escribir como
n  m  100  8  425m  2
Luego todo número de la forma 111...111 se puede escribir como
111...111  111...108  3  4k  3 para cierto k natural.
Supongamos que 111...111  111...108  3  4k  3  a 2 para cierto a  IN
Está claro que a 2 es impar, luego a será impar, y por tanto a  2b  1 , para cierto b  IN .

Luego
a 2  2b  1  4b 2  4b  1  4k  3  4b 2  4b  1 
2

 
4k  4b 2  4b  2  2 2b 2  2b  1 
2k  2b(b  1)  1

Lo cual es contradictorio, pues 2k es par, y 2b(b  1)  1 es impar, llegando a contradicción.


3.16
Sabemos que x 2  x  n  ( x  a)( x  b)  x 2  (a  b) x  ab

Puesto que el término independiente es negativo, a o b deben ser negativos. Por lo tanto,
buscamos a, b positivos tales que:
ab  n
x 2  x  n  ( x  a)( x  b)  x 2  (a  b) x  ab    n  b(b  1)
a  b  1  a  b  1
Los números entre 1 y 100 que se pueden escribir como el producto de un número y el
siguiente son los siguientes nueve:

1 2  2 , 2  3  6 , 3  4  12 , 4  5  20 , 5  6  30 , 6  7  42 , 7  8  56 , 8  9  72 , 9 10  90

3.17
Nos basaremos en la igualdad
1  a  n  m  100
x 2  ax  b  x  n x  m  x 2  (n  m) x  n m  
b  n m  0
La segunda condición implica que n, m tienen el mismo signo, pero entonces la primera
condición implica que los dos no pueden ser negativos, luego n, m serán positivos o cero.

Si b  0 , el polinomio x 2  ax  b  x 2  ax  xx  a  es factorizable siempre, luego hay cien


casos: 1  a  100 .

Supongamos que 0  b  n m  n, m  0

Al no encontrar más restricción, procedemos a contar casos.

Para cada valor de 1  a  100 , las parejas no ordenadas n, m, n, m  0 , tales que n  m  a
(no importa el orden pues (n, m) y (m, n) dan lugar a la misma ecuación pues la ecuación es
simétrica en n, m)

Por ejemplo:
a 1 
a  2  1,1
a  3  1,2
a  4  1,3, 2,2
a  5  1,4, 2,3
a  6  1,5, 2,4, 3,3
a  7  1,6, 2,5, 3,4
a  7  1,6, 2,5, 3,4
a  8  1,7, 2,6, 3,5, 4,4
a  9  1,8, 2,7, 3,6, 4,5
...
a  98  1,97, 2,96, 3,95, 4,94...
a  99  1,97, 2,96, 3,95, 4,94...
a  100  1,99, 2,98, 3,97, 4,96...

Vemos que van por parejas, y que la suma de todos los elementos será
1  1  2  2  3  3  4  4  5  5  ...  49  49  50 
49
49  50
2 n  50  2  50  49  50  50  2500
n 1 2

Y sumando los 100 casos cuando b  0 dan un total de 2600 casos.


Finalmente, 2600 mod1000  600

3.18
Primera versión.
Las secuencias a tratar son de la forma

 2a  1 , 2a  3 , ... , 2a  31    2a  2b  1  1  b  16

La suma será
16 16 16
s   2a  2b  1  162a  1   2a  162a  1  2 a 
b 1 b 1 b 1

16  17
 162a  1  2  32a  16  16  17  32a  16  16  25 a  8
2

Buscamos valores de a de forma que s  25 (a  8) sea un cubo perfecto.

25 (a  8)  23 22 (a  8) es un cubo perfecto  22 (a  8) es un cubo perfecto


22 (a  8)  d 3  2 | d3  2 | d  d  2e  e3  2e   23 e3 
3

22 (a  8)  23 e3  a  8  2e3  a  2e3  8  2 e3  4  
Luego los valores de a buscados son los de la forma a  2e3  4

Ahora solo queda acotar el rango de valores, pues nos imponen


100  2a  1  999  99  2a  998  99 / 2  a  998 / 2  49.5  a  498 
50  a  498
Y por tanto
 
50  2 e3  4  498  50 / 2  e3  4  498 / 2  25  e3  4  249 
25  4  e  249  4  29  e3  253
3

33  27 , 53  125 , 63  216 , 73  343 , luego 4  e  6 , y hay tres secuencias.


Segunda versión. La solución oficial de este problema es mucho más expeditiva.
Queremos determinar secuencias n, n  2, n  4, n  6,..., n  30 con n impar y cuya suma sea
un cubo perfecto.
La suma es 82n  30  23 (2n  30) será un cubo perfecto si y solo si 2n  30 es un cubo
perfecto.
Luego buscamos valores n impares y 101  n  999 tales que 2n  30 sea un cubo perfecto (y
además es par).
Los cubos pares entre 232 y 1968 son 512, 1000 y 1728, que se corresponden con los valores
de n: 241, 485 y 849. Luego hay exactamente tres secuencias válidas.

Fuente de la segunda versión: Solución oficial de la OME.

3.19
a) Los números pares e impares van alternados, luego en la secuencia n(n  1) al menos uno de
los factores es par.
b) De la misma manera, para una sucesión de tres números consecutivos, al menos uno de ellos
ha de ser múltiplo de tres. Y también se puede aplicar el apartado a, luego la sucesión será
múltiplo de 2  3  6 .
      
c) n5  n  n n4  1  n n2  1 n2  1  nn  1n  1 n2  1 , luego podemos aplicar los
apartados anteriores para garantizar que es divisible entre 6. Veamos que también es divisible
entre 5.
Por el Teorema de la división, el número n se puede escribir como n  5k  q , 0  q  4 .
Si n  5k  0  n  5k y sería un factor de n5  n .
Si n  5k  1  (n  1)  5k y sería un factor de n5  n
Si n  5k  4  (n  1)  5k y sería un factor de n5  n .
 
Si n  5k  3  (n2  1)  (5k  3)2  1  52 k 2  2  5  3k  9  1  5 5k 2  6k  2 y sería un
factor de n5  n .
 
Si n  5k  2  (n2  1)  (5k  2)2  1  52 k 2  2  5  2k  22  1  5 5k 2  4k  1 y sería un
factor de n5  n .
Así pues, 5 | n 5  n .
Y puesto que también son divisores 2 y 3, lo será su producto: 2  3  5  30 | n 5  n

3.20
Los múltiplos de 5 acaban todos en 0 o en 5, luego si es uno menor, acabará en 4 o en 9.
Los múltiplos de 4 son todos pares, luego si sumamos uno, serán impares, y por tanto no
pueden acabar en 4.
Por lo tanto, solo nos queda buscar entre los primos acabados en 9 que sean de la forma 4k  1 .
Solo hay dos: 29  4  7  1 y 89  4  22  1 .
Y su suma es 29  89  118 .

3.21
log b 729  a  b a  729  36  323 . Las posibilidades son las cuatro siguientes:
323  b  3, a  6
3   b  9, a  3
2 3

3   b  27, a  2
3 2

3   b  729, a  1
2 3 1

3.22
a) Aplicando el algoritmo de la división, todo entero n siempre será de la forma
3a  0 , 3a  1 , 3a  2
 
n  3a  0  n 2  3a   9a 2  3  3a 2  3k
2

 
n  3a  1  n 2  3a  1  9a 2  6a  1  3 3a 2  2a  1  3k  1
2

n  3a  2  3(a  1)  1  3b  1, tomando b  a  1
 
y por tanto n 2  3b  1  9b 2  3b  1  3 3b 2  b  1  3k  1
2

b) Es una aplicación directa del apartado anterior, pues


 
n 2  3a 2  1  3k  3a 2  3k  1  3 a 2  k  1  3 | 1 absurdo.
 
n 2  3a 2  1  3k  1  3a 2  3k  2  3 a 2  k  2  3 | 2 absurdo.

3.23
En primer lugar dividimos numerador entre denominador:

N2 +7 N+4
-4N +7 N-4
23
N2  7 23
Luego  N 4
N 4 N 4
23
Y por tanto nuestro problema se reduce a determinar las fracciones no irreducibles,
N 4
cuando 23 y N  4 tengan algún divisor común. Puesto que 23 es primo, esto solo pasará
cuando N  4 sea múltiplo de 23.

23 | N  4  N  4  23k  N  23k - 4
5 1994
1  23k - 4  1990  1  4  23k  1990  4  5  23k  1994  k
23 23
 1  k  86

La solución es 86.

3.24
El enunciado nos está diciendo que
1059  k1 d  r

1417  k 2 d  r
2312  k d  r
 3

Restando la segunda ecuación a la primera obtenemos:


358  k2  k1 d  d | 358  2 179

Restando la tercera a la segunda obtenemos:


895  k3  k2 d  d | 895  5 179

Y restando la tercera a la primera obtenemos:


1253  k3  k1 d  d | 1253  7 179

Puesto que d  1 , la única posibilidad es d  179 , y entonces, realizando por ejemplo la


primera división tenemos
1059  5 179  164  r  164

Finalmente, d  r  179  164  15

3.25
n2  23  n2  1  24  (n  1)(n  1)  24

Luego basta demostrar que existen infinitos n tales que 24 | (n  1)(n  1) .

Tomando n  12k  1, por un lado tenemos n  1  12k  12 | n  1

Pero también vemos que n es impar, y por tanto n  1 es par, luego 2 | n  1

12 | n  1
  24 | (n  1)(n  1)  24 | (n  1)(n  1)  24  n  23
2
Finalmente,
2 | n 1 

Nota: Más fácilmente, bastaba tomar elementos de la forma n  24k  1 .

3.26
d | n2  1 

 
  d | n  1  1  n  1  2n  1
2 2

d | n  1  1
2

d | 2n  1  d | 2n  1  4n 2  4n  1
2

d | 4n 2  4n  1

   
  d | 4 n  1  1  4n  4n  1  4n  7
2 2

d | n  1  1 
2

d | 2n  1 
  d | 4n  7  22n  1  5
d | 4n  7 

d | 5  d 1, 5

 1 , 5 | 2 1
2

Las dos soluciones son válidas, basta tomar n  2  



 1 , 5 | (2  1)  1
2

3.27
El número n se puede escribir como n  10a  b para ciertos a  0 y 0  b  9 .
Luego la condición del enunciado es a | 10a  b , y puesto que a | 10a se deduce que a | b

Recíprocamente, si a | b tendremos seguro que a | 10a  b . Luego la solución es todo 0  b  9


y todo a | b .

3.29
En primer lugar, vemos que d 2  2 , pues si d 2  2 entonces n es impar y el resto de divisores
también son números impares, pero entonces d 22 , d33 también son impares, y n  d 22  d33 es la
suma de dos impares, luego par, llegando a contradicción.

Así pues, 1  2  d3  ...  dk  n , y n  22  d33  4  d33

En segundo lugar, d 3  3 , pues si d 3  3  n  22  33  31 que no es aceptable.

En tercer lugar, vemos que d 3 tiene que ser 4, pues


d3 | d33 
  d3 | 4  n  d3  d3  4 , y como d3  1,2,3 , necesariamente d 3  4 .
3

d3 | n 

Y, finalmente, n  d22  d33  22  43  68 , y efectivamente, los divisores de 68 son:


1  2  4  17  34  68 , y 68  d22  d33  22  43

3.30
Sabemos que el número de divisores de n 2  262 338 es 62  138  1  2457 .
Todos estos divisores se pueden agrupar por parejas a , b de forma que a  b  n 2
De forma que, o bien a  b  n o bien a  n o b  n .
Así pues, hay
62  138  1  1  1228 factores de n 2 menores que n.
2
De entre estos 1228 factores, hay 31  119  1  1  639 que son divisores de n, luego el
número buscado es 1228  639  589 .

3.31
ab
 a  b   nab  a 2  b 2  2ab  nab
1 1 n n
   
2

a b ab ab ab
 a  (n  2)ab  b  0
2 2

Interpretando esta última ecuación como una ecuación de segundo grado en la incógnita a ,
tenemos:

a
(n  2)b  (n  2)b 2  4  1  b 2 
(n  2)b  b (n  2) 2  4

2 2

b
2
n  2  n 2  4n 
De aquí se desprende que n2  4n debe ser un cuadrado. Esto es cierto para n  4 .
Si n  4 , n2  4n no puede ser un cuadrado. En efecto, por un lado:
n2  4n  n2  4n  2  (n  2)2

Y por otro lado:


9
n 2  4n  (n  3) 2  n 2  6n  9  4n  6n  9  2n  9  n 
2

Así pues, para todo n  4 , n2  4n está siempre entre (n  3)2 y (n  2)2 , y por tanto no puede
ser un cuadrado.

Para n  3  n2  4n  3 , n  2  n2  4n  4 , n  1  n2  4n  3 , son valores


negativos, y por tanto no son cuadrados.

Así pues, la única solución es n  4 , y para esta solución tenemos


a  4  2  0  b
b
2

Efectivamente, para b  a y n  4 se cumple la igualdad del enunciado:


1 1 2 4
  
a a a aa

Fuente de esta solución: Problemas y Soluciones Volumen 1 (Francisco Javier García Capitán) , pág. 156

3.32
Sea x  BC . Aplicamos el Teorema de Stewart (ver GA/9.1.6) :
n 2 (n  2)  n  2  (n  2)  (n  1) 2 n  x 2  2 
(n  2)(n 2  2n)  n(n  1) 2  2 x 2 
2 x 2  (n  2)(n 2  2n)  n(n  1) 2  n(2n  3) (*)

Aquí vemos que n debe ser par, pues 2n  3 es impar. Sea n  2k . Luego
(*)  2 x 2  2k (4k  3)  x 2  k (4k  3)

De aquí se deduce que k (4k  3) debe ser un cuadrado perfecto, pero esto es imposible.
En efecto:
k (4k  3)  k  4k  (2k )2
pero por otro lado, 4k  1  3k y por tanto : (2k  1)2  4k 2  4k  1  4k 2  3k  k (4k  3)
Así pues k (4k  3) se encuentra entre dos enteros consecutivos: (2k ) 2 y (2k  1)2 , y por tanto
no puede ser entero.

3.33
Si escribimos
 los dígitos de n ,
n  abcd e

Entonces q  a b c y r  d e , y por tanto q  r es el entero que se forma
 

n  a b  d c  e  , y por el criterio de divisibilidad del 11, será divisible entre 11 si y solo si
a  c  e   b  d 

es divisible entre 11, que es precisamente el criterio de divisibilidad del 11 aplicado al número
n.
Así pues, serán todos los números entre 10000 y 99999 divisibles entre 11:
99999 / 11  9090  9090  909  8181 (B)

9999 / 11  909 

Observación: Una manera más elegante de llegar a este resultado es observar que
n  100q  r  99q  q  r , luego q  r será divisible entre 11 si y solo si lo es n .

3.34
 
Utilizamos la desigualdad 2 a 2  b2  a  b (ver DE Tema 1) para obtener 2n3  n4 , es
2

decir, n  2 .
Para n  0 , tenemos la solución a  b  0 .
Para n  1 , tenemos la solución a  1 , b  0 .
Para n  2 , tenemos la solución a  b  2 .
Así pues, las soluciones son tres: n  0 , n  1 y n  2 .

Fuente de la solución: Number Theory Structures, Examples, and Problems (Titu Andreescu, 2009) , pág 47.

3.35
n(n  1)  n(n  1) 
Sabemos que 1  2  3  ...  n  . Sea m   
2  2 
Entonces
n(n  1) n(n  1)
m  m  1  m2   m  1
2

2 2

Y por tanto
n(n  1)
 m2  m  1  m2  2m  1
2

2
Luego
n(n  1) n(n  1)
 m 2  2m  2  2n(n  1)  2n 2  2n  2n  1
2 2

En donde hemos tenido en cuenta que n  3  2n2  2n  4n2  4n  1  (2n  1)2

Y puesto que cualquier número k  2n  1 se puede escribir como suma de dos números de
1, 2 ,..., n , hemos acabado el problema.
Fuente de esta solución: Number Theory Structures, Examples, and Problems (Titu Andreescu, 2009) pág. 48

3.36
Supongamos que a  b  c  0 . Entonces:
abc  ab  bc  ac  ab  ab  ab  3ab  abc  3ab  c  3

Si c  1 tenemos la desigualdad
ab  b  a  ab  b  a  0
que es siempre cierta.

Si c  2 tenemos la desigualdad
ab  2b  2a  2ab  2b  2a  ab  2(b  a)  ab

Luego, puesto que a  b


4a  2(b  a)  ab  4a  ab  4  b

Si b  3 queda la desigualdad
c  2, b  3  3a  6  2a  6a  5a  6  6a  6  a

Si b  2 queda la desigualdad
c  2, b  2  2a  4  2a  4a  4a  4  4a  4  0

Luego las soluciones son:


c  1, a, b  IN , c  2 , b  3, 3  a  5  , c  2, b  2, a  IN
y, puesto que la expresión es simétrica, todas las permutaciones posibles de estos elementos.

4.4
Serán los mismos puntos que en el extremo entre ( 0 , 0 ) y ( 48  3, 281  17 )  ( 45 , 264 ) y ya
sabemos que son en total: mcd ( 45 , 274 )  1  3  1  4 .
Concretamente:
45  ( 3 ,17 )
 15  
3  ( 3  15 ,17  88 )  (18 ,105 )

264 (18  15 ,105  88 )  ( 33 ,193 )
 88 
3  ( 33  15 ,193  88 )  ( 48 ,281)

4.5
( n  k , n) | n  k 
  ( n  k , n) | n  k  n  k
( n  k , n) | n 

4.10
El problema equivale a demostrar que 21n  4 , 14n  3  1, y esto, por el TDB, es una
consecuencia directa de encontrar una combinación lineal de ambas expresiones igual a 1:

 2(21n  4)  3(14n  3)  42n  8  42n  9  1

4.11
an  100  n2 , an 1  100  (n  1)2  100  n2  2n  1  an  2n  1
Luego dn  an , an 1   an , an  2n  1   an , 2n  1 
Y también dn | (100  n2 )  n(2n  1)  200  n
y por tanto dn | 2200  n  (2n  1)  401.
Así pues, d n | 401 . Y este valor se alcanza en algún n. Por ejemplo, para n  200 ,
a200  100  n2  100  2002  100  4  1002  1001  4  100  100  401
a201  a200  2  200  1  100  401  401  401  101
y por tanto d 200  401.

4.15
En primer lugar, podemos suponer m  n  k con k  1 . Debemos demostrar que

(n  k , n)  (n  1  k , n  1)  (n  2  k , n  2)  2k  1

Vamos a utilizar la propiedad (n  k , n) | k , independientemente del valor de n , demostrada en


el problema 4.2.

Luego
( n  k , n) | k  k  a  ( n  k , n)
(n  1  k , n  1) | k  k  b  (n  1  k , n  1)
(n  2  k , n  2) | k  k  c  (n  2  k , n  2)

k  a  ( n  k , n) 
  k | an
( n  k , n) | n 

k  b  (n  1  k , n  1)
  k | b(n  1)
(n  1  k , n  1) | n  1 

k
Puesto que (n, n  1)  1 , de aquí deducimos que k | ab , y por tanto k  ab  b
a

k
De la misma forma deducimos que k | bc , y por tanto k  bc  b
c

Así pues,
k k k k k
(n  k , n)  (n  1  k , n  1)  (n  2  k , n  2)     b   b  2b 
a b c b b

k
Queda por demostrar que 2b   2k  1
b
k
2b   2k  1  2b 2  k  2kb  b  2b 2  k  2kb  b  0  (b  k )(2b  1)  0
b

Pero b | k  b  k  0 , y por otro lado b  1  2b  1  0 , con lo que acabamos la


demostración.

Fuente de la solución: Solución oficial.

5.1.6

17 | 2 x  3 y 
  17 | 15 x  3 y  35 x  y  , puesto que 17 | 3 , necesariamente 17 | 5x  y
17 | 17 x 
17 | 2 x  3 y 
  17 | 2 x  14 y  2x  7 y  , y puesto que 17 | 2 , necesariamente 17 | x  7 y
17 | 17y 

17 | 5 x  y  17 | 25 x  y   10 x  2 y 
  17 | 10 x  2 y  ( x  7 y)  9 x  5 y
17 | x  7 y 

tal y como queríamos ver.



17 | 9 x  5 y 
  17 | 8 x  5 y
17 | 17 x 
17 | 9 x  5 y 
  17 | 12 y - 9 x  3(4 y  3x)  17 | 4 y - 3x  17 | 24 y - 3x   8 y  6 x
17 | 17 y 

17 | 8 x  5 y 
  17 | 8 y  6 x  8 x  5 y   8 y  6 x  8 x  5 y  3 y  2 x
17 | 8 y  6 x
tal y como queríamos ver.

5.2.3
2020  240520 , y por tanto los enteros m y n deben ser de la forma m  2a5b y n  2c5d

Luego
 
2
m 2 n  2020  2a5b 2c5d  240520  22 a52b 2c5d  240520 
2a  c  40
22 a  c52b  d  240520  
2b  d  20

La ecuación 2a  c  40 tiene las siguientes soluciones:


a  0, c  40 , a  1, c  38 , a  2, c  36,..., a  20, c  0, hay 21 en total.
La ecuación 2b  d  20 tiene las siguientes soluciones:
b  0, d  20 , b  1, d  18 , b  2, d  16,..., b  10, d  0, hay 11 en total.
Luego el total de parejas será 2111  231.

5.2.4
Aplicamos las propiedades básicas de los logaritmos.
C  A log 200 5  B log 200 2  log 200 5 A  log 200 2 B  log 200 5 A  2 B 
200C  5 A  2 B
Y ahora aplicamos el TFA:
2C  A

200C  52  23 
C
 52C  23C  5 A  2 B  
3C  B
Por hipótesis, 1  ( A, B, C)  ( 2C , 3C , C )  C ( 2 , 3,1)  C  C  1
Y por tanto: A  2C  2 , B  3C  3 , y C  1 , y finalmente: A  B  C  2  3  1  6 .

5.2.5
28  211  2n  2304  2n  482  2n  k 2  2n  k 2  482  k  48k  48 .
Luego, aplicando el TFA, k  48  2a y k  48  2b , con a  b  k .
Pero entonces, restando las dos igualdades anteriores:

25  3  96  2a  2b  2b 2a b  1 
Y , de nuevo por el TFA, b  5 , y 3  2a b  1  4  22  2a b  a  b  2
Con lo que llegamos a  2  b  2  5  7 , es decir, k  7  5  12 .

5.2.6
1998  2  3  3  3  37
Tenemos 1998  2  3  3  3 37  54  37 y 54  37  17
y es fácil ir comprobando, una a una, que cualquier otra combinación posible da como
resultado una diferencia mayor. Por ejemplo:
1998  2  3  3 3  37  27 104 y 104  27  77

5.2.7
Buscamos el menor número n tal que (n  1)  n  (n  1)  3n sea un cuadrado y sea un cubo
perfecto, es decir:

3n  a 2 , 3n  b3 para cierto ciertos a, b  IN


3n  a 2  3 | a 2  3 | a . Así pues, la descomposición factorial de a y a 2 :
a  p1a1 3a2 p3a3 ... pkak  a 2  p12a1 32a2 p32a3 ... pk2ak

También 3n  b3  3 | b3  3 | b . Así pues, la descomposición factorial de b y b 3 :


b  p1b1 3b2 p3b3 ... pkbk  b3  p13b1 33b2 p33b3 ... pk3bk

Y, puesto que a 2  b3 ,
p12a1 32a2 p32a3 ... pk2ak  p13b1 33b2 p33b3 ... pk3bk

Y por tanto 32a2  33b2  2a2  3b2 . El número 6  mcm(2,3) es el exponente más pequeño
que sea múltiplo de 2 y de 3, luego, el candidato mínimo que cumpla las condiciones
anteriores será cuando tomemos solamente el factor 3 y con su exponente más pequeño:
3n  36  n  35  243 .
Así pues, los números buscados son 242, 243 y 244.

242  243  244  729  36  32  33    


3 2

5.2.8
Primera versión.
Queremos determinar números a  0 y n  3 tales que
2009  a1  a2 ,...  an  a  1  a  2  a  3  ...  a  n  n  a  (1  2  3  ...  n) 
n(n  1)
 na 
2

n(n  1)  n  1   2a  n  1  n
Es decir: 2009  7 2  41  n  a   n a    n   2a  n  1
2  2   2  2
Estudiemos todas las posibilidades:
n 1 7 1
n7,a  7  41  a  7  41   283
2 2
n 1 49  1
n  7 2  49 , a   41  a  41   16
2 2
n 1 41  1
n  41 , a   7 2  49  a  49   28
2 2
n 1 287  1
n  41  7  287 , a  7a 7  137 (*)
2 2
n 1 11
n 1, a   2009  a  2009   2008 (*)
2 2
n 1 2009  1
n  2009 , a  1 a 1  1004 (*)
2 2
n 7  41  1  14
 7  n  14 , 2a  n  1  2a  14  1  7  41  a   136
2 2
n 49  1  82
 41  n  82 , 2a  n  1  2a  82  1  49  a   17 (*)
2 2
n 41  1  98
 7 2  49  n  98 , 2a  n  1  2a  98  1  41  a   29 (*)
2 2
n 2009  1  2
 1  n  2 , 2a  n  1  2a  2  1  2009  a   1003 (*)
2 2
n 1  1  4018
 2009  n  4018 , 2a  n  1  2a  4018  1  1  a   2009 (*)
2 2

Las soluciones marcadas con (*) quedan descartadas por no cumplir las condiciones del
enunciado, luego las soluciones del problema son cuatro:

a  283 , n  7 ; a  16 , n  49 ; a  28 , n  41 ; a  136 , n  14

Segunda versión.
En primer lugar, acotamos los posibles valores de n:
 n 1  n 1
2009  n a    n 
 2   2 
 n 1
La ecuación 2009  n  tiene soluciones n  62.8 , luego n  62 .
 2 

 n  1   2a  n  1  n
2009  7  7  41  n a    n   2a  n  1
 2   2  2
Si n es impar, n  7 , n  41 o n  7 2  49 , pues cualquier otro divisor impar de 2009 es mayor
que 7  41  287 , y se obtienen las sucesiones con 283, 28 y 16.

Si n es par, entonces n / 2 es un divisor de 2009, y por tanto n  14 y a  136 , porque


cualquier otra combinación daría un n  62 .

Fuente de la segunda versión: Solución oficial.

5.2.9
2001  3  23  29  1 3  23  29 , luego el mayor valor posible aparecerá multiplicando los dos
factores más grandes: 23  29  667
Luego será tomando I  1, M  3, N  667  I  M  O  667  3  1  671

5.2.10
En primer lugar, aplicamos las propiedades fundamentales de los logaritmos:
A log 200 5  B log 200 2  C 
log 200 5 A  log 200 2 B  C 
 
log 200 5 A 2 B  C 
5 A  52C  A  2C

5 A 2 B  200C  2352 
C
 2 3C 5 2 C   B
2  23C  B  3C

En el último paso hemos aplicado el TFA, es decir, la unicidad de la factorización de todo


entero.
Está claro que C | A y C | B , luego para que A y B sean coprimos es necesario que C  1 , y
por tanto A  2, B  3 , y A  B  C  2  3  1  6

5.2.11
Por el TFA, puesto que 2310  2  3  5  7  11 , cinco números primos, el número de formas de
escribir 2310  2  3  5  7  11 como producto de tres números será el número de formas que
tenemos de repartir los números  2 , 3, 5 , 7 ,11 en tres grupos, sin repeticiones y sin importar el
orden.
Puesto que uno de los tres elementos puede ser 1 (como mucho uno de ellos, pues suponemos
que a, b, c son distintos), también debemos considerar el caso que uno de los tres grupos esté
vacío, por ejemplo a  1 , b  2  3 , c  5  7  11 .

Organizamos los casos en función del número de elementos de cada grupo:


5!
i) (3,1,1)  C53   10 posibilidades.
3!(5  3)!
5!
ii) (0,1,4)  C54   5 posibilidades.
4!(5  4)!
5!
iii) (0,2,3)  C53   10 posibilidades.
3!(5  3)!
iv) (1,2,2)  5 posibilidades para el primer grupo, y el número de posibilidades de repartir 4
1  4
elementos en dos grupos: 5     5  3  15 posibilidades.
2  2

Luego, en total, hay 10  5  10  15  40 posibilidades.

5.2.12
Primera versión.
  
16 p  1  n3  16 p  n3  1  n  1 n2  n  1 luego 24 p  n  1 n2  n  1 
De todas las posibilidades que podemos plantear, si n  1 es 1 , 2 , 4 , 8 entonces
n2  n  1  n(n  1)  1 será múltiplo de 2, lo cual es absurdo pues es impar (el producto de dos
números consecutivos es par, y si le sumamos 1 será impar).

Veamos la opción n  1  24 , p  n2  n  1
n  1  24  n  17  p  172  17  1  307 , que es primo, y 16 p  1  4913  173

Segunda versión.
De 16 p  1  n3 se deduce que n tiene que ser impar. Luego n  2a  1
 
16 p  1  (2a  1)3  8a3  12a 2  6a  1  16 p  2a 4a 2  6a  3 

8 p  a 4a  6a  32

4a  6a  3 es impar, luego en su factorización no puede tener ninguna potencia de 2.
2

La única opción válida es que a  8 , y por tanto p  4a 2  6a  3  4  82  6  8  3  307 .

Tercera versión.(Utilizando congruencias, que se estudiarán en el Tema 6)


De 16 p  1  n3 se deduce que n  1 (mod 4) , y por tanto n  4a  1 , luego
16 p  1  64a 3  48a 2  12a  1 
16 p  64a 3  48a 2  12a 
 
4 p  a 16a 2  12a  3  4 | a  a  4b 

4 p  4b 16  16b  12  4b  3 
2


p  b 256b  48b  3 2

Y, puesto que p es primo, b  1  p  256  12  48  1  3  307

5.2.13
702  am  r 
 787  bm  702  am 85  5 17  m(b  a) 
787  bm  r r  702  am   
 855  cm  702  am  153  32 17  m(c  a)
855  cm  r 
De donde deducimos que m  17 , y por tanto:
702  41  17  5 

787  46  17  5  r  5
855  50  17  5 
412  dn  s 
 722  en  412  dn 310  2  5  31  n(e  d )
722  en  s   s  412  dn   
 815  fn  412  dn 403  13  31  n( f  d ) 
815  fn  s 
De donde deducimos que n  31, y por tanto:
412  13  31  9 

722  23  31  9  s  9
815  26  31  9

Y, finalmente, m  n  r  s  17  31  5  9  62

5.2.14
El polinomio se podrá expresar de la forma f ( x)  a( x  b)( x  c) , con a, b, c enteros.
Sabemos que 2  3  5  67  2010  f (0)  a(0  b)(0  c)  a b c

Primera parte: a, b, c son positivos.


Nuestro problema se reduce a contar el número de combinaciones posibles de 4 elementos
agrupados en tres cajas a, b, c , teniendo en cuenta que alguna (pero no las tres a la vez) puede
estar vacía (es decir, conteniendo el factor 1). Las cajas b y c son indistinguibles.
a) 4,0,0 a  2010 , b  1, c  1 1 caso
b) 3,1,0 a  p1 p2 p3 , b  p4 , c  1 : 4 casos.
 4
c) 2,1,1 a  p1 p2 , b  p3 , c  p4  : C42    
4!
 6 casos
 2  2!(4  2)!
 4
d) 2,2,0 a  p1 p2 , b  p3 p4 , c  1 : C42    
4!
 6 casos
 2  2!(4  2)!
e) 1,0,3 a  p4 , b  1 , c  p1 p2 p3  : 4 casos.

 4
f) 1,2,1 a  p1 , b  p2 p3 , c  p4  : C42    
4!
 6  6  2  12
 2  2!(4  2)!

g) 0,4,0 a  1 , b  2010 , c  1 : 1 caso


h) 0,3,1 a  1 , b  p1 p2 p3 , c  p4  : 4 casos.
 4
i) 0,2,2 a  1 , b  p1 p2 , c  p3 p4  : C42    
4!
 6 casos/2 =3
 2  2!(4  2)!

Total: 1+4+6+6+4+12+1+4+3 = 41 casos.

Segunda parte: a, b, c con signo.


Dos de estos números pueden ser negativos, no los tres, luego el signo multiplica por cuatro las
posibilidades:
a  0, b  0, c  0 2( x  3)( x  5  67)  2 x 2  676 x  2010
a  0, b  0, c  0 2( x  3)( x  5  67)  2 x 2  676 x  2010
por ejemplo:
a  0, b  0, c  0 (2)( x  3)( x  5  67)  2 x 2  664 x  2010
a  0, b  0, c  0 (2)( x  3)( x  5  67)  2 x 2  664 x  2010

Excepto en el caso a) que solo tiene tres:


2010( x  1)( x  1)  2010 x 2  4020 x  2010
2010( x  1)( x  1)  2010 x 2  4020 x  2010
(2010)( x  1)( x  1)  2010 x 2  2010
(2010)( x  1)( x  1)  2010 x 2  2010

luego el total es 4  40  3  163 polinomios diferentes.

5.2.15
 p p 2  4  444 p
x 2  px  444 p  0  x 
2
Las soluciones serán enteras si y solo si p 2  4  444 p es entero, es decir, si p 2  4  444 p es
un cuadrado perfecto.

 
Si p 2  4  444 p  p p  24  3  37 es un cuadrado perfecto entonces
 
p | p  24  3  37  p | 24  3  37  p  2 , 3, 37 .
Comprobemos estos posibles valores:
p  2  p 2  4  444 p  3556 no es un cuadrado.
p  3  p 2  4  444 p  5335 no es un cuadrado.
p  37  p 2  4  444 p  67081  2592 es un cuadrado, y las soluciones de la ecuación son 111
y -148.

5.2.16
Observamos que todo número natural se puede escribir de forma única de la forma
a  2k b , con b impar y 0  k .
Observamos también que si el número impar b es el mismo, digamos a1  2k1 b y a2  2k2 b ,
Entonces
k1  k2  a1  a2 , k1  k2  a1 | a2 , k1  k2  a2 | a1

y en todo caso uno divide al otro.

Entre 1 y 2n hay n números impares, por lo tanto, si asignamos a cada número xi  2ki bi de
nuestro conjunto su valor impar asociado bi , al haber n  1 elementos en nuestro conjunto,
aplicando el Principio del casillero, al menos dos elementos tendrán asociados el mismo impar,
y por tanto, como hemos visto al principio, uno será divisor del otro.

5.2.17
k será divisor de 1212=224·312, luego será de la forma k=2a3b, con 0  a  24 y 0  b  12 .
1212=224·312 es el menor múltiplo de 66=2636 y 88=224 y k=2a3b, luego 0  b  6 .
12=max{6,b} y de esto deducimos que b=12.
Hay 25 valores diferentes para a, luego hay un total de 25 valores diferentes para k.

5.2.18
[a,b]=1000=2353
[b,c]=2000=2453
[c,a]=2000=2453
Luego a  2i1 3 j1 , b  2i2 3 j2 , c  2i3 3 j3 .
Vamos a estudiar, en primer lugar, los exponentes del 3:
Está claro que j1 , j2 , j3  3 , y que al menos dos elementos de j1, j2 y j3 tienen que ser iguales a
3. No hay más limitaciones.
Las opciones son (0,3,3) , (1,3,3) , (2,3,3) , (3,3,3) , (3,0,3) , (3,1,3) , (3,2,3) , (3,3,0) , (3,3,1) ,
(3,3,2) . Hay 10 en total.
Veamos ahora los exponentes del 2:
i1 , i2  3 y entre i1, i2 uno de los dos tiene que ser igual a 3
i2 , i3  4 y entre i2, i3 uno de los dos tiene que ser igual a 4.
i1 , i3  4 y entre i1, i3 uno de los dos tiene que ser igual a 4.
Si i1=3, entonces i3=4, y la única condición para i2 es i2  3 .
Si i2=3, entonces i3=4, y la única condición sobre i1 es i1<=3.
Los casos posibles son 16:
(i1,i2)=(3,0), (3,1), (3,2), (3,3), (0,3), (1,3), (2,3)
Hay 7 casos diferentes.
Luego el total es 7·10=70 triplas (a,b,c) ordenadas diferentes.

5.2.19
y 2  3x 2 y 2  30 x 2  517  y 2  3x 2 y 2  30 x 2  517 
y 2
    
 10 3x 2  1  10  517  y 2  10 3x 2  1  507  3 132

Casos:

 y  10  3  y  13
2 2

 2 imposible, pues 13 no es un cuadrado.



3 x  1  13 2


 y  10  13  y  23
2 2

 2 imposible, pues 23 no es un cuadrado.



3 x  1  3  13

 y  10  13  y  13  10  179
2 2 2 2

 2 imposible, pues 179 no es un cuadrado.



3 x  1  3  13

 y  10  3 13  y  39  10  49  y  7
2 2

 2 , en todo caso, 3x 2 y 2  12  49  588



3x  1  13  3x  12  x  4  x  2
2 2


 y  10  1  y  11
2 2

 2 , imposible, pues 11 no es un cuadrado.



3 x  1  517

 y  10  507  y  517
2 2

 2 , imposible, pues 517 no es un cuadrado.



3 x  1  517
La única solución posible es 588

5.2.20
Primera versión.
16 p  1 es impar. El cubo de un par es par, luego a es impar: a  2b  1
16 p  1  a 3  (2b  1)3  8b3  12b 2  6b  1 
  
16 p  2b 4b 2  6b  3  8 p  b 4b 2  6b  3 
Por un lado, p es primo, luego impar, y por otro, 4b2  6b  3 es impar.
Luego b  8 y p  4b2  6b  3  307 .

Segunda versión.
Observamos que el cubo de un par es par, luego a debe ser impar, pues 16 p  1 es impar.

16 p  1  a3  24 p  a3  1  (a  1) a 2  a  1 
a  1 es par, a 2  a  1  a(a  1)  1 es impar, luego a  1 debe ser múltiple de 16.
a3  1
Pero p  y por tanto, cualquier otro múltiplo que no sea 16 hará que p no sea primo.
16
Así pues, a  1  16  a  17  p  307 .

Fuente de estas soluciones: artofproblemsolving.com

5.2.21
m  ( a, b)
a  k1m
m | a, b  
b  k2 m
(a, b)a, b  ab  k1 m k2 m  k1 k2 m 2  ma, b  k1 k2 m 2  a, b  k1 k2 m

Y la ecuación queda
m  k1 k2 m  k1 m  k2 m  6  m  k1 k2 m  k1 m  k2 m  6 
m1  k1 k2  k1  k2   6  mk1  1k2  1  6  1  2  3

Las opciones son:


m  1, k1  1  2, k2  1  3  a  3, b  4  (a, b)  1 , a, b  12
m  2, k1  1  1, k2  1  3  a  4, b  8  (a, b)  4 , a, b  8 
m  3, k1  1  2, k2  1  1  a  9, b  6  (a, b)  3 , a, b  18
m  6, k1  1  1, k2  1  1  a  12, b  12  (a, b)  12 , a, b  12 
m  1, k1  1  6, k2  1  1  a  7, b  2  (a, b)  1 , a, b  14

Las opciones marcadas con  no cumplen la ecuación del enunciado, luego las soluciones son
a, b  3,4,2,7,6,9.
5.2.24
Sabemos que a, b(a, b)  a  b , luego en el caso de ser coprimos a, b  a  b , y por tanto
a, b serán las soluciones de la ecuación de segundo grado
x 2  (a  b) x  a, b  0

Si no son coprimos, realizamos el cambio de variable


a  ( a, b) a '
para ciertos a' , b' tales que (a, b)  1
b  (a, b)b'

Y podemos aplicar el método anterior, para finalmente deshacer el cambio de variable.

Luego necesitamos conocer el máximo común múltiplo de a, b . Para ello basta tener en cuenta
que (a, b)   a  b , a, b  .

En el caso concreto del enunciado determinaremos el mínimo común múltiplo mediante el


algoritmo de Euclides:

985928  3972  248  872


985928,3972  3972  248  872,3972  872,3972
3972  4  872  484
3972,872  4  872  484,872  484,872
872  1  484  388
872,484  1 484  388,484  388,484
484  1  388  96
484,388  1 388  96,388  96,388
388  4  96  4
388,96  4  96  4,96  4,96  4
Así pues, 985928,3972  4
985928  246482  4
3972  993  4

Y tenemos que resolver la ecuación x 2  993x  246482  0


  9932  4  246482  986049  985928  121  112
993  11 502
x 
2 491
Y por tanto los números buscados son 502  4  2008 y 491  4  1964

5.2.25
n  50  a 2 

2
 a 2  100  b 2  100  b 2  a 2  (a  b)(a  b)
n  50  b  
2  5  100  (a  b)(a  b)
2 2

Veamos todas las posibilidades:


a  b 1 
  2a  101 no es entero
a  b  100
ab  2 
  2a  52  a  26, b  2  26  0 negativo
a  b  50
ab  4 
  2a  29 no es entero
a  b  25
ab 5 
  2a  25 no es entero
a  b  20
a  b  10
  2a  20  a  10, b  10  a  0
a  b  10
a  b  20
  2a  25 no es entero
ab 5 
a  b  25
  2a  29 no es entero
ab  4 
a  b  50
  2a  52  a  26  b  50  26  24
a b  2 
a  b  100
  2a  101 no es entero
a b 1 

Así pues, las dos únicas posibilidades son:


a  10, b  0, n  102  50  50 , en efecto, 102  50  50 , 02  50  50 .
a  26, b  24, n  262  50  626 , en efecto, 242  50  626 , 262  50  626 .

Las soluciones son 626 y 50

5.3.4
 
Puesto que n3  1  n  1 n2  n  1 , y teniendo en cuenta que n2  n  1  1 ,
n3  1 primo implica que n  1  1  n  2  n2  1  7 . El único primo de esta forma es el 7.

5.3.5

N  2903n  803n  464n  261n  2903n  803n  464n  261n 
Aplicando la proposición anterior:
7  300  2100  2903  803 divide a 2903n  803n
7  29  203  464  261 divide a 464n  261n
Y por tanto 7 divide a N.


Por otro lado, 2903n  803n  464n  261n  2903n  464n  803n  261n 
271 9  2439  2903  464 divide a 2903n  464n
271  2  542  803  261 divide a 803n  261n
Y por tanto 271 divide a N.

Finalmente, puesto que (7,271)  1 , tendremos que 1897  7  271 dividirá a N.

5.3.6
Primera versión.
Está claro que para n  1 , n4  4  5 que es primo.
n 4  4  n 4  4n 2  4  4n 2  (n 2  2) 2  (2n)2  (n 2  2  2n)(n 2  2  2n) 
 
 (n  1)2  1 (n  1)2  1 
Y si n  1 cada factor es mayor que 1, luego n4  4 no puede ser primo.

Segunda versión.
Está claro que para n  1 , n4  4  5 que es primo.

Aplicamos la identidad algebraica x 4  4 y 4  x 2  2 xy  2 y 2 x 2  2 xy  2 y 2 . 
En nuestro caso:

n4  4  n4  4  14  n2  2n  2 n2  2n  2  
Puesto que n2  2n  2  1 , debemos suponer que
n2  2n  2  1  0  n2  2n  1  (n  1)2  n  1

5.3.7
Si n  1 , n4  4n  1  4  5 que es primo. Supongamos que n  1 .
Está claro que si n es par no será primo:
n  2k , k  1  n4  4n  (2k )4  42k  16k 4  16k es un múltiplo de 16.

Supongamos que n es impar.


n 4  4n  n 2   2n   2n 2 2n  2n 2 2n  n 2  2n   2n 2 2n 
2 2 2

 n 2  2n   n 2 2n 1  n 2  2n   2( n 1) / 2 n  
2 2 2

 n 2  2n  2( n 1) / 2 n n 2  2n  2( n 1) / 2 n 

n 1
Esta última igualdad está bien construida pues estamos suponiendo que n es par, luego es
2
entero.

Está claro que n2  2n  2( n 1) / 2 n  1.


(Faltaría demostrar que n2  2n  2( n 1) / 2 n  1 si n  3 ).

5.3.8
Aplicamos el Teorema del Binomio:
n  1n  n n    n n1    n n2  ...    n1  1  n n    n n1    n n2  ...  n 2  1
n n n n n
1  2 1 1  2
 n  n   n  n 
Luego n  1  1  n n    n n 1    n n  2  ...  n 2  n 2  n n  2    n n 3    n n  4  ...  1 ,
n

1  2  1  2 

5.3.9
Teniendo en cuenta que 1993 es impar, basta agrupar por parejas:
1001  1  1000 | 11993  10001993
1001  2  999 | 21993  9991993
1001  3  998 | 31993  9981993

Por lo tanto, 1001 dividirá al total.

5.3.10
 
De la igualdad x 2n 1  y 2n 1  ( x  y) x 2n  x 2n 1 y  x 2n  2 y 2  ...  y 2n
deducimos que si n es impar, a  b | a n  bn . En nuestro caso n  47 impar, luego:
Luego 7  1  6 | 147  647 , 7  2  5 | 247  547 y 7  3  4 | 347  447 , y por tanto
7 dividirá a la suma de todos, tal y como queríamos ver.

5.3.11
Supongamos que n  ab , es decir, n  10a  b . Entonces
10a  b2  10b  a 2  100a 2  20ab  b2  100b2  20ab  a 2  
 
 100(a 2  b 2 )  b 2  a 2  100(a 2  b 2 )  a 2  b 2  99(a 2  b 2 ) 
99(a  b)(a`b)
Que es un número divisible entre 9, entre 11, entre la suma de los dígitos y entre la diferencia
de los dígitos, luego la respuesta es (B).

5.3.12
Observamos que
     
n5  n 4  1  n5  n 4  n3  n3  n 2  n  n 2  n  1  n3 n 2  n  1  n n 2  n  1  n 2  n  1 
 
 n  n 1 n  n 1
2 3

Y por lo tanto se puede siempre escribir como producto de dos enteros mayores que 1. Luego
no es primo.

5.3.13
Observamos que
  2
a 4  4b 4  a 4  4b 4  4a 2b 2  4a 2b 2  a 4  2b 2  4a 2b 2 
 
 a 2  2b 2  2ab a 2  2b 2  2ab 
y que a 2  2ab  2b2  a 2  2ab  2b2 , luego será primo si y solo si
a 2  2ab  2b 2  1

 4

a  4b  a  2ab  2b
4 2 2

1  a 2  2ab  2b2  (a  b)2  b2  2b2  (a  b)2  b2

Las posibilidades son:


( a  b) 2  1 
  a  1  a  4b  1 que no se considera primo.
4 4

b  0  b  0
2

( a  b) 2  0 

  (a  1)  0  a  1  0  a  1  a  4b  1  4  5
2 4 4

b  1  b  1
2

5.3.14
a  2  b  88 y no es primo.
a  3  b  87 y no es primo.
a  5  b  85 y no es primo.
a  7  b  83 y sí es primo . (D)

5.4.1
Aplicando la identidad “Suma por diferencia”:
x 2  y 2  ( x  y)( x  y)  23

Y, puesto que 23 es primo, sus únicos divisores son 1 y 23. Las posibilidades son, por tanto:
x  y  1  x  y  23
  x  12, y  11   x  12, y  11
 x  y  23 x  y  1
 x  y  1  x  y  23
  x  12, y  11   x  12, y  11
 x  y  23  x  y  1

La única solución aceptable con enteros positivos es x  12, y  11 .

5.4.2
Nos encontramos ante la ecuación diofántica
pq  ( p  q)  a

Donde a es uno de los cinco posibles candidatos del enunciado.

Aplicando el SFFT,
a  pq  ( p  q)  pq  p  q  ( p  1)(q  1)  1  ( p  1)(q  1)  a  1

Los números primos entre 4 y 18 son: 5, 7, 11, 13, 17

y probando distintas combinaciones vemos que la única combinación aceptable es


(13  1)(11  1)  12  10  120  119  1
Y la respuesta correcta es (C)

5.4.3
Realizando el cambio de variable a  m2 , b  n2 convertimos la ecuación del enunciado en
a  3ab  30b  517

para ciertos cuadrados a, b que queremos determinar.


a  3ab  30b  517  517  a  3ab  30b  (a  10)(3b  1)  10 
(a  10)(3b  1)  507  3  132

a  10  1  a  11
a  10  3  a  13
a  10  13  a  23
a  10  3  13  39  a  49  7 2
a  10  13  13  169  a  169  10  179
a  10  507  a  517
La única opción con la que obtenemos un cuadrado es a  49 , y por tanto
3b  1  13  b  4  22 .

Así pues, 3m2n2  3ab  3  49  4  588

5.4.4
Completamos cuadrados:
y 2  x 2  84 x  2008  x 2  2  42 x  422  244  ( x  42) 2  244 
y 2  ( x  42) 2  244 
( y  x  42)( y  x  42)  244  22  61

Vamos probando las posibles factorizaciones de 244:


y  x  42  4 
  x  27 / 2, y  65 / 2
y  x  42  61
y  x  42  2 
  x  18, y  62
y  x  42  2  61

Puesto que el enunciado dice que solo hay una solución con enteros positivos, debe ser esta, y
no hace falta seguir probando. La respuesta correcta es 18  62  80 .

5.4.5
1812  x  1  x 3  x 3  3x 2  3x  1  x 3  3x 2  3x  1 
3

1812  1  3x 2  3x  3xx  1 
32760  3xx  1 
10920  xx  1

Puesto que 1002  10000 , vamos probando con números consecutivos superiores a 100 hasta
encontrar la solución 10920  104  105 , así pues, la solución es 1812  1053  1043

Observación: Una manera mucho más elegante de acabar este problema sería
3xx  1  1812  1  (181  1)(181  1)  182  180  2  7  13  22  32  5 
 
xx  1  23  3  5  7  13  23  13  3  5  7   104  105

5.4.6
2 1 1 1 x  y 1
     5( x  y  1)  2 xy  5 x  5 y  2 xy  5 
5 x y xy xy
 5  5 25  15
 x   2 y  5   5   x   2 y  5  5 
25
 
 2 2  2 2 2
 5
2 x   2 y  5  15  5  2 x  2 y  5  15  3  5
 2

Vamos probando posibilidades:


5  2x  1  5  2x  3 
  x  2, y  5   x  1, y  0
 2 y  5  15  2 y  5  5
5  2x  5  5  2 x  15 
  x  0, y  1   x  5, y  2
 2 y  5  3  2 y  5  1
5  2 x  1  5  2 x  3 
  x  3, y  10   x  4, y  5
 2 y  5  15  2 y  5  5
5  2 x  5  5  2 x  15 
  x  5, y  4   x  10, y  3
 2 y  5  3  2 y  5  1

Las únicas soluciones aceptables son: x  3, y  10 ; x  4, y  5 ; x  5, y  4 ; x  10, y  3

6.1
Basta descomponer en factores:
n5  5n3  4n  n(n4  5n2  4)  n(n2  4)(n2  1)  (n  2)(n  1)n(n  1)(n  2) ,
y por tanto es el producto de 5 enteros consecutivos.
En cinco enteros consecutivos siempre encontraremos al menos un múltiplo de 2, un múltiplo
de 3, un múltiplo de 4 (diferente al primero) y un múltiplo de 5, por lo que su producto será
divisible entre 2  3  4  5  120 .

6.2
Sea k  n2  3n  5 . Supongamos que 121  112 | k  11 | k .
Observamos que k  (n  7)(n  4)  33 , y que 11 | 33 , luego 11 | (n  7)(n  4) y por tanto
11 | (n  7) o 11 | (n  4) , por ser 11 primo.
Pero 11  n  7  (n  4) , luego si 11 divide a uno, también dividirá al otro, y por tanto
121 | (n  7)(n  4) , pero 121 | 33 , y por tanto 121 | (n  7)(n  4)  33 , llegando a
contradicción.

6.3
34  34 4  34
5 4
  4 4

45  4  45 1  4  22
6 6
 
 4  225 1  4  245 1/ 2  4  25 1/ 2
56 1 6 6
 6

4

En donde hemos aprovechado que 56 es impar, luego 56  1 es par, y por tanto 56  1 / 2 es  


entero.

Así pues, hemos escrito 34  45 de la forma m4  4n 4 y hacemos la siguiente descomposición:


5 6

  2

m4  4n4  m2  2n2  4m2 n2  m2  2m n  2n2 m2  2m n  2n2  
El factor más pequeño es el de la izquierda, que cumple:
m2  2m n  2n 2  (m  n) 2  n 2  n 2  25 1  28008  24
6
  2002
 102002

6.4
Buscamos una combinación lineal  11a  2b   18a  5b que sea múltiple de 19.
11  18  19
Probamos con la combinación  11a  2b    18a  5b   19a  19b  
2  5  19
Y vemos que tiene solución entera:   13 , b  9
Luego  1311a  2b  918a  5b  19a  b que es divisible entre 19.
Luego 19 | 1311a  2b  19 | 918a  5b
Pero (19,13)  1 , y por tanto 19 | 1311a  2b  19 | 11a  2b
Y de la misma forma (19,9)  1 , y por tanto 19 | 918a  5b  19 | 18a  5b

6.5

Queremos ver que n2  n  1 , n2  2n  1 
n2  n  1  (1)(n2  2n)  n  1  (n  1)
a | n 2  n  1

Luego   a | (n  1)  a | n  1
a | n  2n 
2

a | n  n  1  n(n  1)  1
2
Pero   a | 1  a  1
a | n 1 
y por tanto no hay divisores comunes que no sean la unidad

6.6
Queremos determinar los enteros x para los cuales 2 x  1 | x3  3x  2

1 1 11  27
Realizando la división entera tenemos que x3  3x  2   x 2  x   
2 4 8 8

   
Luego 8 x3  3x  2  4 x 2  2 x  11 2 x  1  27

Luego
   
2 x  1 | x3  3x  2  2 x  1 | 8 x3  3x  2  4 x 2  2 x  11 2 x  1  27 
2 x  1  1  x  0
2 x  1  3  x  1, x  2

 2 x  1 | 27  33  
2 x  1  9  x  4, x  5
2 x  1  27  x  13, x  14

6.7
Supongamos que 16 p  1  n3 para cierto entero positivo n .
En primer lugar vemos que n debe ser impar pues lo es 16 p  1 .

16 p  1  n3  16 p  n3  1  n  1 n2  n  1 
Escribir 16 p como producto de un par a multiplicado por un impar b  1 solo es posible como
a  16 y b  p .

O si lo queremos ver de otra manera:


Puesto que n es impar, n  1 es par y n2  n  1 es impar, luego no puede tener ningún factor
2, y por tanto n  1  16k .
Luego
  
16 p  16k n 2  n  1  p  k n 2  n  1  
k  1 , p  n 2  n  1

 n  0 absurdo
 k  p , 1  n 2
 n  1  n ( n  1)  1  
 n  1 absurdo

En todo caso k  1 , p  n2  n  1  n  1  16  n  17 .
173  1 4913  1 4912
y p    307
16 16 16

6.8
Primera versión.
En este problema utilizaremos un resultado importante:
Si ab es un cuadrado perfecto, y a y b son coprimos, entonces a y b son cuadrados
perfectos.

Buscamos los k  0 para los cuales k 2  pk  k (k  p) sea un cuadrado perfecto.


a) Si k y p son coprimos, podemos aplicar el principio anterior, y deducir que
k y k  p deben ser cuadrados perfectos:
k  q2
r 2  k  p  q 2  p  p  q 2  r 2  (q  r )(q  r )

Y puesto que p es primo,


q  r 1 
    
p 1
 
 p  12
 2 q p 1 q k
q  r  p 2 4

b) Si k y p no son coprimos, es decir, p | k  k  ap


k 2  pk  a 2 p 2  pap  ap 2 (a  1)

Puesto que a y a  1 son consecutivos, son coprimos, y como su producto es un cuadrado


perfecto,podemos aplicar el lema anterior para deducir que a y a  1 son cuadrados perfectos.
Pero los únicos cuadrados consecutivos son 0 y 1, luego su producto es 0, y este valor está
descartado por el enunciado.

Fuente de la versión: Number Theory: Concepts and problems (Andreescu, Dospinescu, 2017) pág. 15

Segunda versión.
p p 2  4n 2
k  pk  n  k  pk  n  0  k 
2 2 2 2

2
Luego vemos que p  4n debe ser un cuadrado perfecto.
2 2

p 2  4n2  a 2  p 2  a 2  4n2  (a  2n)(a  2n)


Y puesto que p es primo y a  2n  a  2n , solo se pueden dar dos casos:
Primer caso:
a  2n  p 2  p2  1
  2a  p  1  a 
2

a  2n  1  2
p 2  2 p  1  p  1
2
Tomando la suma: k  
4 4
p  ( p  1) / 2 2 p  ( p 2  1) 2 p  p 2  1  ( p  1) 2
2
Tomando la resta: k    
2 4 4 4
Segundo caso:
a  2n  p 
  2a  2 p  a  p
a  2n  p 
 2p
p p k  p
p  4n  a  p  k 
2 2
 2
2 
k  0
El caso k  0 queda descartado por el enunciado.

Fuente de la versión: Soluciones oficiales (SE pág. 762)

6.9
24  23  3 , luego todo múltiple de 24 es de la forma 23  3  a , donde a tiene que contener al
menos un factor 2 para completar el 23 a exponente par y un factor 3 para completar el 3 a
exponente par y finalmente un cuadrado perfecto. Los números buscados son de la forma
n  24  32  b  144b con b un cuadrado perfecto, es decir, son números de la forma

n  223c 
2
con c 2  b

 2
 2
n  223c  106  103  223c  103  12c  1000
Puesto que 1000 / 12  83 , hay 83 números que cumplan la condición del enunciado.

6.10
Haciendo una división sintética de polinomios obtenemos
x2 36
f ( x)   x6
x6 x6

36
Que será entero si y solo si es entero, es decir, x  6 | 36  22  32
x6

x  6  1  x  5, x  6  2  x  4, x  6  4  x  2, x  6  3  x  3


x  6  9  x  3, x  6  6  x  0, x  6  12  x  6, x  6  18  x  12
x  6  36  x  30

Es fácil comprobar que, efectivamente, todos estos valores generan imágenes enteras en la
función.

6.11
a  a  a  b  b  b  b  10a  b  3a  4b  10a  b  3b  7a
Luego vemos que 7 divide a b, y puesto que b  10 , la única posibilidad es b  7 , y por tanto
3  7  7a  a  3  a  b  10 (C)

6.12
Puesto que se empaquetan igual número de manzanas, el número de cajas ha de ser un divisor
de 60.
1 manzana  60 cajas es imposible, pues tocaría a una pera por caja, y necesitamos cantidades
diferentes de peras.
2 manzanas  30 cajas es imposible, pues tendríamos, como mínimo:
30
30  31
1  2  3  ...  30   k   15  31  60 peras.
k 1 2
3 manzanas  20 cajas es imposible, pues tendríamos, como mínimo:
20
20  21
1  2  3  ...  20   k   10  21  210  60 peras.
k 1 2
4 manzanas  15 cajas es imposible, pues tendríamos, como mínimo:
15
15 16
1  2  3  ...  15   k   15  8  120  60 peras.
k 1 2
6 manzanas  10 cajas sí es posible:
10
10 11
1  2  3  ...  10   k   55  60 peras.
k 1 2
Podemos hacer la distribución 1  2  3  ...  8  9  15  60 (D)

6.13
Factorizamos el producto:
10  20  30  ...  90  216  34  510  71

Y necesitamos una factorización cuyos exponentes sean todos pares.


Está claro que debemos eliminar el 7, luego tachar el 70, con lo que nos queda:
215  34  59
Si tachamos además el 10 nos queda la factorización
214  34  58
Cuyos exponentes son todos pares, es decir, es un cuadrado perfecto.
Luego necesitamos eliminar 2 (B)

6.14
213  23  210  8  210 , luego serán 8: 1 210 , 2  210 , 3  210 , 4  210 , 5  210 , 6  210 , 7  210 , 8  210
(D)

6.15
7!8!9! 7!1  8  8  9  7!81  7  6  5  4  3  2  34  7  2  3  5  4  3  2  34 
 36 7  2  5  4  2 ( D)

6.16
n 6 6
 1 es un entero si y solo si es un entero, es decir n  6 es un divisor de 6.
n6 n6 n6
Luego
n  6 1 n  7
n6 2 n 8
n6 3 n 9
n  6  6  n  12

Para n  8 el divisor más grande es 4, y no 2. Luego el número de casos posibles es 3 (C)

6.17

b  1 21  22  23  ...  29  210  2123...910  255


a  1  21  22  23  ...  29  210 es la suma de una sucesión geométrica de 10 términos y razón
2, luego
 
a  1  21  22  23  ...  29  210  1  2 1  21  22  ...  28  29 
 
a  1  2 a  210  2a  211  211  1  2a  2  a
Finalmente,
5
 
a  211  1  a  1  211  a  1  211  255  b (B)
5

6.18
Denotando los diez números con a1 , a2 , a3 ,..., a10 , y teniendo en cuenta que
a1  a2  a3  a4  a5  a6  a7  a8  a9  a10  1  2  3  4  5  6  7  8  9  10  55
tenemos
S  a1  a2  a3  a4 

S  a4  a5  a6  a7   3S  a1  a2  a3  2a4  a5  a6  2a7  a8  a9  a10 
S  a7  a8  a9  a10 
 a1  a2  a3  a4  a5  a6  a7  a8  a9  a10  a4  a7  55  a4  a7

Luego 55  a4  a7 es un múltiplo de 3 mayor que 55. El primero es 57  19  3 , pero entonces


a4  a7  2 lo cual es imposible.
El siguiente es 60  20  3 , para el cual S  20 . (C)

Nota: Queda demostrar que existe al menos una configuración con S  20 . En las soluciones
oficiales Kangourou encontramos la siguiente:

6.19
Factorizamos: n 2  2n  3  (n  1)(n  3)  n 2  2n  3  n  1 n  3

Será primo si y solo si uno de estos dos factores es 1:


n 1  1  n  0,  2
n  3  1  n  4, 2

Hay cuatros soluciones posibles. (D)

6.20
Sean H y M el número de chicos y de chicas, respectivamente, en el curso pasado.
1 1 1 1 H M
H  H  M  M  H  M 1  H  M  1  1 H  M  5
5 5 5 5 5
1 1
Por otro lado, implícitamente H y M son enteros, luego H y M son múltiples de 5, y por
5 5
tanto H  M es un múltiple de 5.
Con esto descartamos A, B, y E.
Veamos B:
H  M  25
  2 H  30  H  15  M  10
H M 5 

Veamos D:
H  M  30 35
  2 H  35  H  imposible porque no es entero.
H M 5  2
La única solución válida es B.

6.21
b 
a   11
c  b a ab  1 c 1
  25  11  14  a   b   a  b   (a  b)1    (a  b)
a
b   14
c c c  c c
c 

Y por tanto 25c  (a  b)(c  1)  c  1 | 25  52
a
c  1  25  c  24  b  24  b   14 imposible. Además, el sistema
c
b 
a  11
24 
 no daría soluciones enteras.
a
b  14
24 
c  1  1  c  0 imposible.
Solo nos queda la posibilidad
a  b 20
c  1  5  c  4  25  4  (a  b)  5  a  b  5  4  20    5 (E)
c 4

6.22
N  10000a  2020 para cierto a | N
Luego N  ak para cierto entero k y por tanto
ak  10000a  2020  ak  10000k  2020  22  5 101
Los posibles valores de a son:
1 1
2 2
5 5
101 2
4 4
10 1
202 4
505 10
20 2
404 8
1010 2
2020 4
Total 45

Por otro lado, los 12 números de S acaban todos en 2020, y por lo tanto tenemos que sumar
12  (2  0  2  0)  12  4  48

Finalmente, en total, tenemos 45  48  93 .

6.23
La clave para resolver este problema es ver que la condición nn | mm implica que si p es un
factor primo de n , también lo será de m .
Por otro lado, la condición n | m implica n  1 .

Otra clave importante es ver que la condición ( m  n , 210 )  1 , con 210  2  3  5  7 implica
que si un primo p de un dígito (2,3,5 o 7) divide n, entonces también dividirá a m, y por tanto
dividirá m  n , con lo que ( m  n , 210 )  1.
Así pues, los factores primos de n serán 11, 13, 17…

Si estos factores primos de n estuvieran elevados a 0 o 1 como máximo, por ser también
factores de m , entonces m tendría todos los factores de n elevados a sus potencias respectivas,
es decir, n | m , contradiciendo la tercera hipótesis del enunciado.

Así pues, la factorización de n debe contener algún exponente mayor o igual que 2.

  121
El menor candidato posible de n es n  112  121 , y por tanto 112  11242 .
Puesto que, por hipótesis, 11242 | mm , necesitamos un m que sea múltiplo de 11, mayor o igual
que 242, que no sea divisible por 121 y que ( m  n , 210 )  1 .

m  242 no cumple pues es divisible entre 121.


m  253 no cumple pues m  n  253  121  374 es divisible entre 2.
m  264 no cumple pues m  n  264  121  385 es divisible entre 5.
m  275 no cumple pues m  n  275  121  396 es divisible entre 2.

Finalmente, m  286 cumple las condiciones del enunciado:


m  n  286  121  407  11 37 , y por tanto ( m  n , 210 )  1
m  2 1113 , n | m .
Luego la solución es 407.

Fuente: https://artofproblemsolving.com/wiki/index.php/2020_AIME_I_Problems/Problem_10

6.24
Todo número que cumpla las condiciones del enunciado se podrá escribir como ABCD con
D  0, B, C  0,2,4,6,8, A  2,4,6,8
En total hay 4  5  5 1  100 (B)

6.25
(n  2)!(n  1)! (n  1)!(n  2  1) (n  1)!(n  1)
   (n  1)(n  1)  (n  1) 2
n! n! n!

6.26
Queremos determinar los enteros positivos n cumpliendo  5!, n   5  10!, n  con 5 | n
  
 5!, n   5  10!, n   5  3  23 , n  5 7  52  34  28 , n 
Aplicaremos AR/4.30:
Sea n  2a2 3a3 5a5 7a7 p8a8 p9a9 ... pkak la descomposición factorial de n, con 7  p8 , p9 ,..., pk ,
cumpliendo a2 , a3 , a7  0 y a5  1 pues 5 | n .

 
5 7  52  34  28 , n  2b2 3b3 5b5 17b7 con b2  min( a2 ,8) , b3  min( a3 ,4) , b5  min( a5 ,2) y
b7  min( a7 ,1)

Por otro lado,


 
5  3  23 , n  2c2 3c3 5c5 7c7 p8a8 p9a9 ... pkak con c2  max( a2 ,3) , c3  max( a3 ,1) , c5  max( a5 ,1) ,
c7  max( a7 ,0)
   
Y de la igualdad 5  3  23 , n  5 7  52  34  28 , n deducimos que a8  a9  ...  ak  0
Y también se deduce que:
min( a2 ,8)  max( a2 ,3)  3  a2  8
min( a3 ,4)  max( a3 ,1)  1  a3  4
min( a5 ,2)  1  max( a5 ,1)  a5  3
min( a7 ,1)  max( a7 ,0)  0  a7  1
En total hay 6  4 1 2  48 ( D)

6.27
60  log10 x  2 log10 Mcd ( x, y)  log10 x  Mcd ( x, y)2 

570  log10 y  2 log10 mcm( x, y)  log10 y  mcm( x, y)2 
Sumando las dos igualdades llegamos a
  
630  log10 x  Mcd ( x, y ) 2  log10 y  mcm( x, y ) 2  

 log10 x  Mcd ( x, y ) 2 y  mcm( x, y ) 2  
x  Mcd ( x, y ) 2 y  mcm( x, y ) 2  10630
Utilizando ahora la identidad
Mcd ( x, y)  mcm( x, y)  x  y

Llegamos finalmente a
x  y  x 2  y 2  x3  y 3  10630  xy  10210  2
 2
 ...
 2  5 5
  ...
 5  m  n  420

210 210

Sean x  2a5b , y  2c5d .


Entonces Mcd ( x, y)  2min(a,c )5min(b, d ) , mcm( x, y)  2max( a, c )5max(b, d ) , y por tanto

 
60  log10 x  Mcd ( x, y ) 2  x  Mcd ( x, y ) 2  1060  260560 
x  22 min( a , c )52 min(b , d )  260560 
2a5b  22 min( a , c )52 min(b , d )  260560 
a  2 min( a, c)  60
2a  2 min( a , c )5b  2 min(b , d )  260560  
b  2 min( b, d )  60
 
570  log10 y  mcm( x, y ) 2  y  mcm( x, y ) 2  10570 
2c5d 2max( a , c )5max(b, d )  25705570 
c  2 max( a, c)  570
2c  2 max( a , c )5d  2 max(b, d )  25705570  
d  2 max( b, d )  570
Supongamos a  c , b  d
a  2 min( a, c)  60 a  2a  60 3a  60  a  20
b  2 min( b, d )  60 b  2b  60 3b  60  b  20
  
    
c  2 max( a, c)  570 c  2c  570 3c  570  c  190
d  2 max( b, d )  570 d  2d  570 3d  570  d  190
Y por tanto x  220520 , y  21905190 , Mcd ( x, y)  x , mcm( x, y)  y ,
En efecto:
  2
x  Mcd ( x, y)2  1020  1020  1020  1040  1060
10  2
y  mcm( x, y)2  10190 190  1028510380  10570
Cualquier otra posibilidad nos lleva a una incompatibilidad.
Supongamos a  c , b  d
a  2 min( a, c)  60 a  2a  60 a  20
b  2 min( b, d )  60 b  2d  60 b  360
  
   absurdo.
c  2 max( a, c)  570 c  2c  570 c  190
d  2 max( b, d )  570 d  2b  570 d  150

Supongamos a  c , b  d
a  2 min( a, c)  60 a  2c  60 a  360
b  2 min( b, d )  60 b  2b  60 b  20
  
   absurdo.
c  2 max( a, c)  570 c  2a  570 c  150
d  2 max( b, d )  570 d  2d  570 d  190
Supongamos a  c , b  d
a  2 min( a, c)  60 a  a  60
b  2 min( b, d )  60 b  2b  60
 
  absurdo.
c  2 max( a, c)  570 c  a  570  a  a  570
d  2 max( b, d )  570 d  2d  570

La única solución posible es x  220520  m  40 , y  21905190  n  380 , y por tanto


3m  2n  3  40  2  380  880

6.28
Sea n un número que cumpla las condiciones del enunciado.
20 | n , luego n  20k
n  2019 , luego k  100
20   n   20k    225k  
Aplicaremos detenidamente la fórmula 19.4b.

El exponente 2 del factor 2 genera un 3 en la función  , luego será imposible obtener 20. Hay
que aumentar este exponente.
  
20   225  2k   235  k 
Ahora tenemos un 4  2  8 , que nos impide obtener el 20.

Si aumentamos el exponente del 5 tampoco nos sirve, pues entonces


20   2352  k  
Obtenemos un 4  3  12 … y no son divisores de 20.

Así pues, el factor 2 tiene que tener exponente 4, luego:


  
20   22  5  22 k   24  5  k 
Ahora tenemos un factor 5  2  10 y solo tenemos que multiplicarlo por 2, lo que sucede si y
solo si añadimos un primo p diferente de 2 y de 5. Luego serán todos los números de la forma
n  24  5  p  20  4  p , con 4  p  100  p  25 , es decir: p  3,7, 11, 13, 17, 19, 23 .

Su suma será 24  5  3  24  5 11  ...  24  5  23  24  53  7  11  13  17  19  23  24  5  93

 
Otra combinación posible es 20   24  53   2000 , luego la suma total será
24  5  93  24  53

24  5  93  24  53
Y el resultado pedido es  22  93  22  52  372  100  472
20

6.29
Claramente (E), pues 27 no es primo y 27  2  25 tampoco lo es.
6.30
Sea x la cantidad de dinero de Casper, en céntimos.
Luego x debe ser múltiple de 12, 14 y 15, y por tanto el valor mínimo es
x  mcm(12,14,15)  420 cent.
420
Para esta cantidad, puede comprar  21 caramelos púrpura. (B).
20

6.31
100000  25  55 , luego S está formado por todos los números de la forma 2 a  5b con
0  a, b  5 , y por tanto los productos de dos números de S serán todos aquellos números de la
forma 2 c  5 d con 0  c, d  10 .
En total hay 1111  121 casos.

Pero debemos observar que algunos casos solo se pueden aparecer como el producto de dos
elementos iguales (recordemos que, por separado, el exponente máximo es 5).
 
1  20  50  20  50 20  50 
1  210  25  25
1  510  55  55
 
1  210  510  25  55 25  55 
Luego la respuesta correcta es 121  4  117 (C).

6.32
n
2
n
3 2
Si n es positivo, 4000     2  5 n que será entero para n  0,1,2,3 .
5

5 5
5 n
n
2
Si n es negativo, 4000     25  53  n que será entero para n  1,2,3,4,5 .
5 2
En total hay 9 casos (E)
6.33
Está claro que 1 no puede ser mínimo de S, pues sería divisor de cualquier otro elemento
posible, llegando a contradicción.

El 2 tampoco puede ser mínimo, pues es incompatible con 4, 6, 8, 10 y 12, y con el resto
tenemos un único posible conjunto S  3,5,7,9,11 que no es aceptable porque 3 y 9 no son
compatibles.

El 3 tampoco puede ser mínimo: Los otros candidatos son: 4, 5, 7, 8, 10, 11, y en este caso el 5
es incompatible con el 10, y el 4 es incompatible con el 8 luego eliminando alguno de ellos nos
quedan solo 4 candidatos posibles, y no llegamos a los 6 necesarios.

Con el 4 de mínimo tenemos candidatos posibles 5, 6, 7, 9, 10, 11. El 5 es incompatible con el


10, y con el resto podemos hacer un posible conjunto S  4,6,7,9,10,11. Así pues, el valor
mínimo es 4 (C).

6.34
Aplicando la suma de la sucesión geométrica:
10n  1
An  aa a  a 11
...  
...1  a 100  101  ...10n  a   9
n n

10n  1
Bn  bb b  b
...
n 9

Cn  c
c...c  c  11 ...1  c 
 102 n  1 
   c 
 
10n  1 10n  1 
   9 
2n 2n   9

Cn  Bn  A 2
 c
10 n

 1 10n  1b
10n  1  10n  1 
  a 
2
 10n  1 
  a 2   
2

n
9 9  9   9 
10n  1
puesto que n  0 podemos cancelar el factor para llegar a
9
 10n  1 
 
c  10n  1  b  a 2   
 9 

Esta ecuación es lineal en 10n , luego si es cierta para dos valores de n concretos será cierta
para cualquier n .

 10  1 
Para n  1 c  10  1  b  a 2     11c  b  a
2

 9 
 100  1 
Para n  2 c  100  1  b  a 2     101c  b  11a
2

 9 
11c  b  a 2


2
 101c  b  11c  b  11a 2  a 2  90c  10a 2  9c  a 2
101c  b  11a  

11c  b  a 2  110c  10b  10a 2 


2
   110c  10b  101c  b  10a  11a
2 2

101c  b  11a  101c  b  11a 2



2a 2
 9c  9b  a 2  a 2  9b  a 2  9b  2a 2  b 
9
2a 2 a 2 a2
Queremos encontrar el valor máximo de a  b  c  a   a
9 9 3
Equivale a encontrar el valor máximo de a .

La ecuación 9c  a 2 tiene tres soluciones (a, c) : (3,1) , (6,4) , (9,9) .

Pero sustituyendo para encontrar b : 11c  b  a 2  b  11c  a 2

(3,1)  b  11  9  2 , (6,4)  b  44  36  8 , (9,9)  b  99  81  18

Vemos que la solución (9,9) no es aceptable pues incumple la condición b  9 .

Observamos que se cumple siempre para a  6, b  8, c  4 . Lo podemos ver con un ejemplo:


n  4  62 1111  39996  40000  4  40000  4  8  4 104  1  8 
Si observamos la forma de los números que se obtienen de la forma a 211
 ...1 :
n
1 – 1111 2 – 4444 3 – 9999 4 – 17776 5 – 27775
6 – 39996 7 – 54439 8 – 71104 9 – 89991

Vemos que el siguiente candidato para obtener un número “parecido” a 10n sería
a  9  92  11...11  89...91

Pero vemos que no es aceptable:



n  4  92 1111  89991  90000  9  90000  9  18  9 104  1  18 
Pero b no puede ser 18.

Luego la solución es 6  8  4  18 (D).

Fuente: https://artofproblemsolving.com/wiki/index.php/2018_AMC_12A_Problems/Problem_25

6.35
Primera versión.
Sabemos que 2 | n y que 323 | n , y puesto que 2 y 323 son coprimos, se deduce que 646 | n , es
decir, n  646k . Puesto que 1000  n  10000 , está claro que 2  k  15 .
Por otro lado, vemos que 323  17 19 , luego la lista de todos los números n posibles es:
2 17 19  2 , 2 17 19  3 , 2 17 19  22 , 2 17 19  5 , 2  17  19  2  3
2 17 19  7 , 2 17 19  23 , 2 17 19  32 , 2 17 19  2  5 , 2 17 19 11
2 17 19  3  22 , 2 17 19 13 , 2 17 19  2  7 , 2 17 19  3  5

Tomamos las posibles respuestas que nos ofrece el enunciado y vamos descartando, de menor a
mayor:
324  22  34 y no aparece como divisor de ninguno de los números anteriores.
330  2  3  5 11 y tampoco es aceptable.
340  22  5 17 y este valor sí que aparece como divisor de 2 17 19  2  5
luego 340 es la respuesta correcta (C).

Segunda versión.
Sabemos que 323  17 19 | n , luego el siguiente divisor que buscamos debe ser múltiple de 17
o de 19, porque en caso contrario obtendríamos un número demasiado grande.
Por ejemplo:
Supongamos que 324  22  34 | n . Entonces:
324  22  34 | n
  2  3 17 19 | n  104652 | n  n  104652 absurdo.
2 4

323  17 19 | n

El primer valor de la lista que cumple esta condición es la opción C: 340  22  5 17 , y con este
número podemos obtener un valor de n aceptable: 2 17 19  2  5  6460 , así pues, la respuesta
es (C).

6.36
Primera versión.
   35q  63 p  36q  0  63 p  35q  36q  35q  0  79 p  5q   q
5 p 4
9 q 7

De aquí se deduce que q  7 .


Para los primeros valores de q tenemos que
1  5q
1  9 p  5q  p  , que no es entero para todos los valores q  14 .
9
2  5q
Para q  14 , también podráimos tener 9 p  5q  2 , y entonces: p  .
9
Seguimos probando: El valor q  15 no es aceptable, pero con q  16 tenemos:
1  5 16
p  9 , resultado entero. Por lo tanto la solución es 16  9  7 (A).
9

Segunda versión.
5 p 4 5 4
   q p q
9 q 7 9 7
5 4
Representando las respectivas gráficas f (q)  q , g (q)  q
9 7

Se observa que el primer punto de componentes enteras que está entre las dos gráficas es
(16,9), y por tanto la solución es 16  9  7 (A).

Fuente de estas soluciones: https://artofproblemsolving.com/wiki/index.php/2018_AMC_12B_Problems/Problem_17

6.37
Supongamos que x 2  a x  b  0 tiene dos raíces reales, p'  3 p y q'  3q , para ciertos p y q
enteros. Entonces, aplicando las fórmulas de Vieta:

a  ( p' q' )  3( p  q)


b  p' q'  3 p3q  9 pq
Y por tanto está claro que a y b también son enteros, y que a es múltiple de 3 y que b es
múltiple de 9: a  3 , b  9

Por otro lado, puesto que x2  bx  9a  0 tiene una única solución real, su discriminante será
cero:
0    b2  4 1 (9a)  b2  36a  b2  36a

Luego
9 2  363   81 2  36  3  3 2  4  22
De aquí deducimos que  es múltiple de 3, y que  es múltiple de 2. Así pues, a es múltiple
de 9 y b es múltiple de 18.

 b 2  18k   182 k 2
2
Luego b  18k para cierto k entero, y a     9k 2
36 36 36

con lo que la primera ecuación del enunciado queda


x 2  9k 2 x  18k  0

Con dos soluciones reales diferentes.

Primer caso: k  0 .
La ecuación se reduce a x 2  0 , con una única solución doble x  0 , y por tanto no podemos
considerar que se cumplen las condiciones del enunciado.

Segundo caso: k  1 .
La ecuación x 2  9 x  18  0 tiene soluciones x  3 y x  6 , que satisfacen las condiciones del
enunciado, luego a  9 y b  18 es una solución aceptable para nuestro problema.

Tercer caso: k  1 .
Para que la ecuación x 2  9k 2 x  18k  0 tenga dos soluciones reales diferentes, su
discriminate debe ser positivo:
 
2

0     9k 2  4  1  18k  81k 4  72k  32 9k 4  8k 
 
Así pues, 32 9k 4  8k es un cuadrado perfecto, y por tanto 9k 4  8k es un cuadrado perfecto.

Ahora bien, puesto que k  1 , vemos que se cumplen las desigualdades


9k 4  8k  9k 4  3k 2   2

3k 1
2 2
 
 9k 4  6k 21  9k 4  6k 21  9k 4  8k , puesto que k  1  6k 21  8k

Así pues, tenemos


3k 1
2 2
 
 9k 4  8k  3k 2
2

Es decir, 9k 4  8k se encuentra estrictamente entre dos cuadrados perfectos consecutivos, y por


tanto no puede ser un cuadrado perfecto.
Cuarto caso: k  1 .
La ecuación x 2  9 x  18  0 no tiene soluciones enteras, luego no se cumplen las condiciones
del enunciado.

Quinto caso: k  1.


El valor 9k 4  8k estudiado en el tercer caso, que debe ser un cuadrado perfecto, tomando
ahora h  k , con h  1, se convierte en
9h4  8h

que no puede ser un cuadrado perfecto puesto que (siguiendo razonamientos similares al tercer
caso) tenemos las desigualdades
3h 
2 2
 
 9h4  9h4  8h  3h2  1
2

es decir, se encuentra entre dos cuadrados perfectos consecutivos.

Así pues,la única solución aceptable es a  9 y b  18 .

Fuente de esta solución: Germán García López en Facebook.


6.38
n  a 1000  b 100  c 100  d  x2

Primer caso.
Vamos a suponer que las cifras iguales están en la segunda y tercera posición, como en el
ejemplo del enunciado. Entonces:

m  y 2  (a  1) 1000  b 100  c 10  d  1  a 1000  1000  b 100  c 10  d  1 


 a 1000  b 100  c 10  d  1000  1  x 2  1001

Luego
7 1113  1001  y 2  x2   y  x  y  x 
Las posibilidades son:
y  x 1 
  2 y  1002  y  501 y no es aceptable porque y  9999 .
2

y  x  7 11 13  1001


yx7   y  75  m  y 2  5625

  2 y  11 13  7  150  
y  x  11 13 
 x  75  7  68  n  x  4624
2

y  x  11  
 y  51  m  y  2601
2

  2 y  102  
y  x  7  13  91 
 x  51  11  40  n  x  1600
2

y  x  13   y  45  m  y 2  2025

  2 y  90  
y  x  7 11  77 
 x  45  13  32  n  x  1024
2

y  x  7 11  77  y  45  m  y 2  2025


  2 y  90   no es aceptable.
y  x  13   x  13  45  0
y  x  7  13  91  y  51  m  y 2  2601
  2 y  102   no es aceptable.
y  x  11   x  11  51  0
y  x  11 13  143  y  75  m  y 2  5625
  2 y  150   no es aceptable.
yx7   x  7  75  0
y  x  7  11  13  1001
  2 y  1002  y  501 no es aceptable.
y  x 1 

Segundo caso.
Vamos a suponer que las cifras iguales están en la primera y tercera posición. Entonces:
m  y 2  a 1000  (b  1) 100  c 10  d  1  a 1000  b 100  100  c 10  d  1 
 a 1000  b 100  c 10  d  100  1  x 2  101
Luego
101  y 2  x2   y  x  y  x 
Puesto que 101 es primo, solo hay dos posibilidades:
y  x  101  y  51  m  y 2  2601
  2 y  102   no es aceptable.
y  x 1   x  1  51  0
y  x 1   y  51  m  y 2  2601

  2 y  102  
y  x  101 
 x  101  51  50  n  x  2500
2

Tercer caso.
Vamos a suponer que las cifras iguales están en la primera y segunda posición. Entonces:
m  y 2  a 1000  b 100  (c  1) 10  d  1  a 1000  b 100  c 10  10  d  1 
 a 1000  b 100  c 100  d  10  1  x 2  11

Luego
11  y 2  x2   y  x  y  x 
De nuevo, puesto que 101 es primo, solo hay dos posibilidades:
y  x  11
  2 y  12  y  6  m  y  36  1000 no es aceptable.
2

y  x 1 
y  x 1 
  2 y  12  y  6  m  y  36  1000 no es aceptable.
2

y  x  11

Cuarto caso.
Vamos a suponer que las cifras iguales están en la primera y cuarta posición. Entonces:
m  y 2  a 1000  (b  1) 100  (c  1) 10  d  1  a 1000  b 100  100  c 10  10  d 
 a 1000  b 100  c 100  d  100  10  x 2  110

Luego
2  5 11  y 2  x2   y  x  y  x 
Las posibilidades son:
y  x 1 
  2 y  111  y  111 / 2 no es aceptable.
y  x  2  5 11  110
yx2 
  2 y  57  y  57 / 2 no es aceptable.
y  x  5 11  55
yx5 
  2 y  27  y  27 / 2 no es aceptable.
y  x  2  11  22
y  x  11 
  2 y  21  y  21 / 2 no es aceptable.
y  x  2  5  10
y  x  2  5  10
  2 y  21  y  21 / 2 no es aceptable.
y  x  11 
y  x  2 11  22
  2 y  27  y  27 / 2 no es aceptable.
y x5 
y  x  5 11  55
  2 y  57  y  57 / 2 no es aceptable.
yx2 
y  x  2  5 11  110
  2 y  111  y  111 / 2 no es aceptable.
y  x 1 

No hay ningún caso aceptable.

Quinto caso.
Vamos a suponer que las cifras iguales están en la tercera y cuarta posición. Entonces:
m  y 2  (a  1) 1000  (b  1) 100  c 10  d  a 1000  1000  b 100  100  c 10  d 
 a 1000  b 100  c 100  d  1000  100  x 2  1100
Luego
22  52 11  y 2  x2   y  x  y  x 

Estudiando las posibles combinaciones, como en los casos anteriores, llegamos a las dos únicas
soluciones:
x  14 
  n  x  196  1000 no es aceptable
2

y  36
x  50  
n  x  2500
2

 que es la única solución aceptable.


y  60 m  y 2
 3600

Sexto caso.
Vamos a suponer que las cifras iguales están en la segunda y cuarta posición. Entonces:
m  y 2  (a  1) 1000  b 100  (c  1) 10  d  a 1000  1000  b 100  c 10  10  d 
 a 1000  b 100  c 100  d  1000  10  x 2  1010

Y estudiando las posibles combinaciones observamos que no hay ninguna aceptable.

Así pues, las soluciones para este problema son las cinco parejas siguientes:
(5625,4624), (2601,1600), (2025, 1024) , (2601, 2500), (2500, 3600)
Nota: Podríamos haber desechado muchos casos directamente teniendo en cuenta que x  y y
x  y tienen la misma paridad, y que 999  x2  y 2  10000  31  x  y  100 y por tanto
63  x  y  199 .

6.39
Vemos que
8n  3
17n  9

es irreductible si y solo si 8n  3 y 17n  9 son coprimos, es decir, su máximo común divisor es


1.
Podemos aplicar el Algoritmo de Euclides (ver 4.17) para encontrar el máximo común divisor:

17n  9 8n  3
16n  6 2
n3

Luego  17n  9 , 8n  3    n  3 , 8n  3 
De nuevo

8n  3 n3
8n  24 8
21

Y por tanto  n  3 , 8n  3    n  3 , 21 

Así pues, buscamos todos los enteros positivos n tales que n - 3 es coprimo con 21 , es decir,
todos los números n tales que n - 3 que no sea divisibles ni entre 3 ni entre 7.
Pero n - 3 es divisible entre 3 si y solo si n es divisible entre 3, y n - 3 es divisible entre 7 si y
solo si n  3  7k  n  7k  3 .
Luego serán todos los enteros positivos n tales que no sean de la forma n  3a ni n  7b  3 .

6.40
6n sí es divisible entre 3,luego 6n  1 no será divisible entre 3, pues siempre generará residuo
2. (D)

6.41
Si es divisible entre 2 y entre 3 debe ser divisible entre 6, luego "6 y 7" queda descartada.
Si es divisible entre 10 debe ser divisible entre 5 luego "4 y 5" queda descartada.
Si es divisible entre 6 debe ser divisible entre 2 y entre 3, luego "2 y 3" queda descartada.
Si es divisible entre 2 y entre 5, lo será también entre 10, luego "10 y 11" queda descartada.
La única opción que queda es "7 y 8". (D)

6.42
Sea a el número de movimientos "sacar 30 canicas" y b el número de movimientos "poner 18
canicas".
El número de canicas que quedarán será 71  30a  18b  71  65a  3b es decir, que
quitaremos o pondremos un número de canicas múltiplo de 6.
Luego el máximo se encontrara para 5  71  6  11  11  5a  3b que se obtiene para
a  4, b  3 . (C)

6.43
Si vemos como evoluciona el número primo p2 , vemos que la pirámide tendrá 9 filas. Ahora
vamos marcando las casillas en las que aparecerá el primo p4 y vemos que hay 24 en total.

(C)

6.44
Sea 10  n  99 de forma que 20  n  5  22  n es un cuadrado perfecto. Esto es equivalente a
que 5n sea un cuadrado perfecto, luego n será múltiplo de 5 por un cuadrado perfecto. Son
tres: n  5  4  5  22 , n  5  9  5  32 , n  5  16  5  42 . (C)

6.45
Un número es divisible entre 18 cuando lo es entre 2 y entre 9.
Tomando todas las cifras del 1 al 9 siempre obtenemos un número divisible entre 9 porque la
suma será 45, que es divisible entre 9.
Será, además, divisible entre 2 cuando acabe en cifra par, es decir, 2, 4, 6, 8, luego son 4 casos
de los 9 posibles, la probabilidad es 4/9 (B)

6.46
Analicemos como debe ser es número:
0 | a b 0 nunca, 1 | a b 1 siempre, 2 | a b 2 siempre, 3 | a b 3 , 4 | a b 4 , 5 | a b 5 siempre,
6| a b 6, 7| a b 7, 8| a b 8, 9| a b 9 .

Para que sean consecutivos, al menos se debe cumplir 3 | a b 3 y 7 | a b 7 . Con estos dos datos
ya podemos deducir el número.
3| a b 3  3| a  b  3  3| a  b .
b/a 0 1 2 3 4 5 6 7 8 9
0 X X X
1 X X X
2 X X X
3 X X X X
4 X X X
5 X X X
6 X X X X
7 X X X
8 X X X
9 X X X X

Encontramos muchos candidatos posibles, para que se cumpla 7 | a b 7 , puesto que en la tabla
del 7 el único número acabado en 7 es 7  1  7 , el cociente deberá acabar en 1 y por tanto
7 | a b . Probando probando con los valores de la tabla anterior llegamos al único candidato
aceptable que es a  8 , b  4 , por lo que el número buscado es 841 y la solución es
8  4  1  13 (D)

Nota: Una solución más elegante (y desde luego mucho más rápida para un contexto de una
prueba con tiempo limitado) sería observar que
c | a b c  c | a b 0 , y que 3  5  7  8  840

6.47
Sea n el número de minutos que necesita para recorrer una milla en el primer día. Por
60 60
proporcionalidad directa, el primer día recorrerá millas, el segundo , el tercero
n n5
60 60
y el cuarto . Puesto que se nos dice que las distancias recorridas son números
n  10 n  15
enteros, deducimos que n, n  5, n  10, n  15 son divisores de 60. Los divisores de 60 mayores
de 15 son 20, 30 y el propio 60, luego probando vemos que la única opción aceptable es
n  5, n  5  10, n  10  15, n  15  20 .
60 60 60 60
La distancia recorrida es     12  6  4  3  25 (C)
5 10 15 20

6.48
2009  72  41 luego
a  b  2009  7 41  a  7 41  b 
 2
 a  7 41  b  72  41  b  2  7 41 b  72  41  b  2  7 41b
Sabemos que a es un entero, y en la parte de la derecha de la igualdad, el único elemento
susceptible de no ser entero es 41b . Luego 41b también debe ser entero, y puesto que 41 es
primo, deducimos que b es un múltiplo de 41 multiplicado por un cuadrado perfecto. De la
misma forma deducimos que a debe ser un múltiplo de 41 multiplicado por un cuadrado
perfecto:
a  41x 2 

2
 a  b  41x 2  41y 2  41x  41y  7 41  x  y  7
b  41y  
Buscamos parejas 0  x, y tales que x  y  7 :
x  0, y  7  a  41 02 , b  41 72
x  1, y  6  a  41  12 , b  41  62
x  2, y  5  a  41 22 , b  41 52
x  3, y  4  a  41  32 , b  41  42
Y las tres que quedan, invertiendo los valores de x e y . Son ocho soluciones en total:
( 0 , 2009 ) , ( 41 , 1476 ) , ( 164 , 1025 ) , ( 369 , 656 ) , ( 656 , 369 ) , ( 1025 , 164 ) ,
( 1476 , 41 ) , ( 2009 , 0 )

Fuente: https://youtu.be/quECgYPNCXw

6.49
5

n 1 n
  
n
 2
n
k n  1 n n  1 n  1n  1 n  1
 
 5 n 2  1  nk  5n 2  5  nk

 5n 2  nk  5  n5n  k   5

Las únicas combinaciones posibles con enteros son:


n  5
  25  k  1  k  24
5n  k  1
n  1
  5  k  25  k  20
5n  k  25
Esta segunda solución no es aceptable pues se piden valores positivos, luego la única solución
posible es k  24 (C)

Un segundo razonamiento alternativo sería:


5 n k n2  1 1 5
 2    n   k  5n 
k n 1 5 n n n
5
Puesto que k es entero, y 5n es entero, necesariamente debe ser entero, luego n  1 o n  5
n
Y se llega igualmente al resultado deseado.

6.50
abc  1
(a  1)(b  1)(c  1) divide abc  1 si y solo si es un entero.
(a  1)(b  1)(c  1)
Observamos que
abc  1 abc a b c
    (*)
(a  1)(b  1)(c  1) (a  1)(b  1)(c  1) a  1 b  1 c  1

Y que
a 1 1
2a 1 1 2
a 1 a 1 2 1
b 1 1 3
3b 1 1 
b 1 b 1 3 1 2
c 1 1 4
4c 1 1 
c 1 c 1 4 1 3

Luego
3 4
(*)  2    4
2 3

abc  1
Así pues, solo puede ser 1, 2 o 3.
(a  1)(b  1)(c  1)

Vemos que no puede ser 1, puesto que, expandiendo el denominador:


abc  1 abc  1
1  
(a  1)(b  1)(c  1) abc  1  a  b  c  ab  bc  ac
 a  b  c  ab  bc  ac  0  a  b  c  ab  bc  ac
Lo cual es absurdo, porque 1  a  b  c  ab  bc  ac  a  b  c .

abc  1
Luego solo puede ser igual a 2 o 3.
(a  1)(b  1)(c  1)

Con un razonamiento parecido al primero, vemos que si a  4 , entonces


abc  1 abc a b c
    (**)
(a  1)(b  1)(c  1) (a  1)(b  1)(c  1) a  1 b  1 c  1
1 1 1 a 1 1 4
a  4  a 1  3     1 1 
a 1 4 1 3 a 1 a 1 3 3
1 1 1 b 1 1 5
b  5  b 1  4     1 1 
b 1 5 1 4 b 1 b 1 4 4
1 1 1 c 1 1 6
c  6  c 1  5     1 1 
c 1 6 1 5 c 1 c 1 5 5
Y por tanto:
4 5 6 6
(**)      2
3 4 5 3

abc  1
Luego  2 y ya hemos visto que es imposible.
(a  1)(b  1)(c  1)
Así pues, a  2 o a  3 .

abc  1 2bc  1
Caso 1: a  2  
(a  1)(b  1)(c  1) (b  1)(c  1)

Caso 1.1
2bc  1
2  2(b  1)(c  1)  2bc  1  2(bc  b  c  1)  2bc  1
(b  1)(c  1)
 2b  2c  3  0  3  2(b  c)
Lo cual es imposible, pues la parte izquierda es impar y la parte de la derecha, par.

Caso 1.2
2bc  1
3  3(b  1)(c  1)  2bc  1  3(bc  b  c  1)  2bc  1
(b  1)(c  1)
 3bc  3b  3c  3  2bc  1  bc  3b  3c  4  0
 5  bc  3b  3c  9  (b  3)(c  3)
Las únicas posibilidades son:
b  3  5 b  8
  que no cumple la condición c  b
c  3  1 c  4
b  3  1 b  4
  que sí cumple todas las condiciones.
c  3  5 c  8
Comprobamos la segunda solución:
(a  1)(b  1)(c  1)  1  3  7  21
a  2, b  4, c  8  
abc  1  2  4  8  1  63
Y efectivamente, 21 | 63
Así pues, hemos encontrado la primera solución: a  2, b  4, c  8 .

abc  1 3bc  1
Caso 2: a  3  
(a  1)(b  1)(c  1) 2(b  1)(c  1)

Caso 2.1
3bc  1
 2  3bc  1  4(b  1)(c  1)  4bc  4b  4c  4
2(b  1)(c  1)
 0  bc  4b  4c  5  11  bc  4b  4c  16  (b  4)(c  4)

Las únicas posibilidades son:


b  4  11 b  15
  que no cumple la condición c  b
c  4  1 c  5
b  4  1 b  5
  que sí cumple las condiciones requeridas.
c  4  11 c  15

Comprobamos la segunda solución:


(a  1)(b  1)(c  1)  2  4 14  112
a  3, b  5, c  15  
abc  1  3  5 15  1  224
Y efectivamente, 112 | 224 .

Así pues, hemos encontrado una segunda solución: a  3, b  5, c  15 .

Caso 2.2
3bc  1
 3  3bc  1  3(b  1)(c  1)  3bc  3b  3c  3
2(b  1)(c  1)
4
 0  3b  3c  4  4  3(b  c)  b  c 
3
Y no puede cumplirse pues b y c son enteros.

Las únicas soluciones son a  2, b  4, c  8 y a  3, b  5, c  15 .

Fuente de esta solución: https://youtu.be/qf3qipeZVb8

6.51
En primer lugar vemos que esta ecuación es perfectamente simétrica, por lo que podemos
suponer que las incógnitas están ordenadas:

3
1 1 1 1 1 1  1  1  1   1 
1  a  b  c  1     2  1   1   1   1  1  1    1  
a b c a b c  a  b  c   a 

Así pues, para cualquier terna a  b  c de soluciones, se cumplirá la desigualdad


3
 1
2  1  
 a
3
 1 1
La función f ( x)  1   es estrictamente decreciente, pues 1  es estrictamente
 x x
3
decreciente y x es estrictamente creciente.

Por otro lado, observamos que


3 3
 4  64  5  125
f (3)      2 y f (4)     2
 3  27 4 64

Luego el valor de a (que, recordemos, es el mínimo de la lista) debe cumplir a  3 . Veamos


los casos posibles.

Para a  1 :
 1  1  1   1  1   1  1 
2  1  1  1    21  1    1  1    1
 1  b  c   b  c   b  c 
1 1
es imposible puesto que ,  0
b c

Para a  2 :
 1  1  1  3  1  1   1  1   b  1  c  1 
2  1  1  1    1  1    4  31  1    3  
 2  b  c  2  b  c   b  c   b  c 
4bc  3(b  1)(c  1)  3(bc  b  c  1)  3bc  3b  3c  3 
0  bc  3b  3c  3

Completamos esta última ecuación:


0  bc  3b  3c  3  bc  3b  3c  9  12  (b  3)(c  3)  12 
12  (b  3)(c  3)

Las posibilidades son (recordemos que estamos suponiendo b  c ) :


1 b 3  2  b  3
  b  4, c  15 ;   b  5, c  9
12  c  3 6  c  3
3  b  3
  b  6, c  7
4  c  3

Para a  3 :
 1  1  1  4  1  1   1  1 
2  1  1  1    1  1    6  41  1  
 3  b  c  3  b  c   b  c 
 1  1   b  1  c  1 
 3  21  1    2  
 b  c   b  c 
3bc  2(b  1)(c  1)  2bc  2b  2c  2  0  bc  2b  2c  2

De nuevo completamos cuadrados para resolver esta ecuación:


0  bc  2b  2c  2  (b  2)(c  2)  6  6  (b  2)(c  2)
1 b  2
  b  3, c  8
6  c  2
2  b  2
  b  4, c  5
3  c  2

Llegamos así a la lista final de posibles soluciones ordenadas:


a  2, b  4, c  15 ; a  2, b  5, c  9 ; a  2, b  6, c  7 ; a  3, b  3, c  8 ;
a  3, b  4, c  5

El conjunto total de soluciones son todas las permutaciones posibles que se pueden realizar con
las ternas anteriores.

Fuente de esta solución: https://youtu.be/iYZFWq6sEsc

6.52
1 2 3 1 2 3
  1   1
x y z x y z

1 2 1 2 3
Si x, y  3      1  1  para todo z  1, y la ecuación no tendrá solución.
x y 3 3 z

Caso 1. x  1 .
2 3 2 3
1   1     2 z  3 y , cuyas soluciones son todas las ternas de la forma
y z y z
x  1

 y  2k , k  1
 z  3k

1 2 3 1 1
En efecto,   1  1
1 2k 3k k k

Caso 2: y  1
1 2 3 1 3 1 3 3 1 3x  z
 1   2 1  1   1    1 
x 1 z x z x z z x xz
 xz  3x  z  0  ( x  1)( z  3)  3  0  ( x  1)( z  3)  3

Puesto que x  1  x  1  2 , la única solución posible es


x  1  3  x  2
 
z  3  1 z  2 

1 1 3
En efecto:   1
2 1 2

A partir de aquí podemos suponer x, y  1


Caso 3. x  2 .
1 2 3 2 1 3 z6
 1      4 z  y ( z  6)  yz  6 y  4 z  yz  6 y  0 
2 y z y 2 z 2z
 (4  y )( z  6)  24  0  (4  y )( z  6)  24

z  6 es positivo, luego 4  y  0  1  y  4

y  2  2( z  6)  24  z  6  12  z  6 , obteniendo la solución x  2, y  2, z  6

y  3  z  6  24  z  18 , obteniendo la solución x  2, y  3, z  18

Caso 4. y  2
1 2 3 1 3 1 3
  1       z  3x dando lugar al conjunto de soluciones
x 2 z x z x z
x  k

y  1 , k  1
 z  3k

1 2 3 1 3 k  3k  2k
En efecto:     2 2  2  2  2  0
k 1 k k k k k

Así pues, las soluciones de esta ecuación son todas las ternas siguientes:
( 1 , 2k , 3k ) , k  1 ; ( 2 , 1 , 2 ) ; ( 2 , 3 , 18 ) ; ( k , 2 , 3k ) , k  1

6.53
1 4 1 n  4m 1
     12(n  4m)  m n  0  m n  12n  48m
m n 12 mn 12
Para resolver esta ecuación aplicamos el método de completar cuadrados:
0  m n  12n  48m  (n  48)(m  12)  12  48 
12  48  (n  48)(m  12) 
26  32  (n  48)(m  12)
Puesto que, además, n debe ser impar, el factor n  48 no puede ser divisible entre 2, y por
tanto las únicas posibilidades aceptables son:
1  n  48 
  n  49, m  588
26  32  m  12
3  n  48 
  n  51, m  204
2  3  m  12
6

32  n  48 
  n  57, m  76
26  m  12

6.54
1 1 3 ab 3 2018 2018
     2018(a  b)  3ab  0  ab  a b
a b 2018 ab 2018 3 3
 2018  2018  20182
 0  a   b   2 
 3  3  3
2018 
2
2018  2018 
 a   b  
 3  3 
2
3
22 10092  20182  3a  20183b  2018

Estudiemos los posibles casos:


1  3a  2018  a  673

 
22 10092  3b  2018 b  22 10092  2 1009 / 3  2 1009  673  1358114
2  3a  2018 
  a  2020 / 3 , no es entero.
2 1009  3b  2018
2

2 1009  3a  2018 
  a  4036 / 3 , no es entero
2 10092  3b  2018

Nota: Estos dos casos anteriores se podrían haber rechazado teniendo en cuenta que
3a  2018  1 (mod 3) .

22  3a  2018 
 a  674

 b  340033
1009  3b  2018
2

22 1009  3a  2018 a  2018


  
1009  3b  2018  b  1009

El resto de soluciones se obtienen intercambiando los valores. Así pues, las soluciones son:
( 673 , 1358114 ) , ( 674 , 340033 ) , ( 2018 , 1009 ) , ( 1009 , 2018 ) ,
( 1358114 , 673 ) , ( 340033 , 674 ) .

6.55
Está claro que x  x  y  1984  x  1984  x  1984 .
1984  x  y  y  1984  x  y   1984  x   1984  x  2
2
1984 x

De aquí deducimos que 1984 x debe ser entero, luego 1984 x  26  31x tiene que ser un
cuadrado perfecto.

  2
26  31x  23 31x es un cuadrado perfecto  31x es un cuadrado perfecto.
31x  a 2  31 | a 2  31 | a  a  31b  a 2  312 b 2
31x  312 b 2  x  31b 2
x  31b2  1984  b  1984 / 31  8
Para cada valor 1  b  7 obtenemos un valor x  31b2  x  31b , y un valor
y  1984  x   8
2

31  31b  318  b  318  b
2 2 2

La condición x  y se convierte en
31b 2  31(8  b) 2  b 2  (8  b) 2  b 2  64  16b  b 2 
 0  164  b   0  4  b  b  4
Luego son 3 los posibles valores aceptables. Aunque no es necesario para la resolución del
problema, las soluciones son:
b  1  x  31  y  318  1  31  72  ( 31,1519 )
2

b  2  x  124  y  318  2  1116  (124 ,1116 )


2

b  3  x  279  y  318  3  775  (279 ,775 )


2

A partir de este valor se contradice la hipótesis x  y :


b  4  x  496  y  318  3  496  (496 ,496 )
2

6.56
Las condiciones del enunciado se corresponden con el siguiente sistema de ecuaciones
diofánticas:
n  14 2 a  14b  c

n  15 a  15c  b
2

n  6 3 a  6 2 c  6 a  c

cumpliendo, además: 0  a  6 , 0  c  6 , 0  b  14

Simplificamos la tercera ecuación:


n  63 a  62 c  6a  c  a(63  6)a  c(62  1)  222a  37c

y la sustituimos en las otras dos ecuaciones:


222a  37c  14 2 a  14b  c  0  26a  14b  36c  0  13a  7b  18c



222a  37c  15 a  15c  b  0  3a  b  22c
2

Multiplicamos la segunda ecuación por 7 y le restamos la primera:


0  13a  7b  18c
  0  34a  136c  34a  4c 
0  21a  7b  154c

Luego a  4c

Puesto que 0  a  6 , la única posibilidad es que c  1 , y por tanto a  4 , y


n  222  4  37  1  888  37  925

6.57
  
2009  x2  y 4  x  y 2 x  y 2  a  b
Supongamos en primer lugar, que a y b no son primos entre sí. Entonces mcd (a, b)2 es un
divisor de 2009  72  41 , y por tanto la única posibilidad es que mcd (a, b)  7 . Luego:
a  x  y 2  7a' 
  2 x  7(a'b' )  7 es divisor de x, y puesto que es también divisor de a,
b  x  y  7b' 
2

será también divisor de y. Luego:
x  7 x' 
y  7 y '
2
 
2 2 2 2
 
2 2
 2

( x  y )( x  y )  7 x'7 y ' 7 x'7 y '  7 x'7 y ' x'7 y '  7  41 
2 2

x'7 y' x'7 y'   41


2 2

La única posibilidad es
x'7 y'2  41 , x'7 y'2  1  2 x'  41  1  42  x'  21  7 y'2  41  x'  41  21  20 absurdo.

Luego a y b son primos entre sí.


x  y  a
2

  x  a  y 2  a  y 2  y 2  b  2 y 2  b  a . Luego b  a , b  a es positivo, par



x  y  b
2

ba
y es un cuadrado perfecto.
2

Veamos todas las posibilidades:


a)
a7  ba
  b  a  287  7  280   140 no es un cuadrado.
b  7  41 2

b)
a  41  ba
  b  a  49  41  8   4  22 , sí es un cuadrado.
b  7  7 2

x  y 2  41

  2 x  41  49  90  x  45, y  49  45  4  y  2
2
En este caso:
x  y  49
2

  
En efecto, 45  2 45  22  41 49  2009
2

c)
a 1  ba
  b  a  2009  1  2008   1004 no es un cuadrado.
b  7  7  41  2009 2

Luego las soluciones son x  45 , y  2 .

Fuente de esta solución: Solución oficial de la OME.

6.58
Puesto que los únicos factores de 23 son 23 y 1, tenemos:
c  23
ac  bc  23  c(a  b)  23  
c  1
Supongamos que c  1 . Entonces el sistema queda:
ab  b  44 b(a  1)  44 a  1  b  23  1  22
   (23  a)(a  1)  44  
a  b  23 b  23  a a  21  b  23  21  2

Las soluciones son: a  1, b  22, c  1 y a  21, b  2, c  1 . Efectivamente, las comprobamos:


1  22  22  1  44
a  1, b  22, c  1  
1  1  22  1  23
21  2  2  1  44
a  21, b  2, c  1  
21  1  2  1  23

El caso c  23 no se puede dar puesto que a y b son positivos, y por tanto a  b  1 y


23(a  b)  23

Sin tener en cuenta este hecho, y intentando resolver el sistema, hubiéramos llegado a
soluciones negativas, y por tanto no válidas:
ab  23b  44 ab  23b  44  b(a  23)  44 a  1
   (1  a)(a  23)  44  
23a  23b  23 a  b  1  b  1  a a  21
El número de soluciones es 2.

6.59
abc  70  2  5  7

cde  71  71
efg  72  23  32

La clave está en la segunda ecuación: cde  71 que es un número primo, luego solo puede
ocurrir uno de los tres casos siguientes:
a) c  1, d  1, e  71
Pero entonces 71  fg  72 , lo cual es imposible.
b) c  71, d  1, e  1
Pero entonces ab  71  70 , lo cual es imposible.
c) c  1, d  71, e  1
Tenemos el sistema
ab  2  5  7

 fg  2  3
3 2

La primera ecuación permite los siguientes valores de a :


a  1,2,5,7,2  5,2  7,5  7,2  5  7 , ocho casos diferentes.
La segunda ecuación permite los siguientes valores de f :
f  1,2,22 ,23 ,3,32 ,2  3,2  32 ,22  3,22  32 ,23  3,23  32 , doce casos diferentes.
Luego el total de casos es 8  12  96 .

6.60
Primera versión.
Vamos a resolver la ecuación diofántica y 2  3x 2 y 2  30 x 2  517
Sean a  x 2 y b  y 2 . La ecuación anterior es equivalente a
b  3ab  30a  517  b  3ab  30a  517  b(1  3a)  30a  10  517  10 
b(1  3a)  10(3a  1)  507 (*)

Sea c  1 3a . Puesto que a y b son cuadrados, serán positivos, y también lo será c.

(*)  bc  10c  507  c(b  10)  507  3  132

De todas las soluciones posibles (tomando combinaciones de 3,13,132 ), la única que genera
como soluciones dos cuadrados es:
b  10  3  13  b  49  72  c  13  1  3a  a  4  22
Y por tanto 3x 2 y 2  3ab  3  4  49  588 .

Segunda versión.
  
La ecuación original se puede escribir como y 2  10 3x 2  1  3  132 .
Observamos que, puesto que y es un entero y 3x 2  1 es positivo, y 2  10 debe ser también
positivo. Luego y 2  10  1,3,13,39,169,507 , y por tanto
y 2  11,13,23,49,179,517. El único cuadrado perfecto de esta última lista es 49, luego
y 2  10  39 , y por tanto 3x 2  1  12  x 2  4  3x 2 y 2  12  49  588 .

Fuente de la segunda versión: The Contest Problem Book V (George Berzsenyi)

6.61
4 2 4n  2m
 1  1  4 n  2 m  m n  4 n  2m  m n  0
m n mn

Completando cuadrados:
0  4n  2m  m n  (m  4)(n  2)  8  (m  4)(n  2)  8  23

Hay cuatro posibilidades:


m  4  1 , n  2  8  m  5 , n  10
m42,n24m6,n6
m4 4,n2 2 m 8,n  4
m  4  8 , n  2  1  m  12 , n  3

6.62
Realizamos la sustitución a  p  q  p  a  q  p  q  a  q  q  a  2q
La ecuación se ha transformado en a  2q  a3
 
a  2q  a3  2q  a3  a  a a 2  1  aa  1a  1

Forzosamente uno de los tres factores deben ser 2.


a  2  2q  2  1  3  6  2  3  q  3  p  a  q  2  3  5
Y en efecto, 8  5  3  (5  3)3
Las otras opciones no son aceptables:
a  1  2  a  3  2q  3  2  4  24  2  12  q  12 no es primo.
a  1  2  a  1  2q  1  0  2  0  q  0 no es primo.

La única solución es p  5 , q  3 .

6.63
1. Caso c  0  c  c
1.1 Si a  b  0 , la ecuación queda
a  b  ab  78  ab  a  b  78 , y completando cuadrados:
ab  a  b  1  78  1  (1  a)(1  b)  79 , y 79 es primo, luego:
1  a  79  a  78
 no cumplen las hipótesis.
1 b 1 b  0 

1.2 Si a  b  0 , la ecuación queda


 (a  b)  ab  78  78  ab  a  b , y completando cuadrados:
ab  a  b  1  78  1  (a  1)(b  1)  79
a  1  79  a  78
 no cumple a  b  0
b 1 1 b  0 
a  1  79  a  80
  c  19  a  b  19   80  2  19  82  0
b  1  1  b  2 

Luego no hay solución posible con c  0 .

2. Caso c  0  c  c :
 a  b  c  19
El sistema queda de la siguiente forma: 
 ab  c  97
2.1 Supongamos que a  b  0 . Entonces
a  b  c  19  c  19  a  b
  ab  (19  a  b)  97  ab  a  b  97  19  116
ab  c  97 
Completamos cuadrados:
ab  a  b  116  ab  a  b  1  116  1  (a  1)(b  1)  117  32  13
a 1  3  a  2 
  c  ab  97  2  38  97  21
b  1  3  13  39  b  38
a 1  9  a  8 
  c  ab  97  8  12  97  1
b  1  13  b  12
a 1 1  a  0 
  c  ab  97  0  97  97
b  1  117  b  116

Las soluciones que aparecen son:


a  2 , b  38 , c  21 , a  38, b  2 , c  21
a  8 , b  12 , c  1 , a  12, b  8 , c  1
a  0 , b  116 , c  97 , a  116, b  0 , c  97
2.2 Supongamos que a  b  0 . Entonces
 (a  b)  c  19  c  19  a  b
  ab  (19  a  b)  97  ab  a  b  97  19  116
ab  c  97 

Completando cuadrados:
ab  a  b  116  ab  a  b  1  116  1  (1  a)(1  b)  117

1 a 1  a  0 
  c  ab  97  0  97  97
1  b  117  b  116
1  a  3  a  2 
  c  ab  97  (2)(38)  97  21
1  b  3  13  39  b  38
1  a  9  a  8 
  c  ab  97  (8)(12)  97  1
1  b  13  b  12

Las soluciones que aparecen son:


a  0 , b  116 , c  97 , a  116, b  0 , c  97
a  2 , b  38 , c  21 , a  38 , b  2 , c  21
a  8 , b  12 , c  1 , a  12 , b  8 , c  1
En total 12 soluciones posibles.

6.64
a  1 (1)

a  1  a  1 , b par (2)
b

a  0, b  0 (3)

x  2
(1) x 2  x  1  1  0  x 2  x  2  ( x  2)( x  1)  
 x  1
Las dos soluciones son aceptables.

x  0  x  2  2
(2) x 2  x  1  1  0  x 2  x  x( x  1)  
x  1  x  2  3
La solución x  1 no es aceptable pues entonces el exponente no es par.

(3) x  2  0  x  2  (2)2  (2)  1  4  2  1  5  0 luego es aceptable.

Luego la ecuación tiene cuatro soluciones: x  2 ,  1 , 0 ,  2

6.65
Primera versión.
En primer lugar, vemos que al menos uno de estos tres números tiene que ser 1.
Supongamos, por el contrario, que a , b , c  1 . Entonces podemos escribir
a  x  1 , b  y  1 , c  z  1 con x, y, z  0 . Pero entonces:
abc  a  b  c  1  ( x  1)( y  1)( z  1)  x  1  y  1  z  1  1 
1  x  y  z  xy  yz  xz  xyz  x  y  z  4  xy  yz  xz  xyz  3
Lo cual es imposible pues x, y, z  0  xy  yz  xz  xyz  4

Así pues, podemos suponer que, por ejemplo, b  1 . Entonces la ecuación queda de la forma:
ac  a  c  2  ac  a  c  2  ac  a  c  1  3  (a  1)(c  1)  3
Y los casos posibles son:
a  1  1 a  1  3
  a  2, c  4   a  4, c  2
c  1  3 c 1  1 
Así pues, la única solución es la terna ( 1 , 2 , 4 ) y todas sus permutaciones, pues las incógnitas
a , b , c de la ecuación del enunciado son perfectamente intercambiables.

Segunda versión.
abc  a  b  c  1  abc  a  b  c  1  a(bc  1)  b  c  1 
bc  1 | b  c  1  bc  1  b  c  1  bc  b  c  2  bc  b  c  1  3
 (b  1)(c  1)  3
Veamos los casos posibles:
b  1  1
  b  2, c  4  a  1
c  1  3
b  1  2
  b  3, c  2  a  6 / 5 , y esta solución no es aceptable, pues no es entera.
c 1  1 
b  1  1
  b  2, c  2  a  5 / 3 , y esta solución no es aceptable, pues no es entera.
c  1  1

Y un último caso: b  1  0  b  1 , que ya se consideró en la primera versión.

Tercera versión.
Supongamos que los tres son iguales. Entonces la ecuación queda de la forma
a 3  3a  1 , y entonces:
 
a3  3a  1  a3  3a  1  a a 2  3  1  a | 1  a  1
pero 13  3 1  1, por lo tanto los tres números son son iguales.
Los ordenamos: a  b  c , y al no ser iguales, a  c  a  1  c  a  b  1  b  c  c  c  2c .
Ahora volvemos a la ecuación del enunciado:
abc  a  b  c  1  abc  c  a  b  1  cab  1  a  b  1  c | a  b  1  2c

Solo hay dos posibilidades:


c  a  b  1  ab(a  b  1)  a  b  a  b  1  1 
ab(a  b  1)  2a  2b  2  2(a  b  1)  ab  2  a  1, b  2  c  4
2c  a  b  1  abc  2c  c  3c  ab  3  a  1, b  3  c  5 .
Pero esta solución no es aceptable pues no se satisface la ecuación original:
1 3  5  1  3  5  1
La única solución es a  1, b  2 , c  4 y todas sus permutaciones.
Fuente de las versiones 2 y 3: “Teoría de Números. Entrenamiento de Hidalgo para la Olimpiada Mexicana de matemáticas” pág. 23.

6.66
Si m  0 , la ecuación 1  1  2  3n no tiene solución por el TFA.
Si m  1 , la ecuación 2  1  3n tiene solución n  1 .
Supongamos que m  2 .
2m  1  3n  2m  3n  1 y puesto que m  2 , 4 | 3n  1  3n  1  0 (mod 4)  3n  1 (mod 4)
Pero 3  1 (mod 4)  3n  (1) n  1 (mod 4)  n es par  n  2k
 2
  2
Así pues, 2m  1  32k  3k  2m  3k  1  3k  1 3k  1   
  
Para que 3k  1 3k  1 sea una potencia de 2 es necesario que ambos factores sean una
potencia de dos:
3  1  2
k a

 k con a  b  m
3  1  2b
Pero entonces, sumando las dos igualdades llegamos a 2  3k  2a  2b
Si a, b  2 , entonces 4 | 2a  2b , pero está claro que 4 | 2  3k . Así pues, la menor potencia tiene
que ser 2, es decir, 3k  1  2  3k  3  k  1
La ecuación queda 2m  1  32  9  2m  8  m  3 .
Así pues, las dos únicas soluciones son m  1 , n  1 y m  3 , n  2 .

Fuente de la solución: “Teoría de números: Divisibilidad y Congruencias (Entrenamiento de Hidalgo para la


Olimpiada Mexicana de Matemáticas)”, pág. 25

6.67
Pasando a módulo 8: 8m  7  n2  n 2  7  1 (mod 8)
Las soluciones de la congruencia n 2  1 (mod 8) son: 1 , 3 , 5 , 7 (mod 8) , es decir:

n  8k  1 , n  8k  3 , n  8k  5 , n  8k  7

n2  7
Para cualquier entero k, y en todo caso n  7 será divisible entre 8 y por tanto: m 
2

8
Segunda parte:
n2  7
m  1959  n 2  7  1959  8  n 2  1959  8  7  15665
8
El primer cuadrado superior a 15665 es 1252  15625  15625  7 , pero no sirve pues es impar
y por tanto no es divisible entre 8.
El segundo cuadrado superior a 15665 es 1262  15876  15876  7 , pero no sirve pues es
impar y por tanto no es divisible entre 8.
El tercer cuadrado superior a 15665 es 1272  16129  16129  7  16136  8  2017 , luego
m  2017 .

6.68
En primer lugar, vemos que x  0, y  0 es solución trivial de la ecuación. También vemos que
la parte izquierda es positiva, y por tanto x, y deben tener el mismo signo. Si x, y  es
solución, también lo será  x, y  , así pues, podemos suponer que ambos son positivos.
Vamos a estudiar la presencia del factor 3 en la descomposición factorial de x, y .

Supongamos que x  3m a , y  3n b , con m, n  0 y a, b no divisibles entre 3.


Podemos suponer, además, que m  n .

    
34 23 x 2  y 2  x 3 y 3  34 23 32 m a 2  32 n b 2  33m a 3 33n b3  23 32 m a 2  32 n b 2  33m  3n  4 a 3b3 
23 33 2n
 2m  2n
a b 3
2 2
 3m  3n  4
ab 2 3
3 3 3
 2m  2n
a b 3
2 2
 3m  n  4 3 3
ab
 2

 8 3m  n a  b 2  33m  n  4 a 3b3

Todo cuadrado es 0 o 1 módulo 3 (ver problema #3.16a), luego la suma de dos cuadrados será
0, 1 o 2 módulo 3, es decir, no será divisible entre 3, y por lo tanto, multiplicada por 8 tampoco
será múltiplo de 3. Así pues, la parte de la izquierda no es múltiplo de 3, y por tanto,
observando la parte de la derecha, necesariamente 3m  n  4  0 , y esto solo pasa cuando
m  n  1.

Por lo tanto, la ecuación queda 8 a 2  b2  a3b3 .  


Por simetría podemos suponer que a  b

   
a3b3  8 a 2  b2  8 a 2  a 2  16a 2  ab3  16

Las únicas opciones para b son b  1, 2 .

   
Si b  1  a  16 y 8 a 2  b2  a3b3  8 a 2  1  a3 ningún posible valor de a resuelve esta
ecuación.

Si b  2  a23  16  a 8  16  a  2 . La pareja b  2, a  1 no soluciona 8 a 2  b2  a3b3 .  


Finalmente, la pareja b  2, a  2 sí es solución de 8 a 2  b2  a3b3 :  
 
8 22  22  64  43  2323

Luego x  3m a  3  2  6 , y  3n b  3  2  6 , es la única solución posible (junto a x  0, y  0


y x  6, y  6 ).
Fuente de esta solución: Solución oficial.

6.69
La única combinación aceptable es
y7 
  x  4  7  1  x  1  28  29
x  4 y  1

7.10
Si n  6 , está claro que los factores 3 y 6 están dentro de n ! , luego 6  3  9  2 | n!.
Por lo tanto, solo nos tenemos que ocupar de 1! 2! 3! 4! 5! 153  17  9
Así pues:
1! 2! 3! ...  5! 0 (mod 9)
  1! 2! 3! ...  n! 0  0  0 (mod 9)
6! 7! ...  n! 0 (mod 9) 
Y por tanto el residuo al dividirlo entre 9 es 0.

7.11
Por ejemplo: 62  42 (mod 5) , pero sin embargo, 6  4 (mod 5)

7.12
25  32  4  22 (mod 7)

2  2
50 5
   2 
10 2 10
  4
 
4
(mod 7)  25 (mod 7)  22 (mod 7) 
2  16  2 (mod 7)
4


  2
 24 (mod 7)  22 (mod 7)  4 (mod 7) , luego el residuo es 4.

41  6 (mod 7)  1 (mod 7)  4165 (mod 7)  (1)65 (mod 7)  1 (mod 7)  6 (mod 7) , luego el


residuo es 6.

7.13
  4
211  2048  1 (mod 89)  2 44  2 411  211  14 mod 89  1 mod 89 
2 44  1 (mod 89)  1  1 (mod 89)  0 (mod 89)  89 | 2 44  1

212  4096  22 (mod 97)  2 24  212212  22  22 (mod 97) 


 484 (mod 97)  96 (mod 97)  1 (mod 97) 
 
2 48  2 24 (mod 97)   1 (mod 97)  1 (mod 97) 
2 2

2 48  1 (mod 97)  1  1 (mod 97)  0 (mod 97)  97 | 2 48  1

7.14
9  1mod 10  91003  (1)1003 mod 10  1mod 10  9 mod 10
72  49  1 mod10  7902  72  
451
  1
451
mod10  1 mod10  9 mod10
32  9  1 mod10  3801  3  3800  3  3   2 400
 
 3 mod10  32
400
mod10 
 3 mod10   1 mod10  3 mod10  1 mod10  3 mod10
400

Luego 91003  7902  3801 mod10  9  9  3 mod10  3 mod10 , luego acaba en 3

7.15
11n  2  11n  112 
 n2
  11  12  11 mod 133
n

11  121  12 mod 133


2

12 2 n 1 2n
 
 12  12  12  12
2 n

  12  11 mod133
n

12  144  11 mod 133


2


n2 2 n 1
an  11  12  12 11  12 11n mod 133  0 mod 133  133 | 11n  2  122n 1
n

7.16
Está claro que 2137 2 acaba en 9, luego:
2137 2  9 mod10  1 mod10  2137 2  376
  1
376
 1 mod10

Por otro lado, está claro también que 2137  7 mod10

Por lo tanto
2137753  2137752  2137  2137 2376  2137  2137 2 
376
 2137  1 7 mod10  7 mod10 ,
Luego acaba en 7.

Nota: Este problema también se puede resolver sin congruencias, observando en qué dígito van
acabando las potencias de 2137 n :
n  0 1
n 1 7
n29
n 33

Y para exponentes mayores entramos en un bucle.

7.17
Primera versión.
Estudiamos el dígito de las unidades de 31001:
30  1 , 31  3 , 32  9 , 33  27 , 34  81 , 35  253 , 36  759...

Vemos que se va repitiendo en grupos de 4: {1,3,9,7} , y puesto que 1001  1mod 4 ,


31001 acabará como 31 , es decir en 3.

El mismo análisis hacemos para estudiar el dígito de las unidades de 71002 :


70  1 , 71  7 , 72  49 , 73  343 , 74  2401 ,...
Vemos que se va repitiendo en grupos de 4: {1,7,9,3} , y puesto que 1002  2 mod 4 ,
71002 acabará como 7 2 , es decir en 9.

Finalmente, estudiemos el dígito de las unidades de 131003 :


130  1 , 131  13 , 132  169 , 133  2197 , 134  28561 ,...
Vemos que se va repitiendo en grupos de 4: {1,3,9,7} , y puesto que 1003  3 mod 4 ,
131003 acabará como 133 , es decir en 7.

El producto de un número acabado en 3, un número acabado en 9 y un número acabado en 7 es


un número acabado en 9.

Segunda versión.
Vamos a aprovechar el hecho de que toda potencia de un número acabado en 1 acaba siempre
en 1. En nuestro caso, cualquier potencia de 7  13  91 acaba en 1, y cualquier potencia de
34  81 acaba en 1. Luego:
3100171002131003  13  3100171002131002  13  310017  13  13  31001911002 
1002

 
 13  3  31000911002  13  3  34250911002  13  3  34
250
911002 
 13  3  81250911002  39  3  81250911002

Que claramente acabará en 9.

Fuente de esta versión: The Contest Problem Book V 198S-1988, pág. 62

7.18
  2
Para k  1 : 183  5832  833  7  1  186  183  12  1 (mod 7)
Para k  2 : 183  5832  12  49  1  18  18   1  1 (mod 49)
6 3 2 2

Para k  3 : 183  5832  2  2401  1  18  18   1  1 (mod 2401)


6 3 2 2

Observación: Sin embargo, para k  4 cambia la pauta: 183  1030 (mod 74 )

7.19
Si n es impar, entonces n  2k  1 , y por tanto n2  4k 2  4k  1  4k (k  1)  1
k o k  1 es par, luego k (k  1) es par, y por tanto 4k (k  1) es un múltiplo de 8. Luego
n2  4k (k  1)  1  0  1  1 (mod 8) .

7.20
0  n2  3n  2  (n  1)(n  2) (mod 6)
La tabla de congruencias módulo 6 es suficientemente pequeña para enunciar todas las
combinaciones posibles:
0  2  3  5  6  6  1 (mod 6) , y por tanto: (n  1)  2, 5, 6 (mod 6)  n  1,4, 5 (mod 6)
Y cumpliendo la condición 1  n  25 , tenemos:
n  1(mod 6)  n  1,7,13,19,25
n  4 (mod 6)  n  4,10,16,20
n  5 (mod 6)  n  5,11,17,23
En total, 5  4  4 posibilidades.

7.21
Primera versión.
Vemos el comportamiento de 2n  6  9n para los primeros valores de n:
Antes de nada, observamos que 6  1 (mod 7)
n  1  21  2 6  91  9  2 (mod 7)  2n  6  9n  2  2  0 (mod 7)
n  2  22  4 6  92  81  4 (mod 7)  2n  6  9n  4  4  0 (mod 7)
n  3  23  8  1 (mod 7) 6  93  729  1 (mod 7)  2n  6  9n  1  1  0 (mod 7)

Y, a partir de este valor, se van repitiendo la pauta:


n  4  24  232  1  2 (mod 7)  2 (mod 7)
6  94  93  9  1  9  9 (mod 7)  2n  6  9n  2  2  0 (mod 7)
n  5  25  2322  1  22 (mod 7)  22 (mod 7)
6  95  93  92  1  92  92 (mod 7)  2n  6  9n  4  4  0 (mod 7)
n  6  26  2323  1  23 (mod 7)  23 (mod 7)
6  96  93  93  1  93  93 (mod 7)  2n  6  9n  1  1  0 (mod 7)
Y siempre llegamos al mismo resultado: 2n  6  9n  0 (mod 7) .

Segunda versión.
Partimos de la igualdad 9  2 (mod 7) , luego 9n  2n (mod 7) , y por tanto:
2n  6  9n  9n  6  9n  7  9n  0  9n  0 (mod 7)

7.22
Primera versión.
Vamos a calcular 6  8 (mod 49) directamente, mediante el método de las potencias de
83 83

dos:
61  6 (mod 49)
62  36 (mod 49)
64  36  36  1296  22 (mod 49)
68  22  22  484  43 (mod 49)
616  43  43  1849  36 (mod 49)
632  36  36  1296  22 (mod 49)
664  22  22  484  43 (mod 49)
83  64  16  2  1  683  664616626  43  36  36  6
 43  22  6  15  6  41 (mod 49)

81  8 (mod 49)
82  64  15 (mod 49)
84  15  15  225  29 (mod 49)
88  29  29  841  8 (mod 49)
816  8  8  64  15 (mod 49)
832  15  15  225  29 (mod 49)
864  29  29  841  8 (mod 49)
83  64  16  2  1  883  864816828  8  15  15  8 
 15  29  435  43 (mod 49)
Finalmente, 6  8  41  43  84  35 (mod 49)
83 83

Segunda versión.
Aplicando el Teorema del Binomio:
 n  n  n  1 n 1
(a  b)n  a n   a n 1b1   a n  2b 2  ...   a b  b n
1  2  n  1

Aplicado a nuestro caso, y teniendo en cuenta que n  83 es impar, y por tanto (1)n  1 ,

 n n  n 
6n  (7  1)n  7 n    7 n 1    7 n  2  ...    7  1
1  2  n  1
 n n  n 
8n  (7  1) n  7 n    7 n 1    7 n  2  ...    7  1
1  2  n  1

Y por tanto:
n n  n  3  n 
6n  8n  7 n    7 n  2    7 n  4  ...    7    7 
 2  4  n  3  n  1
 n n  n  3 
 2 7 n    7 n  2    7 n  4  ...    7  7n  
  2  4  n  3 
 n n  n  
 2  7 2  7 n  2    7 n  4    7 n  6  ...    7   14n  49k  14n
  
2  
4  n  3  

En particular, para n  83, a83  683  883  49k  14  83  14  83 (mod 49)


Luego solo nos queda calcular este último residuo:

14  83  1162  49  23  35  14  83  35 (mod 49)

Fuente de la segunda versión: The Contest Problem Book V 1983-1988 (George Berzsenyi, 1997).

Tercera versión.
Aplicando el Teorema de Euler (que se introducirá en el Tema 11)
 1
49  7 2   (49)  491    42 , y a 42  1 (mod 49) si (a, n)  1 .
 7
En nuestro caso (6,49)  (8,49)  1 , y por tanto:
  2
642  1 (mod 49)  683  62421  642 61  61 (mod 49)
842  1 (mod 49)  883  82421  8  8
42 2 1
 81 (mod 49)
1 1 8  6 14
683  883  61  81      14  35 (mod 49)
6 8 48 1
Fuente de esta versión: https://artofproblemsolving.com/wiki/index.php/1983_AIME_Problems/Problem_6
7.23
 
9  99  999  ...  99  00  1 
 9  9  99  (1000  1)  (10000  1)  ...  100
... 
 ... 
999 nueves  999 ceros 
3 4

 9  99  (10  1)  (10  1)  ...  10  1
999

Y observamos que si k  3  10k es múltiplo de 1000, y por tanto 10k  0 (mod1000) , luego
10k  1  0  1  1 (mod1000)
Y por tanto:
 
(103  1)  (104  1)  ...  10999  1  (1)99931  (1)997  1 (mod1000)
Y, finalmente,
 
9  99  (103  1)  (104  1)  ...  10999  1 mod(1000)  9  99(1) mod(1000) 
 891 mod(1000)  109 mod(1000)

7.24
Calculamos directamente los primeros valores de f (n) :
f (0)  0 , f (1)  2 , f (3)  6 , f (4)  14 , f (5)  30 , f (6)  62

Analizando cómo se obtiene f (n) llegamos a la conclusión de que


f (n)  2 f (n  1)  2(n  1)  2n  2 f (n  1)  2

Que es el típico comportamiento de una función exponencial. Mirando los primeros valores
vemos que un buen candidato puede ser f (n)  2n  2 .
Lo vamos a demostrar por inducción:
Para n  1,2,3 es cierto.
Suponiendo que f (n)  2n  2 , entonces
f (n  1)  2 f (n)  2  2(2n  2)  2  2n 1  4  2  2n 1  2 .
Luego es cierto para todo n.

f (100)  2100  2 , y queremos determinar 2100  2 (mod100) .


Calculamos 2100 (mod100) con el “Método de las potencias de 2”:
22  4 (mod100)
24  16 (mod100)
28  16  16  256  56 (mod100)
216  56  56  3136  36 (mod100)
232  36  36  1296  96  4 (mod100)
264  (4)  (4)  16 (mod100)
Luego
2100  264 32 4  26423224  16  (4)  16 (mod100)  56  (4) (mod100)
 224 (mod100)  24 (mod100)  24 (mod100)  76 (mod100)

Finalmente: 2100  2  76  2 (mod100)  74 (mod100) , y el residuo pedido es 74.


7.25
Queremos determinar k 2  2k (mod10) . Estudiemos los residuos 2n (mod10) :
21  2 (mod 10)
22  4 (mod 10)
23  8 (mod 10)
24  4  4  16  6 (mod 10)
25  6  2  12  2 (mod 10)
26  2  2  4 (mod 10)
27  4  2  8 (mod 10)
28  8  2  6 (mod 10)
...
En general: 2n (mod10) genera un ciclo 2,4,8,6 , y en particular, para todo n  4k múltiplo
de 4, 24k  6 (mod10) .

En particular, en nuestro caso:


4 | 2008  4 | 20082 

  4 | 2008  2 k
2 2008

2 2008
2  
2 4 251
 2 2 4 251
4 4 251
 4|2 
2008

k es un múltiplo de 4, y por tanto 2k  6 (mod10) .
También vemos que 2008 es múltiplo de 4, luego 22008  6 (mod10) .
Por otro lado, 2008  8 (mod10)  20082  82  64  4 (mod10)
Y por tanto k  20082  22008  4  6  10  0 (mod10)  k 2  02  0 (mod10)
Finalmente, k 2  2k (mod10)  0  6  6 (mod10) , y el dígito de las unidades es 6.

Fuente de la solución: https://artofproblemsolving.com/wiki/index.php/2008_AMC_12A_Problems/Problem_15

7.26
n y 107n tienen las dos últimas cifras iguales si y solo si
107n  n (mod 100)  7n  n (mod 100)  6n  0 (mod 100)  6n  100k
 3n  50k
De aquí deducimos que n es un múltiplo de 50. El múltiplo de 50 más pequeño es n  50 y ya
satisface la condición del enunciado, pues 107  50 acaba en 50.

7.27
Pasando a módulo 3 la ecuación se convierte en x 2  y 2  0
Hacemos la tabla x 2  y 2 (mod 3)

x0 x 1 x2
y0 0 1 1
y 1 1 2 2
y2 1 2 2

Vemos que la única posibilidad es x  0 , y  0 , es decir, x, y múltiplos de 3. Pero entonces:


2 2
   
3z 2  x 2  y 2  3x'  3 y'  9 x'2 9 y'2  9 x'2  y'2  z 2  3 x'2  y'2 , es decir, z 2 es
múltiplo de 3, y por tanto z es múltiplo de 3.

2 2 2
 
Pero entonces x 2  y 2  3z 2  3x'  3 y'  33z'  9 x'2  y'2  9  3z'2  x'2  y'2  3z'2

Este proceso lo puedo repetir una y otra vez, y esto solo puede ocurrir si x  y  z  0 .

7.28
n  182  n2  36n  324 .
Realizamos la división sintética n 2  36n  324 entre n  2 obtenemos un cociente igual a
n  34 y un residuo igual a 256, luego n  18  256 (mod n  2)
2

Así pues
n  2 | n  18  n  18  0 (mod n  2)  256  0 (mod n  2)
2 2

 n  2 | 256  28

Luego n  2 es una potencia 2 k , 0  k  8


1  n  1 , 21  2  n  0 , 22  4  n  2 , 23  8  n  6 , 24  16  n  14 ,
25  32  n  30 , 26  64  n  62 , 27  128  n  126 , 28  256  n  254
Se comprueba que todos estos valores satisfacen la condición del enunciado.

7.29
Observamos la secuencia de potencias 2n (mod 7) :
21  2 (mod 7)
22  4 (mod 7)
23  1 (mod 7)
...
Siempre es 1, 2 o 4 y por tanto 2n  1 será congruente con 2 , 3 o 5, nunca con 0.

7.30
72  49  1 mod 10 , luego 74   1  1 mod 10 , y por tanto
2

77  7  73  74  7  (1)  1  7mod 10


 
Y por tanto 77   7   1 7   7  7 mod 10
7 7 7 7

Así pues, por cada pareja de potencias de 7 el resultado es 7, y como elevamos 1000 veces, que
es par, el resultado es 7.

7.31
Está claro que pasando estos números a módulo 3 solo pueden haber tres diferentes, luego
forzosamente encontraremos una pareja equivalente, digamos, por ejemplo, que
a  b (mod 3)  3 | a  b

Y ya tenemos un factor divisible entre 3 en el producto del enunciado.


De la misma forma, pasando a módulo 2 solo pueden haber 2 números diferentes, luego puede
suceder uno de los siguientes dos casos:
a  b (mod 2)  2 | a  b
a) 
b  c (mod 2)  2 | b  c
a  b (mod 2)  2 | a  b
b) 
c  d (mod 2)  2 | c  d

En todo caso, encontramos dos factores diferentes múltiples de 2, con lo que el resultado será
múltiple de 4. Puesto que 3 y 4 son coprimos, el producto del enunciado será múltiple de
3  4  12 .

7.35
Sabemos que n  ck ck 1 ck 2 ... c1 c0 es equivalente a
n  ck 10k  ck 110k 1  ck 210k 2  ...  c110  c0
Es decir,
n  p(10) con p( x)  ck x k  ck 1 x k 1  ck 2 x k 2  ...  c1 x  c0 .

Y que las suma de sus cifras será S  p(1) .


Por otro lado, 9  10  1  9 | 10  1  9 | p(10)  p(1)  n  S  n  S (mod 9)

Luego 9 | n  n  0 (mod 9)  S  0 (mod 9)  9 | S

Sabemos que n  ck ck 1 ck 2 ... c1 c0 es equivalente a


n  ck 10k  ck 110k 1  ck 210k 2  ...  c110  c0
Es decir,
n  p(10) con p( x)  ck x k  ck 1 x k 1  ck 2 x k 2  ...  c1 x  c0 .

Y que la suma alternada de sus cifras es T  p(1) .

Por otro lado, 10  1 (mod 11)  p(10)  p(1) (mod 11) , es decir n  T (mod 11) .
Luego 11 | n  n  0 (mod 11)  T  0 (mod 11)  11 | T

7.44
Denotando p a las personas, h a los caballos, s a las ovejas, c a las vacas y d a los patos,
tenemos
p  3h 
 d  9h 
s  4c     T  p  h  s  c  d  3h  h  4c  c  9h  13h  5c
 s  4c 
d  3 p

Aplicando ahora el teorema “Chicken McNugget” (7.43), el valor máximo no representable es


T  13  5  13  5  47 , luego la solución es (B).

7.45
El problema que se plantea es estudiar todas las combinaciones posibles
x  5  y  n  z  n  1

con enteros x, y, z  0

Y determinar aquellos enteros positivos n para los cuales 91 no se puede formar pero sí se
pueden formar 92, 93, 94…

Está claro que, independientemente del valor n , se pueden formar todos los múltiplos de 5,
pues basta ir dando valores a x , con y  z  0 .

Para un valor n cualquiera, está claro que podremos formar seguro todos los valores de la
forma n , n  5 , n  10 , n  15 ,… También todos los valores de la forma n  1 ,
n  1  5  n  6 , n  1  10  n  11 ,…

Todo esto nos indica que la clave para resolver este problema es pasar a módulo 5:

1 2 3 4 5
6 7 8 9 10
11 12 13 14 15

91 92 93 94 95
96 97 98 99 100

Aplicamos el Teorema “Chicken McNugget”, a las combinaciones 5 y n:


91  5n  5  n  n  24
Luego podemos garantizar que n  24 .

Primer caso: n  1 mod 5


Supongamos que n se encuentra en la primera columna, es decir, que n  1 mod 5 .
Vemos que 91 también está en la primera columna, luego nos obliga a n  91 , es decir:
n  96 ,101, ...
Pero entonces no podríamos obtener 92 (ni 93, 94…) con lo que dicho número no es aceptable.

Segundo caso: n  2 mod 5

Si n se encuentra en la segunda columna, n  1 se encontrará en la tercera, y por tanto 2n se


encontrará en la cuarta. En efecto:
n  5k  2  2n  25k  2  10k  4  4 mod 5

También vemos que 2n  1  2n  2  1 mod 5 se puede obtener, y está en la primera


columna.
Luego 2n  1  91, y el primer candidato interesante sería el siguiente: 2n  1  96

2n  1  96  n  47
Comprobamos que, efectivamente, n  47 satisface las condiciones del enunciado:
Están todos los múltiplos de 47, en particular 92  47  45  47  9  5 , todos los múltiplos de
48, 2  47  94 y todos los múltiplos de 94, y observamos que 91 no se puede representar como
combinación de 47, 48 y 5, pero sí podemos representar 96  2(47  1)  2  48 .

1 2 3 4 5
6 7 8 9 10
11 12 13 14 15

47 48

91 92 93 94 95
96 97 98 99 100

Tercer caso: n  3 mod 5

n  3 mod 5  2n  6  1 mod 5 , y por tanto 2n se encontrará en la primera columna. Como


en el caso anterior, deducimos que 2n  91 y el primer candidato interesante sería el siguiente:
2n  96  n  48

Pero con este valor nos encontramos con un problema en la segunda columna: No se puede
representar 92, y por tanto no es válido.

En efecto, todo consiste en ir viendo las posibles combinaciones:


x  0, y  1, z  1  x  5  y  48  z  49  97
x  1, y  1, z  1  x  5  y  48  z  49  102
x  9, y  1, z  0  x  5  y  48  z  49  93

Cuarto caso: n  4 mod 5

En este caso n  1  5 mod 5 y es redundante, 2n  8  3 mod 5 , 3n  12  2 mod 5 y


4n  1 mod 5 . Como en los casos anteriores, imponemos la condición

4n  96  n  24

Y vemos que con este valor todas las cinco columnas están cubiertas, es decir, es una segunda
solución al problema.

Quinto caso: n  5  0 mod 5


En este caso tendríamos n  1  1 mod 5 , es decir, en la primera columna, y por tanto
n  1  91  n  92 . El primer candidato sería n  95 , pero con este valor no podríamos formar
92 (ni 93…), luego no es aceptable.

Así pues, las soluciones de este problema son n  24 y n  47 .

Fuente de esta solución: Vídeo “Momentum Learning” https://youtu.be/fTZP2e-_rjA


8.4
d  (9,30)  3 , y 3 | 21 , luego la ecuación anterior tendrá 3 soluciones diferentes.

Encontramos la primera solución por tanteo:


x0  9 , pues 9  9  81  2  30  21.
Luego el resto de soluciones serán:
30
x1  9   1  9  10  19 , efectivamente: 9  19  171  5  30  21
3
30
x2  9   2  9  20  29 , efectivamente: 9  29  261  8  30  21
3

8.5
(3,10)  1 , luego la congruencia lineal anterior tendrá una única solución módulo 10.
Aunque no sea la más elegante, una manera de resolverla es ir probando números del 1 al 9:
3  1  3  7 mod 10 , 3  2  6  7 mod 10 , 3  3  9  7 mod 10 ,
3  4  12  2  7 mod 10 , 3  5  15  5  7 mod 10 , 3  6  18  8  7 mod 10 ,
3  7  21  1  7 mod 10 , 3  8  24  4  7 mod 10 , 3  9  27  7 mod 10 .

Luego la solución es x  9 .

8.8
Observamos que 9  9  81  1 (mod 82) , luego 9  (9)  81  1 (mod 82) . El inverso de 9
módulo 82 es  9  82  9  73 (mod 82) .

8.18
Primera versión: Mediante el método de las potencias de dos.
Vamos calculando 10322  (mod 100) :
n

10321  32 (mod 100)


10322  32  32  24 (mod 100)
10324  24  24  76 (mod 100)
10328  76  76  76 (mod 100)
...

Observamos que 10322   76 (mod 100) para todo n  2 .


n

Por otro lado,


1032  1024  4  2  10321032  10321024 8  10321024  10328 
10321032 (mod100)  10322  10322 (mod100)  76  76 (mod100)  76 (mod100)
10 3

Segunda versión: Mediante el Teorema Chino del Residuo.


1032  0 (mod 4)  10321032  0 (mod 4)
1032  7 (mod 25)  10322  49  1 (mod 25) 

 10321032  10322516  10322 516
 (1)516  1 (mod 25)

Nos queda el siguiente sistema de congruencias que resolveremos mediante el Teorema Chino
del Residuo:

1032  0 (mod 4)
1032



1032  1 (mod 25)
1032

N  4  25  100 

N1  25  25 y1  1 (mod 4)   x  25  y1  0  4  19  1  76
N 2  4  4 y2  1 (mod 25)  y2  19

Y por tanto: 10321032  76 (mod 4  25) , tal como queríamos ver.

9.2
Primera versión.
2005  5 (mod 1000)
20052  25 (mod 1000)
20053  25  2005  25  5  125 (mod 1000)
20054  125  2005  125  5  625 (mod 1000)
20055  625  2005  625  5  125 (mod 1000)
20056  125  2005  125  5  625 (mod 1000)


2005  125 (mod 1000) si n es impar
n

Vemos la pauta: n  3  

2005  625 (mod 1000) si n es par
n

En N  200511  200512  ...  20052006 hay 1996 sumandos: 998 pares y 998 impares, luego,
tabajando módulo 1000, tenemos:
N  998  125  998  625  998(125  625)  998  750
Haciendo esta última multiplicación (no hace falta hacerla entera) vemos que acaba en 500.

Segunda versión.
Ante todo vemos que 2005  5 (mod1000)

Por un lado calculamos el sumatorio módulo 125:


Si k  3  5k  10(mod 125) , luego el sumatorio es cero.

Por otro lado, calculamos el sumatorio módulo 8:

Observamos que 52  1 (mod 8)


Luego 5k  1 (mod 8) si es par, y 5k  5 (mod 8) si k es impar, luego
1996
511  512  ...  52006  (1  5)  1996  3  5988  4 (mod 8)
2
Ahora aplicamos el Teorema Chino del Residuo:
 N  125  8
x  0 (mod 125)  N1  8

x  4 (mod 8)   N 2  125  125 y  1 (mod 8)  y  5
 x  8  0  y1  125  5  4  2500  500 (mod 1000)

9.3
Si x no es divisible entre tres entonces x  1 mod 3 o x  2 mod 3 .
x  1 mod 3  x  3k  1  x 2  3k  1  9k 2  6k  1  3k (3k  2)  1  x 2  1 mod 3
2

x  2 mod 3  x  3k  2  x 2  3k  2  9k 2  12k  4  3(3k 2  4k  1  1  x 2  1 mod 3


2

En todo caso, x 2  1 mod 3 .

Si x es impar, x  2k  1  x 2  2k  1  4k 2  4k  1  4k (k  1)  1 , aquí k o k+1 es par, y


2

por tanto 4k (k  1) es múltiplo de 8, y por lo tanto x 2  1 mod 8 .


Así pues,
 x  1 mod 3
2

 2
 x  1 mod 8
Y ahora aplicamos el Teorema chino del residuo:
N  8  3  24
N1  3  3 y1  1mod 8  y1  3
N 2  8  8 y2  1mod 38  y2  5
x  3  3  1  8  5  1  9  40  49
x  49  1mod 24

10.1.1
Sabemos que existirá solución pues (7,9)  1 .
7 x  9 y  3  7 x  3  9 y  31  3 y   3 | x  x  3k 
7  3k - 9y  3  7k - 3y  1  7k  1  3 y
Claramente una solución de esta última ecuación es y  2, k  1, luego y  2, x  3, k  3 será
una solución particular de la ecuación del enunciado.
En general, las soluciones serán de la forma y  2  9k , x  3  7k , con k  Z

10.1.2
Sean N a y N b la cantidad de manzanas y naranjas compradas, respectivamente.
Sean Pa y Pb el precio de cada manzana y de cada naranja, respectivamente, en céntimos.

Tenemos las siguientes condiciones:


 N a  N b  12
P  P  3
 a b

N a  Nb
 Pa N a  Pb N b  132
Luego
Pb  3N a  Pb 12  N a   132  Pb N a  3N a  12Pb  Pb N a  132 
3N a  12 Pb  132  N a  4 Pb  44

Esta última ecuación diofántica tiene solución pues d  mcd (1,4)  1 , y 1 | 44 .

Una solución particular de esta ecuación es N a  4, Pb  10 , pero no satisface la condición


N a  N b . El resto de soluciones son de la forma
4
N a  4  k , Pb  10  k
1
k  0  N a  4, Pb  10  N b  8
k  1  N a  8, Pb  9  N b  4, Pa  12
k  2  N a  12, Pb  8  N b  0, Pa  11

Las soluciones posibles son 4 naranjas a 9 céntimos, (con 8 manzanas a 12 céntimos) o 12


manzanas a 11 céntimos (y sin ninguna naranja)

10.1.3
Aplicamos el algoritmo de Euclides:
858  3  253  99
253  2  99  55 

99  1  55  44   (858,253)  11 , y 11 | 33 , luego la ecuación tiene solución.
55  1  44  11 

44  4 11 

Deshacemos los pasos del algoritmo de Euclides:


11  55  44 
44  99  55 

  11  55  44  253  2  99  99  55  253  2  99  99  55 
55  253  2  99 
99  858  3  253
 253  3  99  253  2  99  2  253  5  99  2  253  5858  3  253 
 2  253  5  858  15  253  5  858  17  253

Y multiplicando ambos lados por 3 obtenemos la solución deseada:


11  5  858  17  253  33  15  858  51  253  x  15, y  51

El conjunto general de soluciones serán todas las parejas de la forma:


253 858
x  15  k  15  23k  8  23k , y  51  k  51  78k
11 11

10.1.4
Calculamos el (858,253) mediante el algoritmo de Euclides:
258  147  111
147  111  36 
  (858,253)  3 , y 3 | 369 , luego la ecuación tiene solución.
111  36  3  3 
36  3 12 

Deshaciendo los pasos del algoritmo de Euclides:


3  111  3  36 

36  147  111   3  111  3  36  258  147  3147  111  258  147  3  147  3  111 
111  258  147
 258  4  147  3  111  258  4  147  3258  147   258  4  147  3  258  3  147 
 4  258  7  147
Multiplicando por 123 tenemos 369  123  3  492  258  861  147
luego una solución concreta es x  492, y  861 , y el conjunto de soluciones es
x  492  49k , y  861  86k

10.1.5
x  15  11k , y  27  20k

10.1.6
La sucesión será de la forma a , a  k , a  2k , a  3k , a  4k , a  5k para un cierto ángulo inicial
a, y sabemos que los ángulos internos de un hexágono suman (6  2)  180º  720º , así pues:
a  a  k  a  2k  a  3k  a  4k  a  5k  720  6a  15k  720  2a  5k  240

Esta última ecuación diofántica tendrá solución pues mcm(2,5)  1 .


Una solución es a  100 , k  8 , con lo que el conjunto de soluciones será de la forma:
a  100  5n
 , con n  Z
b  8  2n

Tenemos, además, la restricción 0  a  180 , luego


0  100  5n  180  100  5n  80  20  n  16

El valor máximo se toma con n  15  a  100  5  15  175º .

10.1.7
Primera versión.
Sea n dicho número.
n  10a  9
  10a  9  9b  8  10a  9b  1
n  9b  8 
Esta ecuación diofántica tiene por solución
a  1  9k 
  n  10(1  9k )  9  90k  1 , o equivalentemente, n  90c  1
b  1  10k 

Añadimos una condición más:


n  90c  1
  90c  1  8d  7  90c  8d  8  45c  4d  4
n  8d  7 
Esta ecuación diofántica tiene por solución:
c  4e 
  n  90(4e)  1  360e  1
d  45e  1

Añadimos una condición más:


n  360e  1
  360e  1  7 f  6  360e  7 f  7
n7f 6 
Esta ecuación diofántica tiene por solución:
e  7g 
  n  360(7 g )  1  2520 g  1
d  1  360 g 

Con este resultado ya podemos buscar candidatos, y vemos que con g  1  n  2519 ya
cumple todas las condiciones del enunciado.

Segunda versión.
Sea n el número buscado. Este número se puede escribir como
n  10a9  9  9a8  8  8a7  7  ...  2a1  1

Pero entonces:
n  1  10(a9  1)  9(a8  1)  8(a7  1)  ...  2(a1  1) , es decir:
2,3,4,5,6,7,8,9 | n  1  2520  (2,3,4,5,6,7,8,9) | n  1  n  1  2520k  n  2520k  1

De nuevo, comprobamos que, con k  1 , n  2519 cumple todas las condiciones exigidas.

10.2.1
Para resolver este problema vamos a utilizar que todo cuadrado perfecto es 0 o 1 módulo 3 (ver
Problema 3.22).

Entre el conjunto de todas las soluciones posibles, tomaremos aquella tal que a  b  c sea
mínimo.

Pasando a módulo 3, a 2  2b2  3c 2  2b2 (mod 3) .


Puesto que b 2 es 0 o 1 módulo 3, 2b 2 será 0 o 2 módulo 3. Puesto que es un cuadrado perfecto,
no podrá ser 2 módulo 3, por lo que la única posibilidad es que sea 0 módulo 3:

b2  0 (mod 3)  3 | b2  3 | b

3 | b  3 | 2b 2 

  3 | a  2b  3c  3 | a
2 2 2

3 | 3c 2

3 | b  b  3b'  b 2  9b'2 

2
 
 3c 2  2b 2  a 2  2  9b'2 9a'2  9 2b'2 a'2  9 | 3c 2  3 | c 2
3 | a  a  3a'  a  9a' 
2

3 | c 2  3 | c  c  3c'

2 2 2
b c b2 c2 1 1

a
Luego 2b'2 3c'2  2   3   2  3  2b 2  3c 2  a 2     a'2 
3  3 9 9 9 9 3

Es decir, la terna (a' , b' , c' ) también es solución de la ecuación del enunciado, pero
a b c
a '  b'  c '     a  b  c
3 3 3
Contradiciendo la hipótesis de que esta suma era mínima.
Así pues, no existe ninguna solución a esta ecuación que no sea la trivial a  b  c  0

Fuente de la solución: Solución oficial (OME, página 247)

10.3.1
Siguiendo los razonamientos de los teoremes de este apartado, distinguiremos los siguientes
casos:
a) Si n no es un cuadrado perfecto.
a1) Si n es impar.
El número de soluciones positivas será
1 n
 (n)    i  1
2 i 1
Luego
1 n n
43  47    i  1  2  43  47    i  1
2 i 1 i 1
Y el valor de n más pequeño lo encontraremos tomando los factores primos más pequeños con
los exponentes más pequeños posibles (y en orden inverso):
n  3471 5431 721  346 542 7

a2) Si n es par.
El número total de soluciones será
n n
 (n)  (1  1) i  1  2021  43  47  (1  1) i  1  2  43  47
1
2 i 2 i 2
Y el número n más pequeño que podemos conseguir será
n  2471 3431 52 1  248 342 5

b) Si n es un cuadrado perfecto.
b1) Si n es impar.
n
La fórmula  
i 1
i  1 determina el total de soluciones con x positiva, que pueden ser de la

forma ( x, y) o ( x, y) . Entre ellas tenemos la solución la solución  


n , 0 , que no se
contempla en este problema, luego:
n

 
i 1
i  1  2  2021  1  13  311

Y el número n más pequeño que podemos conseguir será n  3310  512


b2) Si n es par.
n
La fórmula (1  1)  i  1 determina el total de soluciones con x positiva, que pueden ser
i 2

de la forma ( x, y) o ( x, y) . Entre ellas tenemos la solución la solución  


n , 0 , que no se
contempla en este problema, luego:
n
(1  1) i  1  2  2021  1  13  311
i 2

Los valores de n que vamos obteniendo: 214 p310 , 2312 p12 , 24044 son todos mayores que el
encontrado en el apartado a2 (falta por comprobar).

Así pues, el menor número olímpico es n  248 342 5 y si exigimos que además sea impar es
n  346 542 7 .

Fuente: Soluciones oficiales (OME, pág. 886)

10.3.2
a  a 2  8a
Las raíces de f ( x)  x  a x  2a son x 
2

Luego a 2  8a debe ser un cuadrado perfecto: a 2  8a  b2


a 2  8a  b 2  a 2  8a  16  b 2  16  a  4  b 2  16 
2

a  42  b2  16  a  4  ba  4  b  16
Resolvemos todas las combinaciones posibles:
a  4  b  1 a  25 / 2 a  4  b  2 a  9
   
a  4  b  16 b  15 / 2 a  4  b  8 b  3
a  4  b  4 a  8 a  4  b  8 a  9
   
a  4  b  4 b  0 a  4  b  2 b  3
a  4  b  16 a  25 / 2 a  4  b  1 a  9 / 2
   
a  4  b  1 b  15 / 2 a  4  b  16 b  15 / 2
a  4  b  2 a  1 a  4  b  4 a  0
   
a  4  b  8 b  3 a  4  b  4 b  0
a  4  b  8 a  1 a  4  b  16 a  9 / 2
   
a  4  b  2 b  3 a  4  b  1 b  15 / 2

Las soluciones aceptables son 9, 8, 0 y -1 , luego la respuesta correcta es 9  8  0  (1)  16


(C)

Observación.
Una manera mucho más elegante de resolver la ecuación
a  4  ba  4  b  16
Es la siguiente:
a4b u
a  4  ba  4  b  u  v    2(a  4)  u  v  a 
uv
4
a  4  b  v 2

Luego
17 10
u  1 , v  16  a  4 u  2,v 8a  49
2 2
8  17
u 4,v 4a  48 u  1 , v  16  a  4
2 2
 10 8
u  2 , v  8  a   4  1 u  4 , v  4  a  40
2 2
Fuente de esta solución: https://artofproblemsolving.com/wiki/index.php/2015_AMC_10A_Problems/Problem_23

10.4.1
x 2  y 2  3z 2  x 2  y 2  0 (mod 3) .
Ahora aplicamos el resultado de que todo cuadrado es 0 o 1 módulo 3, luego
x 2  0 (mod 3) , y 2  0 (mod 3)  x 2  y 2  0  0  0 (mod 3)
x 2  1 (mod 3) , y 2  0 (mod 3)  x2  y 2  1  0  1 (mod 3)
x 2  0 (mod 3) , y 2  1 (mod 3)  x 2  y 2  0  1  1 (mod 3)
x 2  1 (mod 3) , y 2  1 (mod 3)  x2  y 2  1  1  2 (mod 3)

La única posibilidad válida es x 2  y 2  0 (mod 3)  x  y  0 (mod 3)

Luego
 
x  3x' , y  3 y'  x 2  y 2  (3x' ) 2  (3 y' ) 2  9 x'2 9 y '2  9 x'2  y '2  3z 2 
 
3 x'2  y'2  z 2  3 | z 2  3 | z  z  3z '  z 2  3z '  9 z '2
2

Luego:
 
3 x'2  y'2  9 z '2  x'2  y'2  3z '2

x y z
Llegando a la misma ecuación para valores x' , y ' , z '   , ,  estrictamente más pequeños
 3 3 3
que los primeros. Aquí aplicamos el Principio del Descenso Infinito para deducir que esta
ecuación no tiene ninguna solución exceptuando la trivial x  y  z  0

10.4.2
a2  b2  c2  a2b2  c2  a2b2  a2  b2

Puesto que todos los elementos involucrados son cuadrados, podemos reducir nuestro estudio a
las soluciones positivas, y después duplicar las soluciones encontradas.

Vamos a utilizar que todo cuadrado es 0 o 1 módulo 4.


a 2  0 (mod 4), b 2  0 (mod 4)  a 2b 2  a 2  b 2  0  0  0  0  0 (mod 4)
a 2  1 (mod 4), b 2  0 (mod 4)  a 2b 2  a 2  b 2  1  0  1  0  1  3 (mod 4)
a 2  0 (mod 4), b 2  1 (mod 4)  a 2b 2  a 2  b 2  0 1  0  1  1  3 (mod 4)
a 2  1 (mod 4), b 2  1 (mod 4)  a 2b 2  a 2  b 2  1 1  1  1  1  3 (mod 4)

Así pues, la única posibilidad aceptable es: a 2  0 (mod 4) y b2  0 (mod 4) .


Escribamos a 2  4a' y b2  4b' para ciertos a' , b' .
Luego c 2  a 2b2  a 2  b2  4a'4b'4a'4b'  44a' b'a'b'  22 4a' b'a'b'

y por tanto 4a' b'a'b' debe ser tambíén un cuadrado, y por tanto 0 o 1 módulo 4.

Pero observamos que este proceso lo estamos repitiendo una y otra vez, obteniendo una cadena
decreciente de divisores de 4, con lo que, aplicando la técnica del "descenso infinito de
Fermat", llegamos a la conclusión de que la única solución aceptable es a  b  0 , y por tanto
también c  0 .

11.1
Queremos ver que 7n  3 (mod 10)  7n  43 (mod 100)
Vamos viendo el comportamiento de las potencias de 7 módulo 100:
70  1
73  63  3 (mod 10)
7 7
1

7 4  21  1 (mod 10)
7  49  9 (mod 10)
2

Vemos que hay un bucle de longitud 4, y que 7n  3 (mod 10)  n  3 (mod 4) .


Veamos ahora como son las potencias de 7 módulo 100:
70  1
73  343  43 (mod 100)
71  7
7 4  301  1 (mod 10)
7 2  49

Vemos que también hay un bucle de longitud 4, y que 7n  43 (mod 100)  n  3 (mod 4) ,
luego 7n  3 (mod 10)  n  3 (mod 4)  7n  43 (mod 100) .

11.2
2222  3 (mod 7)
32  9  2 (mod 7)
33  6  1 (mod 7)

5555  3  1851  2
 
22225555  35555  331851 2  33 3   1
1851 2 1851
2  2  5 (mod 7)

5555  4 (mod 7)
42  16  2 (mod 7)
43  8  1 (mod 7)

2222  3  740  2
 
55552222  42222  43740 2  43 4  1
740 2
2  2 (mod 7)
1851

22225555  55552222  5  2  0 (mod 7)

11.3
n2  2 n4
an  2 1 2
n n2 n n2

Vemos que la sucesión está bien definida pues el denominador es siempre superior o igual a 2.

n4
Los valores diferentes de an  serán los mismos que de bn 
n n2
2

b4  0 y es el único cero de la sucesión.


1 n2  n  2
Si n  4 , los valores diferentes de bn  serán los mismos que de cn   .
bn n4
n2  n  2 14
cn   n 3
n4 n4

Veamos los elementos iguales de la sucesión cn :

14 14 14 14
cn  cm  n  3   m3 n m
n4 m4 n4 m4
14 14  1 1  n4m4 nm
 nm   14    14  14
m4 n4 m4 n4 m  4n  4 m  4n  4
 m  4n  4  14  7  2
1
 1  14
m  4n  4
Llegamos a una ecuación diofántica, que tiene dos soluciones positivas:
m  4  1 m  5 m42m6
,
n  4  14  n  18 n  4  7  n  11

Luego a5  a18 y a6  a11 , y el número de valores distintos es 98.

Observación.
En la solución oficial se sigue el siguiente razonamiento:
n4 m4
 2  (n  m)(nm  4n  4m  2)  0 
n n2 m m2
2

nm  4n  4m  2  14  14  (n  4)(m  4)  14

Llegando a las mismas dos soluciones: (5,18) y (6,11) .

11.4
Primera versión.
Vamos a ver que n19  n7  0 (mod 5) , n19  n7  0 (mod 3) y n19  n7  0 (mod 2) , y por tanto
n19  n7  0 (mod 5  3  2) .

a) n19  n7  0 (mod 5) .
Por el PTF, n5  n (mod 5) , luego
  3
n19  n53 4  n5 n4  n3n4  n7 (mod 5)

b) n19  n7  0 (mod 3)
Por el PTF, n3  n (mod 3) , luego
  6
n19  n631  n3 n  n6n  n7 (mod 3)

c) n19  n7  0 (mod 2)
Si n es par, n19 y n 7 son pares y por tanto n19  n7 es par.

Si n es impar, n12 es impar, luego n12  1 es par y por tanto n19  n7  n7 n12  1 es par. 
Segunda versión. Sin usar aritmética modular ni PTF
n19  n7  n7 (n12  1)  n7 (n6  1)(n6  1)  n7 (n6  1)(n3  1)(n3  1)
Y utilitzando las identidades
 
n3  1  (n  1) n2  n  1
 
n3  1  (n  1) n2  n  1
 
n6  1  (n2  1) n4  n2  1
llegamos a
    
n19  n7  n7 (n2  1) n4  n2  1 (n  1) n2  n  1 (n  1) n2  n  1 
Ahora vemos que existen tres números consecutivos divisores de n19  n7 , que son
n  1 , n , n  2 . Al menos uno de ellos será divisible entre 2 y exactamente uno de ellos será
divisible entre 3.
Veamos que n19  n7 también es divisible entre 5:
Si ninguno de los números n  1 , n , n  1 es múltiple de 5 , entonces n  5k  2 o n  5k  3 , y
por tanto
n  5k  2  n2  1  25k 2  20k  5 es divisible entre 5
n  5k  3  n2  1  25k 2  30k  10 es divisible entre 5.
En todo caso, 5 | n2  1 | n19  n7

Fuente de la segunda versión: Solución oficial (SE, pág. 883).

11.5
Antes de nada, observamos que el conjunto de números que al dividirlos entre 3 da residuo 1 es
cerrado por la multiplicación, es decir:
a  3a'1
  ab  (3a'1)(3b'1)  9a' b'3a'3b'1  3(3a' b' a'b' )  1
b  3b'1
Su producto también dará resto 1 al dividirlo entre 3.

Vamos a resolver este problema por reducción al absurdo.


Sea K1 el conjunto de números primos que al dividirlos entre 3 dan resto 1, y K 2 el conjunto
de números primos que al dividirlos entre 3 dan resto 2.
Todo número primo pertenece a K1 o K 2 , puesto que no puede ser múltiplo de 3.
Supongamos que el conjunto K 2 es finito:
K2   p1 , p2 , ... , pn 

Y consideremos el número k  3 p1 p2 ... pn  2 .


No puede ser primo pues entonces pertenecería a K 2 lo cual es absurdo pues
k  p1,..., pn
Luego será producto de primos. Al menos uno de estos factores primos de k no será de la
forma 3k  1 , pues, por la observación anterior, si todos los factores de k son de la forma
3k  1 , su producto también sería de la forma 3k  1 , pero k da resto 2 al dividirlo entre 3, no
da resto 1.
Luego existirá un factor primo pi de k que pertenece al conjunto K 2 , lo cual es absurdo pues
cuando dividimos k  3 p1 p2 ... pn  2 entre pi da resto 2, no 0.
Con esto llegamos a contradicción, y por tanto el conjunto K 2 no puede ser finito.

Observación: En las soluciones oficiales (SE, pág. 1080) se presenta una variación ligeramente diferente.

11.6
Una raíz cuadrada de un entero es irracional o es un entero, luego se trata de determinar si
3n2  2n  2 es un cuadrado perfecto para algún n  1 .

Supongamos que existe un entero k  0 tal que


3n 2  2n  2  k 2  3n 2  2n  2  k 2  0 
 2  4  4  3  (2  k 2 )  2  12k 2  20  2  4(3k 2  5)  2  2 3k 2  5
n    
23 23 23 23
 1  3k 2  5

3

Pero entonces n es un múltiplo de 3, y por tanto 3n2  2n  2 es de la forma 3q  2 , y no


puede ser un cuadrado perfecto como ya vimos en el problema 3.16.

11.7
Estudiando los primeros primos vemos, en efecto, una sucesión de 7 números seguidos
cumpliendo la condición:
29  29 , 30  2  3  5 , 31  31 , 32  25 , 33  3  11 , 34  2  17 , 35  5  7

Vamos a demostrar que no puede haber más.


Cada 8 números naturales habrá uno que dividido entre 8 dé residuo 4:
n  8k  4  22 (2k  1)

La descomposición factorial de dicho número tiene seguro el factor 2 elevado al cuadrado, pues
el paréntesis es impar, y por lo tanto no puede ser divisible entre 2. Así pues, dicho número no
puede cumplir la condición del enunciado.
11.8
Está claro que x  0, y  0 es solución trivial de la ecuación.
Si x  0 la ecuación se reduce a py  0  y  0 , con lo que llegamos a la solución trivial.
Si y  0 por el mismo razomiento llegamos a la misma solución (la ecuación es simétrica).

p x  y   x y  p | x y  p | x o p | y
Supongamos que p | x  x  p x' , y por tanto

px  y   x y  p px' y   px' y  px' y  x' y  px'  x' y  y  ( x'1) y  p | ( x'1) y

Luego p | ( x'1) o p | y .

Si p | y , entonces y  py' y por tanto la ecuación queda


px'  ( x'1) y  px'  ( x'1) py'  x'  ( x'1) y'

Esta última ecuación solo tiene soluciones x'  0, y'  0 y x'  2, y'  2 , es decir, la solución
trivial y x  2 p , y  2 p .

En efecto: p2 p  2 p   4 p 2  2 p  2 p  x y .

Nos queda estudiar el caso p | ( x'1) , es decir x'1  pk , y la ecuación queda


pk  1 1
px'  ( x'1) y  p( pk  1)  pky  pk  1  ky  y   p
k k

La única posibilidad que y sea entero es k  1 , luego y  p  1 , x'  p  1 , x  p( p  1)

En efecto:
   
p p( p  1)  p  1  p p 2  p  p  1  p p 2  2 p  1  p( p  1)2  p( p  1)( p  1)  xy

Luego las soluciones son tres:


x  y  0 ; x  y  2 p ; x  p( p  1), y  p  1

Observación.
En las soluciones oficiales (SE, pág. 849) se presenta un razonamiento parecido:
px' y  x' y  0  x' y  px' y  x' ( y  p)  y  p  x' ( y  p)  y  p 
 x'1 | p
 x' ( y  p)  ( y  p)  ( x'1)( y  p)  
y  p | p
 x'1  p
 x'1   p

Y puesto que p es primo, tenemos que x'1 | p  
 x'1  1
 x'1  1

11.9
Ver DE , problema #10.18
11.10
Se demuestra por inducción sobre n.
Para n  1 , S1  5  51 .
Para n  2 , S2  75  52  3
Supongamos que es cierto hasta n  1 , es decir, existe un S n 1 con n  1 dígitos todos impares
divisible entre 5n 1 .
Para cualquier número impar 1  k  9 , el valor
Sn  k  10n 1  Sn 1

Consiste en añadir el dígito k a la izquierda del número anterior. Por lo tanto, si S n 1 tiene todo
sus dígitos impares, también los tendrá S n .

Vamos a demostrar que existe un número impar 1  k  9 para el cual, además, S n sea divisible
entre 5n .

Puesto que 5n 1 | Sn -1 , podemos escribir Sn 1  a  5n 1 , y por tanto



Sn  k  10n 1  Sn 1  k  10n 1  a  5n 1  5n 1 k  2n 1  a 
5n | Sn  5 | k  2n 1  a  k  2n 1  a  0 (mod 5)

Y en efecto, basta tomar k  3n1 a (mod 5)

En efecto,  3n 1 a  2n 1  a  6n 1 a  a  1n 1 a  a  a  a  0 (mod 5)

Como estamos trabajando en módulo 5, si el valor de k obtenido es par, podemos sumarle 5


para que sea impar.

Nota: Se puede demostrar que el valor de k es único, y por lo tanto existe un único S n para
cada n.

11.11
2k  1
Sea Fkn  1 
n
Vamos a demostrar la identidad del enunciado por inducción sobre k .

Para k  1 es fácil:
21  1 2 1 1 1
F1n  1  1  1   1  , la solución es m1  n .
n n n m1

Supongamos que es cierto hasta hasta k , y veamos que también se cumple para k  1 :
   2  1  
k
1 1
 1  k 1   1  1  k 1 
n/2
F
 2  n2  n / 2  2  n  2 
k

 2k  1  n / 2  2k 1  n  2  1   2k 1  2  n  2k 1  n  2  1 
   k 1     k 1  
 n/2  2  n  2   n  2  n  2 
2k 1  n  1 2k 1  1
   1  Fkn1
n n

 1 
Fkn1  Fkn / 2  1  k 1  , y por tanto, si n es par, ya está demostrado, pues es cierto para
 2  n2
 1 
Fkn / 2 y basta multicarlo por 1   con mk 1  2k 1  n  2 .
 m k 1 

Si n es impar,
 1  2k  1  1 
Fk( n 1) / 2  1    1  1   
 n   (n  1) / 2  n 
 2k 1  2  1   2k 1  2  n  1  n  1  2k 1  1  n 2k 1  1
 1  1         1  Fkn1
 n  1  n   n  1  n  n n

 1
Es decir, Fkn1  Fk( n 1) / 2  1  
 n
Y se demuestra por inducción, pues (n  1) / 2 es entero, tomando mk 1  n ..

Sea par o impar, queda demostrado el enunciado.

11.12
El valor y es solución si y solo si lo es  y , así que podemos suponer y  0 .
Si x  0 entonces la ecuación es y 2  1  20  220 1  1  1  2  4  y  2 , llegando a las
soluciones: 0,  2 .

Supongamos que x  0 .

 
1  2 x  22 x 1  y 2  2 x 1  2  2 x  2 x  2  22 x  2 x  22 x 1  y 2  1  ( y  1)( y  1) 
 
2 1  2  2  ( y  1)( y  1)
x x

De esta última igualdad deducimos que y tiene que ser impar, luego y  1 , y  1 son pares.
Pasando a módulo 4, tenemos
y  4q  0 descartado, pues es par.
y  4q  1  y  1 es múltiple de 4 , y  1 es múltiple de 2 .
y  4q  2 descartado, pues es par.
y  4q  3  y  1 es múltiple de 4, y  1 es múltiple de 2 .
En todo caso, ( y  1)( y  1) es múltiplo de 8, pero 1  2  2 x es impar, luego x  3 .
y  4q  1  y 2  1  16q 2  8q  1  1  16q 2  8q  8q2q  1 y la ecuación queda
 
2x 1  2  2x  y 2  1 
2 1  2  2   8q2q  1  2 q2q  1 
x x 3

2 1  2  2   q2q  1  q | 2  q  2
x 3 x x 3 x 3
k  y  4q  1  22 2 x  3 q  1  2 x 1 k  1

Además, k debe ser impar, pues en caso contrario


y  2x t  1  y 2  22 x t 2  2x 1t  1  y 2  1  22 x t 2  2x 1t  2x (2x t 2  2t )
y la ecuación queda
 
2x 1  2  2x  y 2  1  2x (2x t 2  2t )  1  2  2 x  2 x t 2  2t , impar igual a par, absurdo.

Así pues, y  2 x 1 k  1 o bien y  2 x 1 k  1 para cierto k  1 impar.

Primer caso: y  2 x 1 k  1

Entonces y 2  1  22 x  2 k 2  2x k  1  1  2 x 2 x  2 k 2  k y la ecuación queda 
2 1 2
x
 x 1
  2 2 x x2

k  k 1 2
2 x 1
2 x2
k k 
2

1  k  2 x  2 k 2  2 x 1  2 x  2 k 2
 23 
1  k  2x2 k 2  8   (*)

 
Luego k  1  2x  2 k 2  8  0  k 2  8  0  k 2  8  k  1 , 2

Hemos dicho anteriormente que k era impar, luego solo nos queda la posibilidad k  1 .

Pero sustituyendo k  1 en (*) tenemos 1  1  2 x  2 12  8  0  2 x  2  7 absurdo. No hay  


ninguna solución posible.

Segundo caso: y  2 x 1 k  1
De la misma manera, Entonces y 2  1  22 x  2 k 2  2 x k  1  1  2 x 2 x  2 k 2  k y la ecuación  
queda
   
2 x 1  2 x 1  2 x 2 x  2 k 2  k  1  2 x 1  2 x  2 k 2  k 
1  k  2 x  2 k 2  2 x 1  2 x  2 k 2
 23  
1  k  2x2 k 2  8   (**)

Luego

1  k  2x 2 k 2  8  2 k 2  8    
2k 2  k  17  0  k  3  k  1 , 3

Sustituyendo en (**) vemos que k  1 no es aceptable:



2  1  1  2 x  2 12  8  2 x  2  7 
Así que la única opción aceptable es k  3 , que sustituyendo en (**) nos determina el valor x :
 
1  3  2x  2 32  8  4  2x  2 1  x  4
Y finalmente y  2x 1 k  1  24 1  3  1  24  1  23

Las soluciones son, por tanto, x  4 , y  23 .

Comprobemos que, efectivamente, los valores obtenidos satisfacen la ecuación:


1  24  224 1  1  16  512  529  232 .

El problema tiene cuatro soluciones: x  0, y  2 y x  4, y  23 .

Fuente de la solución: Soluciones oficiales (SE, página 880)

11.13
Pasamos a módulo n:
n | ai ai 1  1  ai ai 1  1  0 (mod n)  ai ai 1  ai  0 (mod n) 
ai ai 1  ai (mod n)

Tomando el primer elemento a1 , vemos que:


a1a2  a1 (mod n) 
  a1a2 a3  a1a2  a1 (mod n)
a2 a3  a2 (mod n)

Y después
a3a4  a3 (mod n) 
  a1a2 a3a4  a1a2 a3  a1 (mod n)
a1a2 a3a4  a1a2 a3  a1 (mod n)

Hasta que, finalmente,


a1...ak 1  a1 (mod n)

Pero si, además, n | ak a1  1  ak a1  a1 (mod n) , en cierta manera lo que hacemos es


completar el círculo, es decir:
a1  a1ak  a1...ak 1ak (mod n)

Pero esto lo podríamos haber hecho con todos y cada uno de los elementos a j , es decir:
a1  a2  ...  ak  a1...ak 1ak (mod n)

Lo cual es absurdo, pues 1  a j  n y son todos diferentes.

Fuente de la solución: "Mathematical Excalibur May-Sep. 2009"

11.14
Buscamos un número entero n con 104  n  105 y que cumpla n2  m  105  n
para un cierto entero m .

n2  m  105  m  n2  n  105 m  n2  n  n(n  1) es divisible entre 105


Puesto que n y n  1 son coprimos, 2555  105 | n(n  1) si y solo si n o n  1 es divisible entre
25 y el otro es divisible entre 25 .

O, equivalentemente, queremos un número k divisible entre 55  3125 y que cumpla


k  1 (mod 32)

3125 | k  k  3125 q 
  k  1 (mod 32)  3125 q  1 (mod 32)  21q  1 (mod 32)
3125  21 (mod 32) 

Puesto que 21 y 32 son coprimos, la ecuación 21q  1 (mod 32)

Tiene por solución q  3 . En efecto 21  3  63  2  32  1

Por otro lado, la condición 104  k  105 obliga a que 4  q  31 , y por tanto tomaremos el
valor q  32  3  29
21(32  3)  21(3)  ()1  1 (mod 32)

Así pues, k  3125  29  90625 .

En efecto, 906252  8212890625

Fuente de la solución: https://math.stackexchange.com/questions/1692048/a-five-digit-number-whose-square-has-its-last-five-digits-equal-to-


the-number

Observación: La solución oficial se puede considerar como un buen ejemplo de lo que no debe
ser la solución a un problema.

11.15
Primera versión.
n 2 n  1
2
1  2  3  ...  n 
3 3 3 3

n 2 n  1
2
Queremos resolver la congruencia  17 mod (n  5)
4

n 2 n  1 n 2 n  1
2 2
 17 mod (n  5)   k (n  5)  17
4 4
 n 2 n  1  4k (n  5)  4 17  4k (n  5)  68 
2

n 2 n  1  68 mod (n  5)
2

Pero observamos que


n  5 mod (n  5)  n 2   5 mod (n  5)
2

n  1  4 mod (n  5)  n  1   4 mod (n  5)
2 2
Y por tanto
n 2 n  1  (5) 2 (4) 2  25 16  400 mod (n  5)
2

Luego
400  68 mod (n  5)  400  68  0 mod (n  5)  332  0 mod (n  5)
 n  5 | 332  22  83

Con las condiciones del enunciado estamos suponiendo implícitamente que n  5  17 , luego
las únicas posibilidades son n  5  83,166 , 332  n  78 ,161, 327 .
Comprobamos estas soluciones, y vemos que se cumple para 78 y para 161, pero no para 327:
n 2 n  1
2
n  327   100 mod (n  5)
4
Luego las soluciones son n  78 ,161 .

Observación. Las comprobaciones exigen cálculo manual con números grandes. Por ejemplo:
n 2  106929

n  327  

(n  1)  107584
2

Segunda versión. Mediante aritmética modular.


n 2 n  1
2
 17 mod (n  5) 
4
n 2 n  1  68 mod (n  5) 
2

n 2 n  1  68  0 mod (n  5) 
2

n  5 | n 2 n  1  68  n 4  2n 3  n 2  68
2

Realizando la división sintética tenemos que


 
n 4  2n3  n 2  68  n3  3n 2  16n  80 n  5  332 
n 4  2n3  n 2  68 332
 n3  3n 2  16n  80 
n5 n5

Y por tanto
n  5 | n 2 n  1  68 
332
Z
2

n5

Y se sigue igual que en la primera versión.

Tercera versión.
n  m  5
Con un cambio de variable: m  n  5  
n  1  m  4
Y por tanto
n 2 n  1 (m  5) 2 (m  4) 2
2
 17 mod (n  5)   17 mod m 
4 4
(5) 2 (4) 2 400
 17 mod m   17 mod m  400  4 17 mod m 
4 4
400  68 mod m  400  68  0 mod m  332  0 mod m  m | 332  n  5 | 332

Y se sigue igual que en la primera versión.

Fuente de estas soluciones: https://artofproblemsolving.com/wiki/index.php?title=2020_AIME_II_Problems/Problem_10&oldid=125812

11.16
Primera versión. Mediante aritmética modular.
Buscamos conjuntos ordenados (a, b, c, d ) , con a  0 , tales que
7 | 0abcd  7 | 10000  0  1000  a  100  b  10  c  d (1)
7 | a0bcd  7 | 10000  a  1000  0  100  b  10  c  d (2)
7 | ab0cd  7 | 10000  a  1000  b  100  0  10  c  d (3)
7 | abc0d  7 | 10000  a  1000  b  100  c  10  0  d (4)
7 | abcd 0  7 | 10000  a  1000  b  100  c  10  d  0 (5)

Puesto que: 10000  4 (mod 7) , 1000  6 (mod 7) , 100  2 (mod 7) y 10  3 (mod 7) ,


las condiciones del enunciado se pueden escribir como:
10000  0  1000  a  100  b  10  c  d  0 (mod 7)  6a  2b  3c  d  0 (mod 7)
10000  a  1000  0  100  b  10  c  d  0 (mod 7)  4a  2b  3c  d  0 (mod 7)
10000  a  1000  b  100  0  10  c  d  0 (mod 7)  4a  6b  3c  d  0 (mod 7)
10000  a  1000  b  100  c  10  0  d  0 (mod 7)  4a  6b  2c  d  0 (mod 7)
10000  a  1000  b  100  c  10  d  0  0 (mod 7)  4a  6b  2c  3d  0 (mod 7)

Restando la primera de la segunda llegamos a


2a  0 (mod 7)  a  0, 7 , y solo puede ser a  7
Restando la tercera de la segunda llegamos a
2b  0 (mod 7)  b  0, 7
Restando la tercera de la cuarta llegamos a
c  0 (mod 7)  c  0, 7
Restando la quinta de la cuarta llegamos a
2d  0 (mod 7)  d  0, 7

Así pues, las soluciones posibles son los todos los números de cuatro cifras que se forman con
los dígitos 0 y 7, es decir, los siete números: 7000, 7007, 7070, 7077, 7700, 7707 y 7777.

Segunda versión. Sin aritmética modular.


El tratamiento sería similar. Por ejemplo:
x  0abcd  10000  0  1000  a  100  b  10  c  d
y  a0bcd  10000  a  1000  0  100  b  10  c  d
7 | x
  7 | y  x  9000a
7 | y
Y puesto que 7 no es divisor de 9000, se llega a 7 | a , es decir, a  0, 7 . Y como no puede ser
cero puesto que estamos suponiendo un número de cuatro cifras, solo queda a  7 .
Y con un argumento similar se deducen los otros tres dígitos.

11.17
a)
2n  1 y 2n  1 son coprimos.
2n  1  k  a 
  2n  1  2n  1  2  kb  ka  k (b  a)
2n  1  k  b 
Y esto es imposible para cualquier k  1 , pues entonces tendríamos k  2 y los números 2n  1
y 2n  1 serían pares, lo cual es absurdo.

b)
Está claro que podemos reducir nuestra búsqueda a divisores comunes primos.
Sea p un divisor común de n 2  1 y 3n  1
Puesto que n2  1  (n  1)(n  1) , p | n2  1 si y solo si p | n  1 o p | n  1 . (ver 4.5)
Supongamos que p | n  1 .
p | n 1 
  p | 3n  1  (n  1)  2n  p | n
p | 3n  1
Sabemos que dos números consecutivos son coprimos, luego es imposible que p sea divisor de
n y de n  1 .
Supongamos que p | n  1 .
p | n 1 
  p | 3n  1  (n  1)  2n  2  2(n  1)  p | n  1
p | 3n  1
Luego p | n  1 y p | n  1 , lo cual es absurdo por el apartado a).

Fuente de esta solución: F.J.G.C. en Facebook.

11.18
En primer lugar pasamos la igualdad a módulo 3:
3x  4 y  5z  3x  4 y  5z (mod 3)  0  1y  (1) z (mod 3)  1  (1) z (mod 3)

de aquí deducimos que z debe ser par: z  2k para cierto k  IN .


3x  4 y  52k  3x  52k  4 y  5k   2   5
2 y 2 k

 2 y 5k  2 y 
Por lo tanto los dos factores de la parte derecha deben ser ambos potencias de 3.
Pero estos dos factores no pueden ser ambos múltiples de 3, pues aplicando el Algoritmo de
Euclides:
mcd  a , b   mcd  a , a  b 

En nuestro caso, sumando ambos factores:


  
mcd 5k  2 y , 5k  2 y  mcd 5k  2 y , 2  5k 
Y claramente 2  5k no es múltiple de 3.

La única opción válida es que sean factores coprimos, y por tanto


5k  2 y  3x
 k
5  2 y  1

Restamos las dos ecuaciones para llegar a


2  2 y  3x  1  2 y 1  3x  1 (*)

Ahora observamos que si y  3 entonces 2 y 1 es un múltiplo de 16 y por tanto, pasando a


módulo 16 llegamos a
0  3x  1 (mod 16)  1  3x (mod 16)

Estudiemos las potencias de 3 módulo 16:


x  0  30  1 (mod 16)
x  1  31  3 (mod 16)
x  2  32  9 (mod 16)
x  3  33  27 (mod 16)
x  4  34  81  5  16  1  1 (mod 16)
x  5  35  243  3 (mod 16)

Vemos que generan un ciclo, es decir, que x ha de ser un múltiple de 4: x  4s para cierto
s  IN . Luego la ecuación se convierte en
2 y 1  3x  1  34 s  1  81s  1  81s  2 y 1  1

Pero ahora, pasando a módulo 5, tenemos


2 y 1  1  1s  1 (mod 5)  2 y 1  0 (mod 5)
Lo cual es imposible, pues ninguna potencia de 2 es múltiple de 5.

Con todo esto hemos demostrado que la hipótesis inicial y  3 es inaceptable, y por tanto
y  3 , y ahora podemos testear caso por caso en la ecuación (*):
y  1  211  4  3x  1  4  1  3x  5  3x no tiene solución.
y  2  22 1  8  3x  1  8  1  3x  9  3x  x  2
Y resolvemos el sistema:
5k  2 y  3x
 5k  4  9
 
5  9  4  5
k

 k   k   k  k  1  z  2k  2

5  2  1
y

5  4  1 
5  1  4  5

Así pues, la única solución aceptable de este problema es la terna pitagórica conocida
32  42  52 .

12.10
  5
a) Aplicando PTF, 36  1 (mod 7)  331  356 1  36 3  15  3  3 (mod 7)
b) 4 , c) 9

12.11
Aplicando el PTF, 212  1 (mod13) , luego
 
21000  21283 4  212
83
 24  183  24  24 (mod13)  16 (mod13)  3 (mod13)
El residuo es 3.

12.12
Por el PTF, 1116  1 (mod17) .
 
104  16  6  8 , luego 11104  11166 8  1116 118  1 118  118 (mod17)
6 6

  4
112  121  2 (mod17)  118  112  24 (mod17)  16 (mod17)  1 (mod17)

Finalmente: 11104  1 (mod17)  11104  1  0 (mod17)  17 | 11104  1

12.13
El número a no puede ser múltiplo de 5, pues en ese caso mcd (a,35)  1. Luego podemos
aplicar el PTF para garantizar que a 4  1 (mod 5) , y por tanto:
  3
a 4  1 (mod 5)  a12  a 4  13  1 (mod 5)
De la misma manera, a no puede ser múltiplo de 7, y de nuevo aplicamos el PTF para
garantizar que a 6  1 (mod 7) , y por tanto:
  2
a6  1 (mod 7)  a12  a6  12  1 (mod 7)
Luego:
a12  1 (mod 5) 

  a  1 mod5,7  a  1 mod 35
12 12

a  1 (mod 7)
12

pues 5,7  mcm(5,7)  5  7  35 ya que mcd (5,7)  1

12.14
Sabemos que 4k  4 (mod 6) para todo k (se demuestra fácilmente por inducción sobre k)
Observamos que la secuencia an parece dar siempre residuo 4 módulo 7:
a1  4  4 (mod 7)
a2  44  256  7  36  4  4 (mod 7)

Veamos que, en general, si k  4 (mod 6)  4k  4 (mod 7) . En efecto:


Por el PTF, 46  1 (mod 7) luego
  j
k  4 (mod 6)  k  6 j  4  4k  46 j  4  46 44  44  4 (mod 7)
Luego an  4 (mod 7) para todo n, en particular, para n  100 .

12.15
Queremos ver (a,42)  1  a6  1  0 mod(3  7  8)  a6  1 mod(3  7  8)

Aplicamos tres veces el PTF:


  3
(a,42)  1  3 | a  a 2  1mod(3)  a 6  a 2  13  1 mod(3)
(a,42)  1  7 | a  a 6  1mod(7)
(a,42)  1  2 | a  a  1mod( 2)  a 6  16  1mod( 2)

Y ahora:
a 6  1 mod(3) 

a 6  1 mod(7)  a 6  1 mod(3,7,2)  a 6  1 mod(3  7  8)
a 6  1 mod( 2)

12.16
Vemos que 7 | 2,3,4,5,6 y por tanto podemos aplicar el PTF:
  2
26  1 (mod 7)  220  236  2  26 22 (mod 7)  1222 (mod 7)  4 (mod 7)
3  1 (mod 7)  3  3  3  (mod 7)  1  1 (mod 7)
6 30 6 5 6 5 5

4  1 (mod 7)  4  4
6 40
 4   4 (mod 7)  1  4 (mod 7)  4 (mod 7)  4 (mod 7)
6 6  4 6 6 4 6 4 4

En donde hemos aplicado que 4  16  2 (mod 7)  4  4   2  4 (mod 7)


2 4 2 2 2

5  1 (mod 7)  5  5
6 50
 5   5 (mod 7)  1  5  5  25  4 (mod 7)
86  2 6 8 2 8 2 2

6  1 (mod 7)  6  5  5  (mod 7)  1 (mod 7)


6 60 106 6 10

Finalmente, 220  330  440  550  660  4  1  4  4  1  14  0 (mod 7)

12.17
Primera versión.
Aplicando el PTF a nuestro caso:
1335  133  3 (mod 5) 

1105  110  0 (mod 5)
  133  110  84  27  3  0  4  2  4 (mod 5)
5 5 5 5

84  84  4 (mod 5) 
5

275  27  2 (mod 5) 
n5  n (mod 5)  n  4 (mod 5)

Por otro lado:


133  1 (mod 3)  1335  15  1 (mod 3) 

110  1 (mod 3)  1105  (1)5  1 (mod 3)
  133  110  84  27  1  1  0 (mod 3)
5 5 5 5

84  0 (mod 3)  84  0 (mod 3)
5

27  0 (mod 3)  27  0 (mod 3)
5 

Por el PTF, n3  n (mod 3)  n5  n2n3  n2n  n3  n (mod 3) , luego


n  0 (mod 3)

Está claro que n5  1335  1105  845  275  1335  n  133


El primer número n  133 que cumple n  4 (mod 5) y n  0 (mod 3) es 144, y el siguiente es
176.

Enseguida (????) vemos que 176 es ya demasiado grande, por lo tanto la solución debe ser
n  144 .

Segunda versión.
133  3 (mod10)  1335  35  243  3 (mod10)

110  0 (mod10)  1105  0 (mod10) 
  n  2  0  4  7  4 (mod10)
5

84  4 (mod10)  84  4  4 (mod10)
5 5

27  7 (mod10)  27  7  7 (mod10)
5 5 

Pero por otro lado,


n2  n (mod 2)  n5  n2n2n  n3  n2n  n2  n(mod 2) , y por tanto:
n5  n (mod 2)

  n  n (mod10)
5

n  n (mod 5) 
5

Así pues, n5  n  4 (mod10) , y por lo tanto el número n buscado acaba en 4.

Observamos que 133,110 , 84 son números muy próximos a múltiplos de 27:


133  5  27  1335  55  275 

110  4  27  1105  45  275   n5  1335  1105  845  275  275 55  45  35  1
84  3  27  845  35  275 
5
5
 5 5 5
 5 n5  n 
n  27 5  4  3  1  27  4393  4393  5   
5

27  27 

Como antes, sabemos que n  133 , y sabemos que acaba en 4, luego los candidatos son 134,
144, 154, 164…

Está claro que n  134 nos vamos a quedar cortos.

Probando (y utilizado mucho, pero mucho cálculo !!!!) con n  144 :


5
144 16  16  1048576
n  144       4315 , que se aproxima bastante a 4393.
27 3 3 243

Puesto que estamos trabajando con potencias quintas, que crecen de forma muy rápida, está
claro que el siguiente candidato n  154 ya no será muy próximo al valor buscado, por lo que
podemos asegurar que n  144 es la solución del problema.

Nota: El enunciado lleva implícito que este número existe. Ninguna de las dos soluciones
demuestra que 1335  1105  845  275  1445 , son buenos argumentos que justifican que
n  154 es un buen candidato para este resultado.

Fuente de esta solución: https://artofproblemsolving.com/wiki/index.php/1989_AIME_Problems/Problem_9


12.18
p | 29 p  1  29 p  1  0 (mod p)  29 p  1 (mod p) 
29    1
p 2 2
 1 (mod p)  292   p
 1 (mod p)

Pero, aplicando el PTF, sabemos que a p  a (mod p) para todo a , luego


  p
1  292  292 (mod p)  1  292 (mod p)  292  1  0 (mod p) 
p | 292  1  840  23  3  5  7  p  2 , 3 , 5 , 7
De los cuatro candidatos posibles, los tres primeros son satisfactorios:
p  2  292  29  1  1 (mod 2)
p  3  293  29  2  1 (mod 3)
p  5  295  29  4  1 (mod 5)
(en donde seguimos aplicando PTF : a p  a (mod p) )
Pero el cuarto primo no es satisfactorio: p  7  297  29  1  1 (mod 7)
Luego las soluciones son tres: p  2 , 3 , 5 .

12.19
Vamos a demostrar que solo el 1 es coprimo con toda la sucesión
an  2n  3n  6n  1 , n  1
Es decir, para todo k  1 , existe un m tal que k , am   1 .
Para ello es suficiente demostrarlo para los números primos.
En efecto, supongamos que se cumple para todo primo p .
Supongamos que no es cierto para cierto k  1 compuesto. Sea p uno de los factores primos
de k .
k , an   1 para todo n  1 .
p|k 
  p, an   1 para todo n  1 , contradiciendo la hipótesis.
k , an   1

Para p  2,3 , está claro que p | a2  48 .


Si p  3 , por el Pequeño teorema de Fermat,
2 p 1  1 (mod p)  3  2 p 1  3 (mod p)
3 p 1  1 (mod p)  2  3 p 1  2 (mod p)
6 p 1  1 (mod p)

y por tanto
6a p  2  6  2 p  2  6  3 p  2  6  6 p  2  6 
 3  2 p 1  2  3 p 1  6 p 1  6  3  2  1  6  0 (mod p)  p | 6a p  2

Y por el Corolario al Lema de Euclides, llegamos a p | a p  2   p, a p  2   p  1 .

Fuente de la solución: Soluciones oficiales (SI , pág. 821)


13.8
i | 2 j  1  2 j  1  0 (mod i)  2 j  1 (mod i)
Si i  1 , 2  0 (mod1) , 1  0 (mod1) , y por lo tanto, trivialmente 2 j  1 (mod i) para j  1 .
Si i  2 , 2  0 (mod1) , 1  0 (mod1) , y por lo tanto, trivialmente 2 j  0  0 (mod i) para todo
j , luego no se cumple la condición del enunciado.
Supongamos que i  3 .
Aplicando el Teorema de Euler, sabemos que si (2, i)  1 , es decir, si i es impar, entonces
2 (i )  1 (mod i) , y por tanto basta tomar j   (i) , que sabemos cumplirá 1   (i)  i .

Si (2, i)  1, es decir, si i es par, no existirá ningún j tal que 2 j  1 (mod i) , pues entonces:
2 j  1 (mod i)  2  2 j 1  1 (mod i)
Y por tanto x  2 j 1 sería solución para la congruencia 2 x  1 (mod i) , pero ya vimos en en el
Tema 9 que no tiene solución.
Tomando j  1 , es imposible, pues 2  1 (mod i) .
Luego se cumple para todos los 1  i  1000 impares, es decir, para 500 números.

13.9
Estudiar los ocho últimos dígitos de la expansión binaria de 271986 es equivalente a determinar
(en binario) 271986 (mod 256)
 1
 (256)   (256)  2561    128 ,
 2
Puesto que 256 | 27 , podemos aplicar el Teorema de Euler para garantizar que
27128  1 (mod 256) .
 
15
Por otro lado, 271986  2715128 66  27128 2766  1152766  2766 (mod 256)

Calcularemos 2766 (mod 256) con el “método de las potencias de dos”:


272  729  729  217  39 (mod 256)  2764   39 (mod 256)
32

(39)2  1521  241  15 (mod 256)   39   39


32
 2 16
   15 (mod 256)
16

 
(15)2  225  (31) (mod 256)   15  (15)2  (31)8 (mod 256)
16 8

 
4
(31)2  961  193  (63) (mod 256)  (31)8  (31)2  (63)4 (mod 256)
 (63) 
2
(63)2  3969  129 (mod 256)  (63)4 2
 1292 (mod 256)
1292  1(mod 256)
Por lo anterior: 2766  2764272  1  217  217 (mod 256)

Y la expresión binaria de 217 es "11011001"

13.10
Queremos determinar a2008 (mod1000) , donde n se ha definido recursivamente:
a1  1 , a2  2a1 , a3  3a2 , ... , a2008  2008a2007
Por un lado, 2008  251  8  2008  0 (mod 8)  n  2008a2007  0 (mod 8) .

Por otro lado, 2008  16  125  8  2008  8 (mod 125)

 1 4
 (125)   (53 )  53 1    53 100 , luego, aplicando el Teorema de Euler, para cualquier
 5 5
número a , con (a,125)  1 , se cumple a100  1 (mod 125) .

Observamos que
2007  7 (mod 100)  20074k  74k  2401k  1k  1 (mod 100)

a2006  2006a2005  (2  1003)a2005  2a2005  1003a2005 es múltiplo de 4, pues seguro que a2005  2 ,
Y por tanto a2007  2007a2006  1 (mod 100)  a2007  100k  1 para cierto k, luego
a2008  2008a2007  2008100k 1  2008100k  2008  2008  8 (mod 125)

Finalmente aplicamos el Teorema Chino del Residuo:


n  0 (mod 8)

n  8 (mod 125)
N  8  125 

N1  125  125 y1  1 (mod 8)   n  125  y1  0  8  47  8  3008  8 (mod1000)
N 2  8  8 y2  1 (mod 125)  y2  47

Así pues, la solución al problema es "008".

Fuente de la solución: Olympiad Number Theory Through Challenging Problems (Justin Stevens, 3E) pág.49

13.11
Queremos calcular f (17)  f (18)  f (19)  f (20) (mod 100) , donde f (x) se ha definido
recursivamente: f1 ( x)  x , f 2 ( x)  x f1 ( x ) ,..., f ( x)  f 4 ( x)  x f3 ( x ) .
 1
 (25)   (52 )  251    20  a 20  1 (mod 25) si (a,25)  1
 5
Los calcularemos por separado.

Primera parte: f (20) (mod 100)


202  400  0 (mod100)  20k  0 (mod100) para todo k par, y puesto que
f 3 (20)  20 f2 ( 20) es par, está claro que f (20)  20 f3 ( 20)  0 (mod 100) .

Segunda parte: f (19) (mod 100)


f (19)  19 f3 (19)  (1) f3 (19) (mod 4)

  f (19)  1 (mod 4)
f 3 (19)  19 f 2 (19)
es impar 

f 2 (19)  19 f1 (19)
es claramente impar, luego
f3 (19) (mod  (25))  f3 (19) (mod 20)  19 f2 (19) (mod 20)  1 (mod 20) , y por tanto:
f (19)  19 f3 (19)  19 f3 (19) (mod ( 25))  191  4 (mod 25) , pues 19  4  76  3  25  1
f (19)  1 (mod 4) 
Resolvemos el sistema  mediante el Teorema Chino del Residuo:
f (19)  4 (mod 25)
N  4  25  100 

N1  25  25 y1  1 (mod 4)  y1  1   f (19)  25 1  (1)  4 19  4  279  79 (mod 100)
N 2  4  4 y2  1 (mod 25)  y2  19

Tercera parte: f (18) (mod 100)


f 3 (18)  18 f2 (19)  2  9 2  2 f2 (19)  9 f2 (19)  2k
f (19)

Y por tanto f (18)  18 f 3 (19)  2 f 3 (18)  22k  4k  0 (mod 4)


184  1 (mod 25) (!!!!)
f 3 (18)  18 f2 (18)  (2  9) f2 (18)  2 f2 (18)  9 f2 (18) es múltiplo de 4
f 4 (18)  18 f3 (18)  1 f3 (18)  1 (mod 25)
f (18)  0 (mod 4) 
Resolvemos el sistema  mediante el Teorema Chino del Residuo:
f (18)  1 (mod 25)
N  4  25  100 

N1  25  25 y1  1 (mod 4)  y1  1   f (18)  25 1  0  4 19 1  279  76 (mod 100)
N 2  4  4 y2  1 (mod 25)  y2  19

Cuarta parte: f (17) (mod 100)


17  1 (mod 4)  f (17)  17 f3 (17)  1 (mod 4)
f (17)  17 f3 (17)  17 f3 (17) mod ( 25) (mod 25)  (**)
f3 (17) mod  (25)  f 3 (17) mod 20  17 f2 (17) (mod 20)  17 f2 (17) mod ( 20) (mod 20)  (*)
 (20)  8 , luego: f 2 (17) mod  (20)  f 2 (17)  1717  117  1 mod 8
(*)  171  17 (mod 20)
(**)  1717 mod 25

Vamos a calcular esta última potencia por el "método de las potencias de dos":
17 2  289  14 (mod 25)
17 4  14 14  196  4 (mod 25)
178  (4)  (4)  16 (mod 25)
  2
1717  17 281  178 17  16 2 17  256 17  6 17  102  2 (mod 25)
Así pues, f (17)  2 (mod 25) .
Como antes, aplicamos el Teorema Chino del Residuo para determinar f (17) (mod 100)
f (17)  1 (mod 4) 

f (17)  2 (mod 25)
N  4  25  100 

N1  25  25 y1  1 (mod 4)  y1  1   f (17)  25 1 1  4 19  2  177  77 (mod 100)
N 2  4  4 y2  1 (mod 25)  y2  19

Y finalmente,
f (17)  f (18)  f (19)  f (20) (mod 100)  77  76  79  0  232 (mod 100)  32 (mod 100)

Fuente de la solución: Olympiad Number Theory Through Challenging Problems (Justin Stevens, 3E) pág.50

13.12
Queremos determinar 72014 (mod 1000) .

Primera versión.
Aplicamos el Teorema de Euler:
 (1000)  400 , y puesto que (7,1000)  1 , aplicando el Teorema de Euler:
1  7 (1000)  7400 mod (1000)

  5
Luego 72014  7400514  74005  714  7400  714  1  714  714 mod (1000)

7 2  49  73  343  7 4  2401  401  75  2807  807 


 76  5649  649  77  4543  543

  2
714  77  5432  * * *849 (no hace falta hacer toda la multiplicación)
72014  714 mod (1000)  849 mod (1000)

Fuente de esta versión: Solución oficial (Ver SE, página 333)

Nota: En la Solución oficial (SE, página 333) se presenta otra solución alternativa.

Segunda versión.
Como es habitual en este tipo de problemas (ver Problemas #11.5 y #13.3), calcularemos por
separado 72014 (mod 8) y 72014 (mod 125) para después determinar 72014 (mod 1000) mediante
el Teorema Chino del Residuo.

a) 72014 (mod 8)
72  49  1 (mod 8)  72014  72   1007
 11007  1 (mod 8)

b) 72014 (mod 125)


 1 4
 (125)   (53 )  53 1    53 100 , luego, aplicando el Teorema de Euler, para cualquier
 5 5
número a , con (a,125)  1 , se cumple a100  1 (mod 125) .

En nuestro caso se cumple (2014,125)  1, luego 7100  1 (mod 125)


 
Y por tanto 71000  7100
10
 110  1 (mod 125)
 
2
72000  71000  12  1 (mod 125)
7 2  49 (mod 125)
73  343  93 (mod 125)
7 4  651  151  26 (mod 125)
75  182  57 (mod 125)
7 6  219  24 (mod 125)
7 7  168  43 (mod 125)

Y por tanto 714  77  


2
 432  1849  99 (mod 125)

Y por último: 72014  72000  714  1  99  99 (mod 125)

Finalmente, aplicamos el Teorema chino del Residuo:

c)
7 2014  1 (mod 8)
7 2014  99 (mod 125)
N  8  125  1000
N1  125  125 y1  1(mod 8)  y1  5
N2  8  8 y2  1(mod125)  y2  47 (*)
x  125  5  1  8  47  99  37849  849 (mod1000)

Nota: Esta segunda versión no es operativa en un contexto de una competición en la que no se


pueden usar calculadoras, pues, por ejemplo, el paso (*) anterior exige demasiado cálculo.

16.1
Primera versión.
Sea n  a  b .
a  b | a b  b a  a b  b a  0 (mod n)  a n  a  (n  a)a  0 (mod n) 
a n  a  (a)a (mod n)

Supongamos que a es impar. Entonces la condición anterior es equivalente a:


aa
a n. a  a a (mod n)  1  n  a (mod n)  1  a 2 a  n (mod n)
a

Aplicando el Teorema de Euler, puesto que a, b  1  a, n  1 , sabemos que


a ( n)  1 (mod n)

 (n)  n
Luego basta con resolver  (n)  2a  n  a  .
2

Veamos que para n  2 p con p primo y p  1 (mod 4) se cumple esta condición.


 1  1
n  2 p   (n)  2 p1  1    p  1
 2  p
Luego:
 ( n)  n p 1 2 p 3p 1
a  
2 2 2

Observamos que la condición p  1 (mod 4) garantiza que a es par, puesto que


p  1 (mod 4)  3 p  1  2 (mod 4)

3p 1 4 p  3p 1 p 1
Y por tanto, deducimos que a  b  n  2 p  b  2 p   
2 2 2

Queda solamente garantizar que a y b así definidos son coprimos. En efecto:


 3 p  1 p  1  3 p  1, p  1  p  3 , 4 2
 ,    1
 2 2  2 2 2
tal y como queríamos ver.

Segunda versión.
Veamos que todo par de la forma a  2n  1 , b  2n  1, con n  2 satisface la condición del
enunciado.
En primer lugar vemos que (a, b)  1 aplicando el Algoritmo de Euclides:
(a, b)  ( 2n  1 , 2n  1 )  ( 2n  1 , 2n  1  2 )  ( 2n  1 , 2 )  1
Observamos que
a 2  2n  1  4n2  4n  1  1 (mod 4n)
2

b2  2n  1  4n2  4n  1  1 (mod 4n)


2

a b  b a  a 2 n 1  b 2 n 1  a  a 2 n  b 2 n  2 1  a  a 2 n  b  b 2( n 1) 
  n
 a  a 2  b  b2   n 1
 a  1n  b  1n 1  a  b  4n  0 (mod 4n)  0 (mod a  b)

Tal y como queríamos ver.

Fuente de estas soluciones: www.artofproblemsolving.com

17.2
Calculando a mano los primeros términos de esta suma observamos su pauta:
20 1 21 2
0 0 0 1  0  0
3 3 3 3
2 3
2 4 3 1 1 2 8 6 2 2
2    1 1 3     2 2
3 3 3 3 3 3 3 3 3 3
4 5
2 16 15 1 1 2 32 30 2 2
4    5 5 5     10   10
3 3 3 3 3 3 3 3 3 3
6 7
2 64 63 1 1 2 128 126 2 2
6     21   21 7     42   42
3 3 3 3 3 3 3 3 3 3
8 9
2 256 255 1 1 2 512 510 2 2
8     85   85 9     170   170
3 3 3 3 3 3 3 3 3 3

Si n  2 es par, 2n  1 es múltiple de 3.
En efecto: 2  1 (mod 3)  2n  (1)n  1 (mod 3) si n es par.
Por lo tanto:
 2n   2n  1 1  2n  1  1  2n  1
 3    3  3  3  3  3
     
 2n 1   2n 
Y también vemos que si n  2 es par,    2  3  , luego
 3   
2 n 1
2 n 1
 2 1 1 
n
2 1 2
n
2 
n 1
 2 n  1
2  2    2     2 3 
3 3  3 3 3 3  3   
es decir, van por parejas: un valor y su doble.

Añadimos el sumando correspondiente a 1001 para tener 501 parejas completas:


1001
 2n  1001 2n  1001 2n  1  2n  1   1001  2n  1  1001 n
  3     3     3  2 3     3 3    2  1 
n0   n2   n2     n2   n2
n par n par n par

 1001 
   500   500 
   2n   500    22 n   500    4n   500  (*)
 nn par
2   n 1   n 1 
 
k
1  r k 1
Aplicamos la fórmula de la serie geométrica:  r n 
n 0 1 r
500
 500 n  1  4501 1  4501 4501  1

n 1
4 n
   4
 n 0 
  1 
1 4
 1 
3
 1 
3
1

Finalmente:
4501  1 4501  1
(*)   1  500   501
3 3

A este resultado le debemos restar el último valor que hemos contado de más:
 21001  2  21000 
 
   
 2 21000  1 2   2 21000  1 2  2 21000  1
   
 
 3   3  2 
     3 3  3 3 3

Por lo tanto, el resultado es


4501  1
 501 
  
2 21000  1 4501  1  2 21000  1


 501 
3 3 3
4501  1  21001  2 4501  21001  1 22501  21001  1
  501   501   501 
3 3 3
21002  21001  1 2  21001  21001  1 21001  1
  501   501   501
3 3 3
1 (mod 3) si n es par
Nota: 2n  
2 (mod 3) si n es impar

17.3
Primera versión.
Dibujando una cuadrícula 10x10 y tachando manualmente uno por uno todos los múltiplos de
3, vemos que hay 50 múltiplos de 2 de los cuales 16 son múltiplos de 3 luego
50  16 34 17
P  
100 100 50

Segunda versión.
100 
Hay 100 / 2  50 números pares, y hay   16 números divisibles entre 2 y 3, luego hay
 6 
50  16  34 números divisibles entre 2 y no entre 3.
50  16 34 17
P  
100 100 50

17.10
La potencia más alta de 7 que divide a 1000! viene dada por:
1000  1000  1000  1000 
 71   142 ,  7 2   20 ,  7 3   2 , y si k  4  7 k   0

1000 
Luego   k   142  20  2  164
k 1  7 

La potencia de 7 más alta que divide a 500! viene dada por:


 500   500   500   500 
 71   71 ,  7 2   10 ,  73   1 y si k  4  7 k   0

 500 
Luego   k   71  10  1  82
k 1  7 

1000  1000! 1000!


Finalmente, teniendo en cuenta que    
 500  500! 500! 500!
2

1000 
la potencia más alta de 7 que dividirá   será 164  2  82  0 , luego 7 no es un divisor.
 500 

17.11
 200  200! 200! 200 199 198  ...  2 1
n       
 100  100!100! 100! 100  99  98  ...  2 1
2 2 2 2 2 2

200 199 198  ... 102 101



100  99  98  ...  2 1

En el denominador aparecen todos los primos menores de 100 al menos una vez, luego
necesitamos el número primo p menor que 100 que aparezca al menos dos veces en el
200
numerador, es decir, de forma que 3 p  200  p   66
3
El mayor número primo menor que 66 es 61, y 100  2  61  122  200 , y
100  3  61  183  200 , y solo uno de estos dos factores 61 se anulará con el 61 del
denominador.
La solución es 61.
17.12
Estudiamos la parte de la derecha:

      
Dn  2n  1 2n  2 2n  4 ... 2n  2n 1  2n  1 2 2n 1  1 4 2n  2  1 ...2n 1 2  1   
 2  4  ...  2 2  12  12
n 1 n n 1 n2
 1...2  1 
2 2  12  12
1 2  ... ( n 1) n n 1 n2
 1...2  1  2 n ( n 1) / 2
2 
n
12  12  1...2  1
n 1

n2

todos impares

n(n  1)
Luego el factor primo 2 aparece con multiplicidad v2 Dn  
2

Para determinar la multiplicidad del factor 2 en la parte de la izquierda, utilizaremos la fórmula


de Polignac:

k 
v2 (k!)    i 
i 1  2 


k   k 
1
Pero x   x , luego v2 (k!)    i    i  k  i  k
i 1  2  i 1 2 i 1 2

Así que llegamos a


n(n  1) n(n  1)
k! Dn  v2 (k!)  v2 ( Dn )  k  v2 (k!)  v2 ( Dn )  k 
2 2

Por otro lado,


   
Dn  2n  1 2n  2 2n  4 ... 2n  2n 1  2n 2n 2n ... 2n  2n          n
 2n .
2

 n(n  1) 
Demostraremos más adelante que, para todo n  6 , se cumple 2n  
2
!
 2 

y por tanto, para todo n  6 , no existe solución de la igualdad del enunciado, pues tendríamos

n(n  1)  n(n  1) 
k  k!  ! 2  Dn
n2

2  2 

Comprovamos manualmente los valores n  5 :


 
D1  21  20  1 , D2  22  1 22  2  6 , D3  23  1 23  2 23  22  168     
 4

D4  2  1 2  2 2  2 2  2  20160
4
 4 2
 4 3

D5  25  125  225  22 25  23 25  24   999360

1! 1 , 2! 2 , 3! 6 , 4! 24 , 5! 120 , 6! 720 , 7! 5040 , 8! 40320
9! 362880 , 10! 3628800

Y vemos que las únicas soluciones son n  1, k  1 y n  2 , k  3 .

La desigualdad
 n(n  1) 
2n   ! para n  6
2

 2 
Se demuestra de la siguiente manera:
 65
Para n  6 es cierta porque 26  6.9  1010 y  ! 15! 1.3  10
2
12

 2 

Observación:
Esto es lo que aparece en la solución oficial, Pero ¿Cómo se puede saber esto sin
calculadora? En Youtube encontramos el siguiente razonamiento:
 65
 ! 15! 15  14  13  12  11  10  9  8  7  6  5  4  3  2 
 2 
 8 8 8
 8
 8
 8
 8
 8  7  6  5  4  3  2  238  24  32  5  7 
8 8  2 38

 238  24  23  25  224 4  3 5  236  26


2

Para n  7 tenemos
 n(n  1)  n(n  1)
 ! 15!16  17  ...   236  16n ( n 1) / 2 15  236  24( n ( n 1) / 2 15)  236  24( n ( n 1) / 2 15) 
 2  2
 2 n  24  n 2  2 n  24  2 n  24
 22 n  2n  2n  2n  2n
2 2 2 2 2

17.13
Puesto que 23 es primo, 236  x | 2000! 2000  1999  1998  ...  3  2  1 si y solo si 23 divide a
algún número de este producto.
2000  23  86  22 , luego hay 86 números divisibles entre 23, de los cuales, puesto que
86  23  3  17 hay 3 divisibles entre 232 , así pues, en el número 2000! encontramos
86  3  89 veces el factor 23 .
Así pues, 6  x  89  x  86 .

17.14
n2 1 ! 
en un entero si y solo si n! | n 2  1 .
n
 
n!n

Siguiendo con las definiciones introducidas en 17.4 sobre orden p-ádico de un número,
tenemos que
v p (a)  v p (b)  a | b

Luego vamos a ver para qué enteros n entre 1 y 50 se cumple:


v p n !  v p n 2  1 !
n
   
Por otro lado, se cumple
v p a b  b  v p a 

Así pues, queremos comprobar la desigualdad


n  v p  n !  v p n 2  1 !   (*)
Podemos reducir nuestro estudio a las potencias de números primos.

Primer caso: Si n  p , un número primo. Aplicando la fórmula de Polignac:


 n 2  1
 

v p n2 1 !      ( p  1)  0  ...  0  p  1
k 1  p 
v p n!  1  0  0  ...  1 (ver 17.14b)

Y por lo tanto la desigualdad (*) se convierte en


p  p 1

Lo cual no es cierto para ningún p.

Segundo caso: Si n  p 2 , el cuadrado de un primo. Aplicando la fórmula de Polignac:


   
v p n 2 1 !  v p p 4 1 !  p 3  p 2  p  3
v p n!  p  1 (ver 17.14c)

Y por lo tanto la desigualdad (*) se convierte en


p 2  p  1 p 3  p 2  p  3  0  p  3  3  p

Lo cual es cierto para todo primo excepto el 2.

Caso general: Se puede demostrar que para todo n  p i con i  2 se satisface la desigualdad
(*)

Así pues, hay 16 números para los cuales no se verifica la desigualdad, y 50  16  34 números
para los que sí se satisface (D).

Fuente de esta solución: https://artofproblemsolving.com/wiki/index.php?title=2019_AMC_10A_Problems/Problem_25&oldid=132200

18.1
Por el TDB, (m, n)  am  bn para ciertos enteros a, b . Luego
(m, n)  n  am  bn  n   m  n  m n  n
       a  b    a    b 
n  m n  m  n  m  n  m  m

m n
Con lo que nuestro problema se reduce a demostrar que   es un entero.
n  m 

n  n 1
Ahora aplicamos la igualdad m   n  (Ver Tema 2 de PC)
 m  m  1

m  n  1  n  1  n 1  n 1
   m   n   que es un número combinatorio, y por tanto entero.
n  m  n  m  n  m  1  m  1
Fuente de la solución: Number Theory Concepts and problems (Andreescu, Dospinescu… 2017) pág. 65

20.5
Basta aplicar n  p1a1 p2a2 ... prar   (n)  a1  1a2  1...ar  1
 (n) es impar  ai  1 es impar 1  i  r  ai es par 1  i  r  ai  2bi 1  i  r

 n  p1a1 p2a2 ... prar  p12b1 p22b2 ... pr2br  p1b1 p2b2 ... prbr 
2

20.6
Está claro que si n  p q1 con p, q primos, entonces  (n)  q  1  1  q es primo.
Sea n  p1a1 p2a2 ... prar y supongamos que  (n)  a1  1a2  1...ar  1 es primo. Entonces está
claro que r  1 . Luego
n  p a y  (n)  a  1 primo, y podemos escribir n  p ( n )1 con  (n) primo, como queríamos.

20.7
1099  2  5  299  599
99

El número total de divisores es (99  1)(99  1)  1002  10000


De estos divisores, los múltiplos de 1088 serán aquellos de la forma 2a5b , con 88  a, b  99 , es
decir, 12 12  144 números.
144 6
Luego la probabilidad es 
10000 625

20.8
Puesto que 6 solo se factoriza como 2  3 o 6 1, el número n solo puede constar de dos factores
primos:
a  1  2
n  p a qb    a  1, b  2  n  p  q con p  q .
2

b 1  3
O bien:
n  p a ,  a  1  6  a  5  n  p5

20.9
1010  2  5  210 510 , y sus divisores son todas las combinaciones posibles de la forma
10

n  2a5b con 0  a, b  10 . Hay 112  121 en total.

157  3  5  37 57 , y sus divisores son todas las combinaciones posibles de la forma


7

n  3a5b con 0  a, b  7 . Hay 82  64 divisores posibles.


Pero los divisores de la forma n  305b  5b con 0  b  7 ya aparecen como divisores del
primer número, por lo tanto los restamos. En total hay 64  8  56 divisores.

 
11
1811  2  32  211 322 , y sus divisores son todas las combinaciones posibles de la forma
n  2a3b con 0  a  11 y 0  b  22 . Son 12  23  276

Pero de estos están repetidos aquellos de la forma


n  2a30 con 0  a  10 : 11 divisores, y aquellos de la forma
n  203b con 1  b  7 : 7 divisores. (¡Atención! No contemos el divisor 1 dos veces)
Luego hay 276  11  7  258 nuevos.

Así pues, hay un total de 121  56  258  435 divisores.

20.10
Para n  1 ,  (1)  1 ,  (2)  2 y está claro que no cumple la condición. Luego n  2 , y por tanto
 (n) ,  (n  1)  2 y en consecuencia  (n)   (n  1)  7   (n) ,  (n  1)  5 .

 (n)  1  n  1 y ya hemos visto que no cumple la igualdad del enunciado.

Luego las posibilidades que quedan son:


a)  (n)  2 , (n  1)  5
b)  (n)  3 , (n  1)  4
c)  (n)  4 , (n  1)  3
d)  (n)  5 , (n  1)  2

Teniendo en cuenta que:


 ( n)  2  n  p
 ( n)  3  n  p 2
 ( n)  5  n  p 4
n  p  q, p  q
 ( n)  4  2  2  
n  p
3

Tenemos
a) n  p , n  1  p 4
b) n  p 2 , n 1  p q
n  p 2 , n  1  p3
c) n  p 3 , n  1  q 2
n  p q , n  1  q2
d) n  p 4 primo y n 1  q primo.

Observamos que en todos los casos, uno de los dos números consecutivos es la potencia par de
un número primo, y con esto vamos probando casos:
n  3 , n  1  4   (3)  2 ,  (4)  3
22  4   ninguna cumple.
n  4 , n  1  5   (4)  3 ,  (5)  2
n  8 , n  1  9   (8)  4 ,  (9)  3
32  9   ambas cumplen la condición del enunciado.
n  9 , n  1  10   (9)  3 ,  (10)  4
n  15 , n  1  16   (15)  4 ,  (16)  5
24  16   cumple la segunda.
n  16 , n  1  17   (16)  5 ,  (17)  2
n  24 , n  1  25   (24)  8 ,  (25)  3
52  25   cumple la segunda.
n  25 , n  1  26   (25)  3 ,  (26)  4
n  48 , n  1  49   (48)  10 ,  (49)  3
7 2  49   ninguna cumple.
n  49 , n  1  50   (49)  3 ,  (50)  6
n  80 , n  1  81   (80)  10 ,  (81)  5
34  81   ninguna cumple.
n  81 , n  1  82   (81)  5 ,  (82)  4
n  120 , n  1  121   (120)  16 ,  (121)  3
112  121   cumple la segunda.
n  121 , n  1  122   (121)  3 ,  (122)  4
n  168 , n  1  169   (168)  16 ,  (169)  3
132  169   ninguna cumple.
n  169 , n  1  170   (169)  3 ,  (170)  8
n  288 , n  1  289   (288)  18 ,  (289)  3
17 2  289   ninguna cumple.
n  289 , n  1  290   (289)  3 ,  (290)  8
n  360 , n  1  361   (360)  24 ,  (361)  3
192  361   cumple la segunda.
n  361 , n  1  362   (361)  3 ,  (362)  4

Y paramos porque hemos conseguido las seis soluciones pedidas. El resultado es


8+9+16+25+121+361=540

Fuente de esta solución: www.artofproblemsolving.com/wiki/index.php/2019_AIME_I_Problems/Problem_9

20.11

2004  22  3 167  20042004  22  3 167 2004
 24008  32004 167 2004 .

Los divisores de 20042004 son todos los números de la forma

d  2a  3b 167c , con 0  a  4008 , 0  b  2004 , 0  c  2004 .

Luego  d   a  1b  1c  1 , y por tanto

 d   2004  a  1b  1c  1  22  3 167

O equivalentemente, todas las posibilidades x, y, z con x y z  22  3 167 y


1  x  4009 , 1  y  2005 , 1  z  2005 .

Colocamos primero los "2": Hay 6 formas diferentes.


2–2-/
2-/-2
/-2–2
22 - / - /
/ - 22 - /
/ - / - 22

Ahora tenemos que colocar el 3 y el 167, en cualquiera de las tres posiciones, luego el total será
6  3  3  54 formas diferentes.

20.12
Sean J, C y Z las edades respectivas de Joey, Chloe y Zoe.
Tenemos que J  C  1 y Z  1
En cada aniversario tenemos la siguiente pauta:

Zoe 1 2 3 4 Z
Chloe C C+1 C+2 C+3 C+Z-1
Joey C+1 C+2 C+3 C+4 C+Z

Está claro que C es múltiple de 1, y existirán ocho números más Z1 , Z 2 ,..., Z8 tales que

Z1 | C  Z1  1 , Z2 | C  Z2  1, ..., Z8 | C  Z8  1

Puesto que Zi | Zi , entonces está claro que Zi | C  1 , es decir, el número C  1 tiene


exactamente ocho divisores diferentes de 1. Luego C  1 tiene nueve divisores contando el 1.
Aplicando 20.4, tenemos que
C  1  p1a1 p2a2 ... prar
entonces
9  a1  1a2  1...ar  1

Con un solo factor primo sería imposible, pues como mínimo tendríamos el resultado
C  1  28  256 , una edad absurda.

Con dos factores primos, la única posibilidad aceptable para una edad es
9  2  12  1  C  1  2232  36

Y por tanto C  36  1  37  J  C  1  38

Zoe 1 2 3 4 5 Z
Chloe 37 38 39 40 41 ...
Joey 38 39 40 41 42 C  37  Z

Nos piden determinar el siguiente Z tal que Z | J  Z | 37  Z  Z | 37 , pero puesto que 37 es


primo, la única posibilidad es Z  37  J  37  37  74 , y la respuesta correcta es 7  4  11
(E).

20.13
1 1 1 yx 1 xy
      N  xy  N ( x  y ) 
x y N xy N x y
xy  Nx  Ny  0  ( x  N )( y  N )  N 2  0  ( x  N )( y  N )  N 2

Que esta ecuación tenga exactamente 2005 soluciones es una forma de decir que existen
exactamente 2005 formas diferentes de escribir N 2 como producto de dos enteros positivos, es
decir, que N 2 tiene exactamente 2005 divisores, pues existe una biyección clara entre los
divisores de un número y sus expresiones como producto:
d  d  (N / d )
Ahora aplicaremos el problema 20.5: El número N es un cuadrado perfecto si y solo si  (N ) es
impar.
Si la factorización canónica de N es N  p1a1  p2a2  ...  pnan , la factorización canónica de N 2 es
 
N 2  p12a1  p22a2  ...  pn2an , y por lo tanto  N 2  (2a1  1)(2a2  1)...(2an  1)  2005

Por otro lado, 2005  5  401 , luego la única formas posibles son:
a) a1  2 y a2  200 , y por tanto  ( N )  (a1  1)(a2  1)  3  201  603 , un número impar, y por
tanto N es un cuadrado perfecto.
b) a1  1002  N  p110021   ( N )  1003 impar, luego es un cuadrado perfecto.

21.5
1815  3  5 112
Sabemos que n  2a3b para ciertos enteros positivos a, b . Sabemos que entonces
2a 1  1 3b 1  1 1 a 1
 ( n) 
2 1

3 1
 
 2  1 3b 1  1
2

A falta de algo mejor, vamos viendo los diferentes valores que se obtienen:
k 2k 1  1 3k 1  1
1 3 8
2 7 26=2·13
3 15=3·5 80=24·5
4 31 242=2·112
5 63=3·31 728=23·7·13

Vemos que la única combinación que se adapta a nuestro problema es a  3 y b  4 :

2

1 31
 
1
2  1 34 1  1  3  5  2  112  1815 . Así pues, n  2334  8  81  648 .
2

También podría gustarte